Вы находитесь на странице: 1из 171

PREFACE

NTS, a project of its own kind is aimed at setting standards of excellence in educational testing. NTS is a rapidly growing and credible entity of national repute. Its context and potential coincides with the expressed need of the Government for the establishment of a National Accreditation Council and Testing Service. Establishment of this set-up has been necessitated by the prevailing lack of credibility and absence of standardized testing facilities in our national Institutes. Institutions waste massive amount of work hours in preparing and conducting diverse kind of entrance tests with least credible and authentic consequences. NTS has made a bid to restore some order to the growing chaos in the field of testing. It has developed the capability to offer the testing services to all national institutes providing education. Its content is aimed at identifying the ability-oriented strengths among prospective candidates for IT education. NTS is a multifaceted and comprehensive facility that can provide services at an enormous scale in all dimensions of testing. Based on a vision aimed at providing quality services in educational testing and assessment mechanisms for various stakeholders, NTS is committed to promoting the overall quality of education as a value-added adjunct. It also extends Services for the employed & unemployed professionals to get satisfactory jobs in their relevant fields through an assessment of their knowledge base, skills, and professional expertise & abilities. NTS is a unique and trend-setting venture that has shown great promise.

Student Guide

NTS INTRODUCTION

NTS was formed in 2001 with the basic objective of establishing a credible testing system to evaluate the knowledge level of students against a consistent and standard yardstick. It was felt that although our universities and other educational institutions are producing a large number of qualified professional and nonprofessional manpower, there is no single standard for quality evaluation of this educated human resource. Since there is no single syllabus right from basic to the advanced levels of education, the normal performance evaluation procedures fail to measure the quality as well as the extent of discrepancy between the existing and the desirable quality for individual candidates. Similarly, a big variation in syllabi and the apparent status of the educational institutions have created a huge gap between the haves and have-nots of the society. The students from big name institutions always have an added advantage where mediocre students are preferred over excellent students from low status institutes because of the brand name of the institutions they carry with them. NTS addresses these important issues by providing testing procedures that evaluate the knowledge and skills of individuals on a unified scale using standardized measures that are independent of class differentiation. In a short period of three years, NTS has made a significant contribution by developing and implementing training and testing pro grams for individual students, educational institutions and the potential employers of the country. The hallmarks of the NTS system are its consistency, transparency, reliability and efficiency. Being a non-profit organization, it takes pride in offering a cost-efficient system that could meet the needs of individuals and organizations to their utmost satisfaction. NTS continues to strive for excellence through research and development in its testing system and related services. The goal of the organization is to keep contributing towards improvement in education by building standards for educational and professional testing. Its ultimate aim is to be recognized nationally and internationally as a credible and reliable name in the field of testing.

1.1

NTS BACKGROUND

T h e r e i s a n e v e r-increasing demand of quality professionals around the globe. Our universities and other educational institutions are producing large quantities of professionals , y e t t h e r e a p p e a r s t o b e n o s i n g l e s t a n d a r d f o r the p e r f o r m a n ce e v a l u a t i o n o f t h e s e p r o f e s s i o n a l s a n d t h e i r q u a l i t y a s s u r a n c e . S i n c e t h e r e isnt a s i n g l e s t a n d a r d s y l l a b u s , t h e p e r f o r m a n c e e v a l u a t i o n procedures fail to measure the quality as well as the extent of discrepancy b e t w e e n t h e e x i s t i n g a n d t h e d e s i r a b l e q ua l i t y o f i n d i v i d u a l c a n d i d a t e . This situation is raising a lack of confidence among the candidates and the entrepreneurs. To alleviate this, there is an immediate need for incorporating an autonomous a n d a s o v e r e i g n b o d y , w h i c h s h o u l d c a t e r t o t h e n e e d s o f the entrepreneur and harness the individual talent. This body should be made responsible for bridging the gap between the education industry and the practical/professional industry by providing an evaluation and testing standard . This Non G o v e r n m e n t O rganization (NGO), under the name of NTS National Testing S e r v i c e f a c i l i t a t e s t h e e v a l u a t i o n o f g r a d u a t e o r u n d e r g r a d u a t e candidates for a d m i s s i o n t o a n y university. NTS will not only lead the e d u c a t i o n a l industry to

Student Guide

a healthy competition but i n s t i t u t i o n s i n a l l field s .

also

i m p r o ve

the

standards

of

the

educational

1.2.

NTS Objectives

Within the overall premise of contributing to the improvement of the quality of education in Pakistan, the important objectives of NTS include: T o i m p r o v e u p o n t h e s h o r t c o m i ngs in our prevailing performance evaluation system of candidates through establishing a credible, technically appropriate and accurate system of performance evaluation for students of different educational institutions at various levels. To provide a sta ndardized and uniform criterion for selecting candidates seeking admission to various universities in the country and abroad and also to provide a basis on which students, individuals and institutions can a s s e s s p e r f o r m a n c e f o r p r e p a r a t i o n o f v a r i o u s t y p e s of tests conducted by different international testing agencies. E n a b l i n g assessment and ranking not only in a particular subject on a general competency level but also help in identification of areas of s t r e n g t h s a n d w e a k n e s s e s i n i t s s u b -c o n t e n t s o n a d i s a g g r e g a t e d b a s i s . To provide a reliable and efficient system for evaluating the knowledge b a s e a n d s k i l l s o f c a n d i d a t e s i n c l u d i n g p r o f e s s i o n a l s - employed and unemployed. E s t a b l i s h i n g a credible and reliable measure of assessing the students k n o w l e d ge i n c o m m o n l y t a u g h t p r o g r a m s f o r r e l a t i v e r a n k i n g . To bridge the gap between academic preparation and the practical market need s i n a h i g h l y d y n a m i c a n d c o m p e t i t i v e e n v i r o n m e n t .

1.3

NTS COMMITMENT

National Testing Service (NTS) is a non profit, autonomous organization established with the basic objective of developing a credible, technically appropriate and accurate system to evaluate the knowledge level of students of different educational institutions at various levels. It thus assists in carrying out an estimated ranking of students for admissions in various educational institutions and also provides potential employers with a credible yardstick for evaluation and induction of quality human resource. NTS operates on national level and is committed to:

1.3.1

Promoting Overall Quality of Education

Based on a vision aimed at providing quality Services in educational testing a n d a s s e s s m e nt mechanisms for various stakeholders, NTS is committed to p r o m o t i n g t h e o v e r a l l q u a l i t y o f e d u c a t i o n a s a v a l u e -a d d e d a d j u n c t .

1.3.2

Performance Evaluation

NTS undertakes programs that are aimed at establishing a credible, technically appropriate and accurate system of performance evaluation of students/candidates of various educational institutions at different levels.

1.3.3

Assessing Knowledge, Skills & Professional Abilities

Student Guide

It also extends services for the employed and unemployed professionals to get satisfactory jobs in their relevant area through an assessment of their knowledge base, skills, and professional abilities.

1.3.4

A Rapidly Growing & Credible Entity

NTS is a rapidly growing and credible entity of national repute. Its context and potential coincides wit h the expressed need of the Government for the establishment of a National Accreditation Council and Testing Service.

1.3.5

Conducting Various Types of Tests

While the beginnings were made by conducting General Tests through an ITfocused thrust, NTS has expand ed its services to other disciplines of educational testing. It now provides services for conducting Subject Tests by providing facilities both for individual candidates and institutions. These tests are prepared and assessed by experts and professionals from relevant fields.

1.3.6

Linkages & Networks

NTS now has growing linkages and associations with educational and professional organizations & Institutions, including those engaged in talent s e a r c h a n d c a r e e r & e d u c a t i o n a l p r o m o t i o n a c t i v i t i e s , a t b o t h n a t i o n a l and international levels.

1.4
1.4.1

NTS- BENEFITS
Students

NTS aims at being beneficial for both students and institutes.

NTS Tests enable students to draw benefits in a number of ways: It provides a single standard for measuring knowledge base and also skill le v e l e v a l u a t i o n o f c a n d i d a t e s . Candidates can have a more transparent, credible and standardized assessment of their knowledge. Besides being economical, NTS declares the test results in just a few days time. N T S s c o r e s a r e a r e l i a b l e s o u r c e t o m e a s u r e candidates knowledge in commonly taught academic courses. T e s t i n g t h r o u g h N T S a u t o m a t i c a l l y l e a d s t o s t a n d a r d i z a t i o n a t a national level, which eliminates chances of large dropouts during the course of education.

1.4.2

Institutions

Besides students, NTS is als o u s e f u l f o r t h e e d u c a t i o n a l i n s t i t u t e s a n d s i m i l a r entities: Institutes, wishing to get admission tests prepared for their candidates, and also wanting to get them scored in a more transparent and in a less time consuming manner can rely on NTS. T h i s te s t i n g s e r v i c e i s h i g h l y r e l i a b l e , e f f i c i e n t a n d s t a n d a r d i z e d ; s o a n y o f the institutes can share their responsibility with NTS. The preparation and holding of tests consume a lot of time and effort. The institutes can lessen this burden of responsibility of conducting tests by sharing it with NTS. Preparation of registration forms, roll number slips, question papers, answer sheets, satisfactory arrangements for invigilation, paper marking, result declaration etc. are arranged by NTS. T h e I n s t i t u t e s c a n totally rely on NTS for efficiency of Services including transparency of results.

Student Guide

The Institutes are provided with a credible system of performance ranking.

1.5

NTS SERVICES
National Aptitude/General Testing

NTS provides its services in vast areas:

1.5.1

NTS c o n d u c t s G e n e r a l T e s t s s o a s t o e v a l u a t e t h e c r i t i c a l t h i n k i n g s k i l l s o f analytical reasoning and also performance assessment of candidates with regard to quantitative & verbal ability. General Tests also called National Aptitude Tests, help establishing a reliable and credible measure of judging the knowledge base of students in commonly taught programs.

1.5.2

Services - Measuring Knowledge Base and Skill Levels

The NTS Subject Tests are prepared and designed with a view to assessing t h e q u a l i f i c a t i o n s , k n o w le d g e b a s e , c o m p e t e n c y a n d s k i l l l e v e l o f t h e candidates in a specific area of study.

1.5.3

Services for Test Preparation

N T S p r o v i d e s c o m p l e t e g u i d e l i n e s w i t h r e g a r d t o T e s t P r e p a r a t i o n . A guide f o r u s e r s i s a v a i l a b l e o n o u r w e b s it e a s w e l l a s i n t h e m a r k e t i n p r i n t e d f o r m . Model Questions and Sample Tests are also available on the website for helping the candidates to practice and be well prepared for the test. Furthermore Reference Books are also recommended by NTS to be a n additional source of help for the test takers for computer Science Subject Tests.

1.5.4

Administrative Services

NTS provides an administrative service for not only the candidates but also the institutions. All administrative arrangements regarding registration, test c e n t e r s e l e c t i o n , t e s t e x e c u t i o n , r e s u l t p r e p a r a ti o n a n d t h e n i t s r e p o r t i n g e t c , are taken as a responsibility by NTS.

1.5.5

Admission Test Services for Institutions

NTS conducts Admission Tests for various institutes and institutions, a ll services like the arrangements of test registra tion, paper generation, test execution, paper marking, result declaration etc. are pro vided by NTS for these entities.

1.5.6

NTS also conducts Online Tests for the candidates efficient and credible e v a l u a t i o n o f t h e i r k n o w l e d g e a n d skills. A r r a n g e m e n t s l i k e t h e a v a i l a b i l i t y of labs, content development for the assessment, online paper generation and test execution and online result formation immediately after the test, are made by NTS.

Services for Online Testing

1.5.7

Establishing Performance Ranking

NTS also assists in carrying out an estimated ranking of students as well as professionals. This ranking is not only in a particular subject on a general competency level but also helps in the identification of areas of strengths and weaknesses in its sub-contents on a disaggregated basis.

Student Guide

1.5.8

Services for Score Reporting

F o r G e n e r a l T e s t s , N T S d e c l a r e s i t s r e s u l t s e l e c t r o n i c a l l y w i t h i n s e v e n d a y s of t h e t e s t b e i n g c o n d u c t e d , t h e r e s u l t s o f e a c h c a n d i d a t e are s e n t t o t h e i r h o m e s t h r o u g h a f a s t a n d r e l i a b l e c o u r i e r s e rv i c e . The result of Online Subject Tests is displayed online immediately after the candidate completes his/her test. The score of candidates is also reported in the form of a certificate at their respective addresses.

1.5.9

Services to Facilitate Employment

NTS extends its services for employed and unemployed candidates to get satisfactory jobs in their relevant areas through a very estimated and careful judgment of their knowledge base and skills. I t f a c i l i t a t e s a c c e s s t o j o b o p p o r t u n i t i e s f o r v a r i o u s p r o fe s s i o n a l s a n d candidates both within the country and abroad on the basis of their scores in NTS Subject Tests.

1.5.10 Statistical Analysis and Reports


N T S by virtue of it s f l e x i b l e a n d u s e r f r i e n d l y s o l u t i o n c a n b e t t e r h e l p t h e i n s t i t u t i o n s a n d o r g a n i z a ti o n s t o h a v e s t a t i s t i c a l a n a l y s i s a n d r e p o r t f o r administrative tasks and it will ultimately lead to true decision making and managerial tasks.

1.5.11 Survey
NTS has very robust and accurate system for processing forms and surveys. I n a d d i t i o n t o t h e e d u c a t i o n a l a n d p r o f e s s i o n a l t e s t i n g s e r v i c e s N T S h a s the survey facility for different organizations, government data collection and other methods to get feedback from people.

1.6
1.6.1

NTS PARTNERS
COMSATS Institute of Information Technology

COMSATS, through a notification in 1979, setup the COMSATS Institute of Information Technology (CIIT), which became functional in April 1998. CIIT is a world - c l a s s t e a c h i n g i n s t i t u t e , c r e a t i n g k n o w l e d g e b a s e a n d a d v a n c e d t e c h n o l o g i e s t h a t a r e i n t e n d e d t o s h a p e t h e n e w d ir e c t i o n o f t h e c o u n t r y a n d educate its future leaders for the 21st century. The aim of the Institute is to train the countrys human resource in the field of Information Technology so as to equip them with emerging IT trends. The C I I T i s f u l l y e q u i p p e d t o c o p e w i t h t h e n e e d s o f a g r o w i n g a n d f a s t -paced IT industry , flexible enough to adapt to the challenges of the future. The CIIT has a mission to deliver new ideas and products through research, development and education in strategic partnership with IT industry and organizations. C I I T h a s a t o t a l o f f i v e c a m p u s e s a c r o s s t h e c o u n t r y , t w o i n I s l a m a b a d , one each in Wah, Abbottabad and Lahore.

Online T e s t
N T S i s c u r r e n t l y c o n d u c t i n g o n l i n e e x a m (2 0 %) for t h e s t u d e n t s a t C I I T Abbottabad and it will likely to be adopted in all the campuses in near future. Students while taking this kind of test can have a better vision and from

Student Guide

academic and teaching point of view NTS can prepare different statistical a n a l y s i s a n d r e p o r t s t o i m p r o v e t h e q u a l i t y o f e d u c a t i o n , p erformance and

knowledge.

Admission Tests for CIIT


National Testing Service (NTS) has been conducting Admission Tests for CIIT a c r o s s all i t s c a m p u s e s s i n c e J u l y 2 0 0 2 . T h e G e n e r a l T e s t s a r e c o n d u c t e d f o r a d m i s s i o n s a t b o t h G r a d u a t e a n d P o s t -G r a d u a t e l e v e l s c a t e r i n g f o r c a n d i d a t e s from different educational backgrounds.

1.6.2

Virtual University

Virtual University (VU) is one of the priority projects of the Ministry of Science and Technology (MoST), which participates as a key contributor in t h e s o c i o -e c o n o m i c d e v e l o p m e n t o f t h e c o u n t r y . V U i s b a s e d o n a n e x c e l l e n t telecommunication infrastructure and provides educational facilities on a nationwide basis.

NTS Conducts Admission Tests for VU


N T S c o n d u c t e d a d m i s s i o n t e s t s ( N A T -I S ) a t v a r i o u s P r i v a t e V i r t u a l C e n t e r s (PVCs) of VU in August 02. To facilitate the conduct of these Tests, NTS provided Services on a countrywide basis simultaneously in 135 centers spread over 51 major cities of Pakistan. On behalf of the satisfactory and successful services provided by NTS, Virtual University reposed its complete trust in the authenticity and credibility of NTS initial testing for the university which they reconfirmed by their pledge to continue availing the services of NTS for any further testing. We certainly v i ew t h i s w i t h g r e a t a p p r e c i a t i o n a n d e n c o u r a g e m e n t .

1.6.3

Higher Education Commission (HEC)

The Higher Education Commission has been setup to facilitate the d e v e l o p m e n t o f the u n i v e r s i t i e s o f P a k i s t a n t o b e come w o r l d -c l a s s c e n t e r s o f education, research and deve lopment. A critical additional challenge is to v a s t l y i n c r e a s e a c c e s s t o h i g h e r e d u c a t i o n i n P a k i s t a n s o t h a t o u r talented people are not denied avenues for personal and professional growth. The H E C i n t e n d s t o p l a y i t s p a r t i n s p e a r h e a d i n g t h e b u i l d i n g of a k n o w l e d g e b a s e d e c o n o m y i n P a k i s t a n . P e o p l e a r e t h e r e a l a s s e t s o f P a k i s t a n h o w e v e r it is only through education and research that our country can profit from this g r e a t a s s e t . A w e l l -e d u c a t e d g r a d u a t e i s t h e b u i l d i n g b l o c k o f a k n o w l e d g e b a s e d e c o n o my , a n d i t i s f o r t h i s r e a s o n t h a t t h e H E C i s f o cu s i n g o n t h e i s s u e of quality of higher education.

C o n d u c t i n g S c h o l a r s h i p Tests f o r HEC
In February 2003 NTS successfully conducted Subject tests for various areas of study across Pakistan on behalf of the Higher Education Commission and since then all the scholarship tests (Subject/General) offered by HEC are carried out by NTS .

1.6.4

Pakistan Telecommunication Corporation Limited (PTCL)

Pakistan Telecommunication Corporation (PTC) set sails for its voyage of glory in December 1990, taking over operations and functions from Pakistan Telephone and Telegraph Department under Pakistan Telecommunication Corporation Act 1991. This coincided with the Government's competitive

Student Guide

policy, encouraging private sector participation and resulting in award of l i c e n s e s f o r c e l l u l a r , c a r d -o p e r a t e d p a y p h o n e s , p a g i n g a n d , l a t e l y , d a t a communication services. Since then, PTCL has been working vigorously to meet the dual challenge of t e l e c o m d e v e l o p m e n t a n d s o c i o -e c o n o m i c u p l i f t o f t h e c o u n t r y . T h i s i s characterized by a clear appreciation of ongoing telecom scenario wherein convergence of technologies continuously changes the shape of the sector. A measure of this understanding is reflected by progressive measures such as e s t a b l i s h me n t o f t h e c o m p a n y ' s m o b i l e a n d I n t e r n e t s u b s i d i a r i e s i n 1 9 9 8 . The telecom sector of Pakistan has also entered into a new era. The monopoly of PTCL is now over and the sector is moving towards liberalization. While liberalization and competition are synonymous, the future scenario also poses some challenges to the market dominance of PTCL. The Company is ready to face this challenge and maintain its dominating position, while guarding its revenue streams in the face of forthcoming competition.

Recruitment Test for PTCL


Being an elite client, PTCL uses the services of National Testing Service [ N T S ] f o r h i r in g p e o p l e f o r t h e o r g a n i z a t i o n . A r o u n d t h o u s a n d s o f c a n d i d a t e s apply for jobs in PTCL through the testing services of NTS.

1.6.5

Bolan Medical College


education in the province of

Bolan Medical College as seat of medical Balochistan has a remarkable history.

Admission Test for BOLAN Medical College


NTS has broadened its services and has also conducted admission test for Bolan Medical College Quetta, the conduct and outcome of which was to the entire satisfaction of all candidates. The students could tally their score with the key displayed afterwards.

1.6.6

Engineering University Khuzdar

N T S d o e s n o t l i m i t i t s s e r v i c e s t o a s p e c i f i c e d u c a t i o n a l g r o u p b u t has also provided its services for the conduct of Admission Tests at Engineering U n i v e r s i t y Kh u z d a r.

1.6.7 Federal Urdu University of Arts, Science and Technology, Islamabad


The government has established a Federal Urdu University of Arts, Sciences and Tech n o l o g y I s l a m a b a d t h r o u g h a n o r d i n a n c e w i t h U r d u t o r e m a i n t h e main language of instruction and teaching. The university will have its principal seat at Islamabad and will have the Federal Government Urdu Science College, Karachi, and Federal Government Urd u Arts College, Karachi, as constituent institutions. Under the rule, the university will have academic, financial and administrative autonomy, including the power to employ officers, teachers and other employees while the university will be controlled by a senate, a syndicate and an academic council.

Recruitment Test for Urdu University

Student Guide

The National Testing Service [NTS] provided its services to the University fo r t h e h i r i n g p u r p o s e s o f t h e i r fa c u l t y m e m b e r s i n e i g h t s u b j e c t s t h u s m a k i n g the selection tra nsparent.

1.6.8 Board of Intermediate and Secondary Education, Abbottabad


Recruitment Test
Abbotta bad Educational Board signed up the services of NTS for hiring employees. A Computer Science subject test was conducted for testing the academic skills of the candidates. The assessment was later used to determine the merit for final selection.

1.6.9 Others

NT S n o w h a s g r o w i n g l i n k a g e s a n d a s s o c i a t i o n s w i t h m a n y e d u c a t i o n a l a n d professional organizations and institutions, including those engaged in talent search , career and educational promotiona l activities, at both national and international levels. As a rapidly growing and credible entity of national repute, the context and potential of NTS coincide with the expressed need of the Government for the establishment of a National Accreditation Council and Testing Service.

1.7

NTS Tests

N T S g e n e r a l l y c o n d u c t s t w o kinds of tests: General Tests Subject Tests

T h e s e a r e c o n d u c t e d b o t h a s c o m p u t e r- a i d e d , p a p e r b a s e d a n d o n l i n e t e s t s , a s r e q u i r e d.

General Tests
NTS conducts different undergraduate students; types of General Tests for the graduate and

The General Test comprises Quantitative, Analytical Reasoning and Verbal Sections for assessment.

Subject Tests
Subject Tests are designed to evaluate the educational skills in a specific area of study. So far NTS has conducted the Subject Test in different s u b j e cts:

1. 2. 3. 4. 5. 6. 7.

Physics Electronics/Engineering Biology Chemistry Pharmacy Computer Science Mathematic s

Student Guide

8. 9. 10. 11. 12. 13. 14. 15. 16. 17. 18. 19. 20.

Telecommunication Biochemistry Management Science Health Sciences Environmental Science Earth Science Statistics Veterinary Science Economics Agric ultural Science Islamic Studies Pakistan Studies English

1.8

NTS TEST CENTERS

NTS Head Office is in Islamabad, it conducts tests across Pakistan, for any Institute or institution requesting our services, providing sufficient time is provided for preparation.

1.9

SCHEDULE AND INFORMATION

There are several sources for obtaining information and schedules on the NTS Tests.

i ii iii iv

A d v e r t i s e m e n t s t h r o u g h P r i n t e d Media Advertisements through Electronic Media NTS Allied Institutions NTS and its Allied Institutions' Website

1.10 CONTACT NTS


For Tests and Test Center queries related to Score Reporting, and also for other reasons like putting complaints or providing Feedback, you may: Visit NTS at its website i.e. http://www.nts.org.pk Write to NTS at Secretary National Testing Service (NTS) O f f i c e # 1 0 11, First Floor Plaza 2000 P l o t # 4 3 , M a r k a z I- 8 , I s l a m a b a d . Email D i r e c t o r N T S at directornts@ciit.net.pk directornts@nts.org.pk ntscoord@ciit.net.pk support@nts.org.pk

Email NTS at

Student Guide

10

TECHNICAL REVIEW

Student Guide

11

2.

Technical Procedures and Operations

2.1 Content Collection


As NTS is an autonomous body following a set of SOPs, trying to mainta in the National and International state of the art technology and standards. Keeping this vision in mind, NTS team would like to share the most important and central activity that is how the contents are being gradually increased, updated, the template upon which questions are being collected and after carefully collecting the questions how quality is controlled. Template creation and paper generation for individual tests is flexible and fair enough to meet the requirements of different clients and produces results is different scenarios reliably. NTS test engine and paper processing mechanisms are matured and error prone and we are still trying to introduce new features. NTS is highly conscious about the sensitive issue of contents. To date, NTS has conducted

The Subject Test in several subjects and has formulated Subject Committees for each of those subjects. The committee members consist of subject specialists from various institutes and organizations, providing valuable input in building up an authentic and reliable database. Subject Committees These committees are made for each subject i.e. Engineering, Biology, Physics etc., comprising of a Chairperson being a PhD doctor and at least four members who have a Masters Degree. Making curricula, policies, identifying other subject experts for the Review and Question Making Committee are all responsibilities of this working group. Other committees formulated by the Subject Committee are: i. Review Committees ii. Question Making Committees
NTS operations and methodology for content collection and improved quality is very secure and protected. We put the contents in encrypted/coded format into the central database along with other precautions to make the system safe and fool proof. NTS has huge central database in different areas of study. The method of question collection is of two types.

Paper Base Content Collection Online Content Collection

2.1.1 Paper Based Content Collection


For paper based content collection a template has been designed with some field s and attributes, and information required are associated with each question. Some are narrated Education Level of the Question Difficulty Level of the Question (Easy, Moderate, Difficult) Reference Information As the question can be obtained from the book, own creativity (self-made), website etc., in any case the additional information is required. As an example in case a question is picked from the book the following information will be linked with the question Title & edition of the referenced book Publisher of the referenced book Topic & subtopic the questions belong to Page No. of the Book.

Student Guide

12

Field
Subject: Section: Branch/Topic Sub- Topic Type: Statement: o Conceptual o Practical o logical

Difficulty Level: Max/Min Time Correct Answer: Possible Choice: (Min 4, Max 5)

o Easy o Moderate o Hard

Author: Website: Book:

o o o

Details:

Student Guide

13

The Sample Template for Content Collection

After collecting the paper based question it is carefully entered into the central repository by skilled Data Entry Operators. Each Question passes through different stages of Quality Contro l and undergoes the detailed review to ensure the Quality and finally it becomes the part of NTS Question Bank.

2.1.2 Online Content Collection


As it is the age if IT and internet so NTS has also the online mechanism for building question bank we have the user friendly web based front end to collect the questions in different disciplines. The requirement is almost the same i.e. to ensure the quality of questions.

The Snapshot of the Front End for Online Content Collection is Give Below

Student Guide

14

2.2 Review Process 2.2.1 Review by Reviewer


As soon as question is added by the data entry operator, it becomes available to the reviewer for review. Reviewer takes care of the typing mistakes, spelling, and formatting (font face, font size) of the question. Questions that are marked as reviewed by reviewer become noneditable for this operator.

2.2.2 Technical Review

Student Guide

15

After review from reviewer, it is then viewed technically by a person/committee nominated by the peer committee relevant to that subject for the verification of the educational level, difficulty level, quality, correct answer, time, etc., regarding each question

2.2.3 Grammatical Review


The grammatical reviewer takes care of all the questions for grammatical and language point of view. Note: in both cases either paper based or online content collection, all the question passes through the same review process.

2.3

Paper Generation Process

NTS information system is capable of generating fully-formatted paper by selecting questions randomly and/or according to the template requirement. The Following can be specified as an input while generating the Paper Education level of the paper Subjects/Topics/Subtopics to be included in the paper. No. of easy, moderate, difficult questions to be included from each subject Selection of the questions can be controlled by including or excluding topics/subtopics. Selection of a question can also be limited on the basis of No. of times question used

Note: By default, the system automatically selects 30% easy, 40% moderate and 30% difficult questions to generate a paper. However this can be changed. After the template is defined, as many sample papers can be generated as required having the following in it; Each sample paper can be a distinct one. Each sample paper can be re -randomized to change the sequence of the question.

If the No. of questions selected from a topic or subtopic is not according to requirement, then same template can be redefined on topic or subtopic level by limiting the No. of questions from each topic or subtopic. The discussed process of paper generation is for both Paper based and online test. For online test the questions are picked randomly from the template constructed from the question bank and the test engine manipulate s the sequence of questions for every candidate

Test Engine
For online test NTS has developed a very robust, user friendly, stable and secured Test Engine for online test. It has very distinguished features like on spot registration, shuffling the questions randomly for each candidate, result display immediately after the completion of test, tracking of user session, flexible enough to cope with the requirement of a particular exam, continuous time monitoring, pass box for placing questions temporarily, pictures handling in the question statement and question choice, can accommodate multiple true answers, manage scenario based questions, and toggle freely between Question Box and Pass Box.

Student Guide

16

Glimpse of the web based online test application is given below User Registration Process

Instructions for the Test

Student Guide

17

Question Box

Pass Box

Student Guide

18

Paper Checking and Result Preparation


NTS has very reliable, error free, efficient and transparent system for paper checking and result preparation. It is fast enough and flexible to cope with the requirements of any client. As a reference we are quoting the Bolan Medical College, Quetta Entry Test we meet the requirement of delivering the result after few hours on the same day the test was conducted. The result was handed over along with merit lists in their required format and the notable feature was display of answer key on the spot form where any candidate could calculate his/her result because the carbon copy of answer sheet was given to the candidates after they finished their test. Despite all these facts there was not a single error in the result which shows the competency, reliability, transparency of NTS Paper Checking and Result Preparation System. We are continuously improving our System day by day by introducing new features and customization. A team of skilled professionals are dedicatedly working on the task.

Student Guide

19

Sample of an Answer Sheet is shown.

Student Guide

20

PATTERNS AND PRACTICE

Student Guide

21

Quantitative Ability

The Quantitative section measures your basic mathematical skills, understanding of elementary mathematical concepts, and the ability to reason q u a n t i t a t i v e l y a n d s o lv e p r o b l e m s i n a q u a n t i t a t i v e s e t t i n g . T h e r e i s a balance of questions requiring basic knowledge of arithmetic, algebra, geometry, and data analysis. These are essential content areas usually studied at the high school level. The questions in the quantita tive section can also b e from Discrete Quantitative Question Quantitative Comparison Question D a t a I n t e r p r e t a t i o n Q u e s t i o n etc.

The distribution in this guide is only to facilitate the candidates. This d i s t r i b u t i o n i s n o t a p a r t o f t e s t t e m p l a t e , s o , a test may contain all the q u e s t io n s o f o n e f o r m a t o r m a y h a v e a r a n d o m n u m b e r o f q u e s t i o n s o f different formats. T h i s c h a p t e r i s d i v i d e d i n t o 4 m a j o r s e c t i o n s . T h e fi r s t d i s c u s s e s t h e s y l l a b u s /c o n t e n t s i n e a c h s e c t i o n o f t h e t e s t r e s p e c t i v e l y a n d t h e r e maining three sections address the question format, guide lines to attempt the questions in each format and some example questions.

2.1
2.1.1

General Mathematics Review


Arithmetic

T h e fo l l o w i n g a r e s o m e k e y p o i n t s , w h i c h a r e p h r a s e d h e r e t o r e f r e s h y o u r knowledge of basic arithmetic principles.

Basic arithmetic
F o r a n y n u m b e r a, e x a c t l y o n e o f t h e f o l l o w i n g i s t r u e : o a is negative o a is zero o a is positive The only number that is equal to its opposite is 0 (e.g.

a = aonlyif a = 0 )

If 0 is multiplied t o a n y o t h e r n u m b e r , i t w i l l m a k e i t z e r o (

a 0 = 0 ).

P r o d u c t o r q u o t i e n t o f t w o n u m b e r s o f the same sign are always positive a n d o f a different sign are always negative. E.g. if a positive number is multiplied t o a n e g a t i v e n u m b e r t he r e s u l t w i l l b e n e g a t i v e a n d i f a negative number is divided by another negative number the result will be positive.

See the following tables for all combinations.

Student Guide

22

+ +

or or or or

+ +

= = = =

+ +

The sum of two positive numbers is always positive. The sum of two negative numbers is always negative. Subtracting a number from a n o t h e r is the same as adding its opposite

a b = a + ( b )

The reciprocal of a number a is

1 a

The product of a number and its reciprocal is always one

1 =1 a 1 b

Dividing by a number is the same as multiplying by its reciprocal

a b = a

Every integer has a finite set of factors (divisors) and an infinite set of m u l t i p l i e r s. If a a n d o o o o b a r e t w o i n t e g e r s ,t h e f o l l o w i n g f o u r t e r m s a r e s y n o n y m s a i s a divisor o f b a i s a factor o f b b i s a divisible b y a b i s a multiple o f a

T h e y a l l m e a n t h a t w h e n a i s d i v i d e d b y b t h e r e i s n o r e m a i n d e r. Positive integers, other than 1, have at le ast two positive factors. Positive integers, other than 1, which have exactly two factors, are known a s prime n u m b e r s. Every integer greater than 1 that is not a prime can be written as a product of primes. To find the prime factorization of an integer, find any two factors of that number, if both are primes, you are done; if not, continue factorization until each factor is a prime. E.g. to find the prime factorization of 48, two factors are 8 and 6. Both of t h e m a r e n o t p r i m e n u m b e r s , s o c o n t i n u e t o f a ctor them. Factors of 8 are 4 and 2, and of 4 are 2 and 2 (2 2 2). Factors of 6 are 3 and 2 (3 2). So the number 48 can be written as 2 2 2 2 3 . The L e a s t C o m m o n M u l t i p l e ( L C M ) o f t w o i n t e g e r s a a n d b i s t h e s m a l l e s t integer which is divisib le by both a and b, e.g. the LCM of 6 and 9 is 18.

Student Guide

23

The Greatest Common Divisor (GCD) of two integers a and b is the largest integer which divides both a and b, e.g. the GCD of 6 and 9 is 3. T h e p r o d u c t o f G C D a n d L C M o f t w o i n t e g e r s i s e q u a l t o t h e p r o d u cts of numbers itself. E.g. 6 9 = 54 3 18 = 54 ( w h e r e 3 i s G C D a n d 1 8 i s L C M o f 6 a n d 9 ). Even numbers are all the multiples of 2 e.g. ( , 4, 2, 0, 2, 4, ) O d d n u m b e r s a r e a l l i n t e g e r s n o t d i v i s i b l e b y 2 ( , 5 , 3, 1 , 1 , 3 , 5 , ) If tw o i n t e g e r s a r e b o t h e v e n o r b o t h o d d , t h e i r s u m a n d d i f f e r e n c e a r e even. If one integer is even and the other is odd, their sum and difference are odd. The product of two integers is even unless both of them are odd. When an equation involves more than one operation, it is important to c a r r y t h e m o u t i n t h e correct order. T h e c o r r e c t o r d e r i s P a r e n t h e s e s , E x p o n e n t s , M u l t i p l i c a t i o n a n d D ivision, A d d i t i o n a n d S u b t r a c t i o n , o r j u s t t h e f i r s t l e t t e r s PEMDAS t o r e m e m b e r t h e p r o p e r o r d e r .

Exponents and Roots


Repeated addition of the same number is indicated by multiplication: 17 + 17 + 17 + 17 + 17 = 5 17 Repeated multiplication of the same number is indicated by an exponent: 5 17 17 17 17 17 = 17

5 I n t h e e x p r e s s i o n 1 7 , 17 is called b a s e a n d 5 i s t h e exponent . For any number b: b as factor. 1 = b and b n = b b b , w h e r e b i s u s e d n times

For any numbers b and c and positive integers m and n: o o o o

b m b n = b m+ n bm = b m n bn (b m ) n = b mn

b m c m = (bc ) m
n is positive if n is even, and negative if n is odd.
2

If a i s n e g a t i v e , a

T h e r e a r e t w o n u m b e r s t h a t s a t i s f y t h e e q u a t i o n x = 9 : x = 3 a n d x = 3 . The positive one, 3, is called the (principal) square root of 9 and is denoted by symbol

9.

Clearly, each perfect square has a square root:

0 = 0,

9 = 3,

36 = 6 ,

169 = 13 ,

225 = 25

etc.

Student Guide

24

F o r a n y p o s i t i v e n u mb e r a t h e r e i s a p o s i t i v e n u m b e r b t h a t s a t i s f i e s t h e equation

a = b.
( a )2 = a a = a .

For any positive integer,

For any positive numbers a and b:

ab = a b
o

and

a = b

a b

a +b a + b
o

as

5 = 25 = 9 + 16 9 + 16 = 3 + 4 = 7
( a )2 = a ,

Al t h o u g h i t i s a l w a y s t r u e t h a t
2

a 2 = a is

true only if a is

positive as

( 5) = 25 = 5 5

For any number a ,

a =a
n

n 2

F o r a n y n u m b e r a , b a n d c:

a (b + c) = ab + ac
and if

a (b c ) = ab ac

a0
(b c) b c = a a a

(b + c ) b c = + a a a

Inequalities
For any number a and b, exactly one of the following is true:

a>b

or

a=b

or

a < b.

For a n y n u m b e r a a n d b , For any number a and b, For any number a and b, The symbol

a > b means a < b means a = b means

that that that

a b is a b is a b is

positive. negative. zero.

means greater than or equal to and the symbol

than or equal to. E.g. the statement number greater than 5. T h e s t a te m e n t

x 5 means

means less

that x can be 5 or any

2 < x < 5 is

an abbreviation of

2 < x and x < 5.

Adding or subtracting a number to an inequality preserves it. If

a <b,

then

a + c < b + c and a c < b c .

Student Guide

25

e .g.

5 < 6 5 + 10 < 6 + 10

and

5 10 < 6 10

Adding inequalities in same direction preserves it: If

a < b and

c<d,

then

a+c <b+d . ac < bc ac > bc


a b < c c.

Multiplying or dividing an inequality by a positive number preserves it. If

a < b and c is

a positive number, then

and

M u l t i p l y i n g o r d i v i d i n g a n i n e q u a l i t y b y a n e g a t i v e n u m b e r r e v e r s e s it. If

a < b and c is

a negative number, then

and

a b > c c.

If sides of an inequality are both positive and both negative, taking the reciprocal reverses the inequality. If If

0 < x < 1a n d a i s

positive, then

xa < a . m >n,
then

0 < x < 1 a n d m and n a r e 0 < x < 1,


x > x.

integers with

xm < x n < x .

If

then

If

0 < x < 1 , then

1 1 >x >1 x and x

Properties of Zero
0 is the only number that is neither negative nor positive. 0 is smaller than every positive number and greater than every negative number. 0 is an even integer. 0 is a m u l t i p l e o f e v e r y i n t e g e r . For every number For every number

a a

a + 0 = aanda 0 = a .

: a0

= 0.

For every positive integer

n : 0n = 0 .
a 0 and 0 a = a 0
are undefined symbols.

For every number

(including 0):

For every number

(other than 0):

0 =0. a

Student Guide

26

0 is the only number that is equal to its o pposite:

0 = 0 .

If t h e p r o d u c t o f t w o o r m o r e n u m b e r s i s 0 , a t l e a s t o n e o f t h e m i s 0 .

Properties of One For any number For any

a a : a 1 = a and = a . 1 n n 1 = 1 number : .

1 is the divisor of every integer. 1 is the smallest positive integer. 1 is an odd integer. 1 is not a prime.

Fractions and Decimals


When a whole is divided into n equal parts, each part is called one nth of the whole, written

1 . n

For example, if a pizza is cut (divided) into 8 equal

slices, each slice is one eighth (

1 ) 8

of the pizza; a day is divided into 24

e q u a l h o u r s , s o a n h o u r i s o n e t w e n t y -f o u r t h one twelfth (

1 ) 24

of a day and an inch is

1 ) of a foot. If one works for 8 hours a day, he works eight 12 8 t w e n t y -fourth ( ) of a day. If a hockey stick is 40 inches long, it 24 40 ) of a foot. measures forty twelfths ( 12 1 1 8 40 The numbers such as , , and , in which one integer is written 8 24 24 12

o v e r t h e s e c o n d i n t e g e r , a r e c a ll e d fractions . T h e c e n t e r l i n e i s c a l l e d t h e fraction bar. T h e n u m b e r a b o v e t h e b a r i s c a l l e d t h e numerator , a n d t h e n u m b e r b e l o w t h e b a r i s c a l l e d denominator . The denominator of a fraction can never be 0. A fraction, such as

1 , 24 40 , 12

in which the denominator is greater than

n u m e r a t o r , i s k n o w n a s a proper fraction . I t s v a l u e i s l e s s t h a n o n e . A fraction, such as in which the denominator is less than numerator, is

k n o w n a s an improper fraction. I t s v a l u e i s g r e a t e r t h a n o n e .

Student Guide

27

A fraction, such as,

i s a l s o k n o w n a s a n improper fraction . B u t , I t s v a l u e i s o n e . Every fraction can be expressed in decimal form (or as a whole number) by dividing the number by the denominator.

12 12

in which the denominator is equal to the numerator,

3 = 0.3, 10

3 = 0.75, 4

8 48 100 = 1, = 3, = 12.5 8 16 8

Unlike the examples above, when most fractions are converted to decimals, the division does not terminate, after 2 or 3 or 4 decimal places; rather it g o e s o n f o r e v e r w i t h s o m e s e t o f d i g i t s r e p e a t i n g it .

2 3 5 17 = 0.66666..., = 0.272727..., = 0.416666..., = 1.133333... 3 11 12 15

Student Guide

28

To compare two decimals, follow these rules: o Whichever number has the greater number to the left of the decimal point is greater: since 11 > 9, 11.0001 > 9.8965 and since 1 > 0, 1.234 > .8. (Recall that i f a d e c i m a l i s w r i t t e n w i t h o u t a n u m b e r o n left of decimal point, you may assume that a 0 is there, so, .8 = 0.8). If the numbers to the left of the decimal point are equal, proceed as follows:

If the numbers do not have the same number of digits to the right of the decimal point, add zeroes to the end of the shorter one to make them equal in length. Now compare the numbers ignoring the decimal point. For example, to compare 1.83 and 1.823, add a 0 to the end of 1.83 forming 1.830. Now compare them, thinking of them as whole numbers without decimal point: since 1830 > 1823, then 1.830 >1.823.

There are two ways to compare fractions: o Convert them to decimals by dividing, and use the method already described to compare these decimals. For example to compare and

2 5

1 , 4

convert them to decimals.

2 1 = 0.4 a n d = 0.25 . 5 4

Now, as 0.4

> 0.25,

2 5

>

1 . 4 2 5
and

Cross multiply the fractions . F o r e x a m p l e t o c o m p a r e cross

multiply:

1 , 4

2 5

1 4

Since

2 4 > 1 5 ,

then

2 1 > 5 4.
the denominators, the

While comparing the fractions, if they have same

fraction with the larger numerator is greater. For example

3 2 > . 5 5
the

If the fractions have the same numerator, the fraction with smaller denominator is greater. For example

3 3 > . 5 10

T w o f r a c t i o n s a r e c a l l e d equivalent fractions i f b o t h o f t h e m h a v e s a m e decimal value. For example,

A n o t h e r w a y t o c h e c k t h e e q u i v a l e n c e o f t w o f r a c t i o n s i s t o c r o s s -multiply. If both of the products are same, the fractions are equivalent. For Example, to compare

1 5 = 2 10

as both of these are equal to 0.5.

2 5

with

6 , 15

c r o s s -multiply. Since

2 15 =6 5 ,

both of the

fractio n s a r e e q u i v a l e n t . Every fraction can be reduced to lowest terms by dividing the numerator and denominator by their greatest common divisor (GCD). If the GCD is 1,

Student Guide

29

the fraction is already in lowest terms. For example to reduce

both numerator and denominator by 5 (which is GCD of 10 and 15). This will reduce the fraction to

10 , 15

divide

To multiply two fractions, multiply their numerators and multiply their denominators. For example

2 . 3

To multiply a number to a fraction, write that number as a fraction whose denominator is 1. For example

3 4 3 4 12 = = . 5 7 5 7 35

When a problem requires you to find the fraction of a number, multiply that fraction with the number. For example, to
80

3 3 7 3 7 21 7 = = = . 5 5 1 51 5

find two fifth (

2 ) 5

of 200,

2 2 200 400 200 = = = 80 5 1 5 / multiply: 5 . a The reciprocal of a fraction is another b

fraction

T o d i v i d e o n e f r a c t i o n b y the other fraction, multiply the reciprocal of divisor with the dividend. For example,

b a

since
2

a b =1 b a

22 11 22 7 2 = = = 2. 7 7 7 11 1

To add or subtract the fractions with same denominator, add or subtract numerators and keep the denominator. For example

4 1 5 + = 9 9 9

and

4 1 3 = . 9 9 9

Percents
T h e w o r d percent m e a n s h u n d r e d t h . W e u s e t h e s y m b o l % t o e x p r e s s t h e w o r d percent. F o r e x a m p l e 1 5 p e r c e n t m e a n s 1 5 h u n d r e d t h s a n d c a n b e written with a % symbol, as a fraction, or as a decimal.

20% =

To conve r t a d e c i m a l t o a p e r c e n t , m o v e t h e d e c i m a l p o i n t t w o p l a c e s t o t h e right, adding 0s is necessary, and add the percent symbol (%). For example, 0.375 = 37.5% 0.3 = 30% 1.25 = 125% 10=1000% To convert a fraction to a percent, first convert that fraction to decimal, than use the method stated above to convert it to a percent. To convert a percent to a decimal, move the decimal point two places to the left and remove the % symbol. Add 0s if necessary. For example, 25% = 0.25 1% =0.01 100% = 1 You should be familiar with the following basic conversions:

20 = 0.20 . 100

1 5 = = 0.50 = 50% 2 10

1 2 = = 0.20 = 20% 5 10

Student Guide

30

1 = 0.25 = 25% 4
For any positive integer

3 = 0.75 = 75% 4

a : a% o f

100 is

a.

For any positive numbers

a a n d b : a % of b = b% of a
actual change 100% . original amount
For example:

The percent change in the quantity is

If the price of a lamp goes from Rs .80 to Rs.100, the actual increase is

20 1 100% = 100% = 25% . 80 4 a < b a If , the percent increase in going from to b is always greater perce n t d e c r e a s e i n g o i n g f r o m b t o a .
Rs.20, and the percent increase is To increase a number by n u m b e r b y k % , multiply it by 1 k % . F o r e x a m p l e , t h e v a l u e o f a n investment of Rs. 20,000 after 25% increase is 20,000 (1 + 25%) = 20,000 (1.25) = 25,000 .

than

k% ,

multiply it by

1 + k% ,

and to decrease a

If a number is the result of increasing another number by

o r i g i n a l n u m b e r d i v i d e i t b y 1 + k% , a n d i f a n u m b e r i s t h e r e s u l t o f decreasing another number by k % , to find the original number, divide it by 1 k % . F o r e x a m p l e , T h e g o v e r n m e n t a n n o u n c e d a n 20% i n c r e a s e i n s a l a r i e s . I f a f t e r t h e i n c r e m e n t , T h e s a l a r y o f a p a r t i c u l a r e m p l o y e e i s R s . 1 8 , 000, w h a t was the original salary? Original salary (in Rs.) =

k% ,

to find the

current salary 18,000 18,000 = = = 15,000 1 + percent increase 1 + 20% 1.20

Ratios and Proportions


A r a t i o i s a f r a c t i o n t h a t c o m p a r e s t w o q u a n t i t i e s t h a t a r e measured in t h e same units. The first quantity is the numerator and the second quantity is denominator. For example, if there are 16 boys and 4 girls, we say that the ratio of the number of boys to the number of girls on the team is 16 to 4, or

16 . 4

This is often written as 16:4. Since a ratio is just a fraction, it can

b e r e d u c e d o r c o n v e r t e d t o a decimal or a percent. The Following are different ways to express the same ratio:

16 to 4 ,

1 6 : 4 , 16 , 4 , 4 1

0.25 ,

25% a:b,
then the

If a set of objects is divided into two groups in the ration fi r s t g r o u p c o n t a i n s

a a +b

of the total objects and similarly the second group

Student Guide

31

contains

b a +b

of the total number of objects. This rule applies to extended

ratios, as well. If a set is divided into three groups in the ratio the first group contains

a :b :c ,

then

a a +b +c

of the total objects, and so on.

A proportion is an equation that states that two ratios are equivalent. Since ratios are just fractions, any equation such as

a single fraction is proportion. This proportion states that 4 relates to 6 in same ratio as 10 relates to 15. F o r e a c h p r o p o r t i o n o f t h e form

4 10 = 6 15

in which each side is

a c = b d

ad = bc .

This p r o p e r t y c a n b e u s e d

to solve proportions for unknowns (variables). For example: If 3 oranges cost Rs.5, how many oranges can you buy for Rs.100. To solve this problem we have to set up a proportion. If the number of oranges for Rs.100 is x , then:

3 x 3 100 = 3 100 = x 5 x = x = 60 5 100 5


Averages
T h e average o f a s e t o f n n u m b e r s i s t h e s u m o f t h o s e n u m b e r s d i v i d e d b y n.

average =

sumof nnumbers n

or simply

A=

Sum n

t h e t e c h n i c a l n a m e f o r t h e s e kind of averag e s i s A r i t h m e t i c M e a n . If you know the average of n numbers, multiply that average with n to get the sum of numbers. I f a l l t h e n u m b e r s i n a s e t a r e the s a m e , t h e n t h a t n u m b e r i s t h e a v e r a g e . A s s u m e t h a t t h e a v e r a g e o f a s e t o f n u m b e r s i s A . I f a n e w number x is added to that set, the new average will be; o o o Greater if x is greater than the existing average Smaller if x is smaller than the existing average Unchanged if x is equal to the existing average

A r i t h m e t i c s e q u e n c e i s a n o r d e r e d s e t o f n u m b e r s , such that, the difference between two consecutive numbers is the same. I f t h e r e i s a n a r i t h m e t i c s e q u e n c e o f n t e r m s , t h e n t h e average calculation can be made simple b y using these rules. o The average of the terms in that sequence will be the middle term, if n is odd. o If n is even, the average will be the average of two middle terms.

2.1.2

Algebra

Student Guide

32

Polynomials

A monomial is any number or variable or product of numbers and variables. For example

3, 4, x , y , 3x , 2 xyz, 5 x 3,1.5 xy 2 , a3b 2 a r e


The coefficient of

all monomials.

The number that appears in front of a variable in a monomial is called the coefficient.

5 x3

is

5.

If

there

coefficient is either 1 or 1, because

means

1x

and

x m e a n s 1x .

is

no

number,

the

A polynomial is a monomial or the sum of two or more monomials. Each monomial that makes up the polynomial is called a term of that polynomial. I f a p o l y n o m i a l h a s o n l y o n e t e r m i t i s a s i m p l e monomial, if it has two terms, it is known as binomial and if it has three terms, it is called trinomial. Two terms are called like terms if they differ only in their coefficients. and

5 x3

2 x 3

a r e l i k e t e r ms , w h e r e a s ,

5 x3

and

5x2 a r e

not.

If like terms are involved in a polynomial, they can be combined, by adding their coefficients, to make that polynomial simpler. The polynomial is e q u i v a l e n t t o t h e p o l y n o m i a l x + 9 x 7 . All laws of arithmetic are also applicable to polynomials. Most important of them is PEMDAS.
2

3x 2 + 4 x + 5 x 2 x2 7

Polynomials can be added, subtracted, multiplied or divided. To add two polynomials, put a plus sign between them, erase the parentheses, and combine like terms. Example: What is the sum of Solution:

5 x 2 + 10 x 7

and

3x 2 4 x + 2 ?

(5 x 2 + 10 x 7) + (3 x2 4 x + 2) = 5 x 2 + 10 x 7 + 3 x2 4 x + 2 = 8x2 + 6x 5
To subtract two polynomials, reverse the signs of subtrahend, and add two polynomials as done before. Example: Subtract

3x 2 4 x + 2

from

5 x 2 + 10 x 7

Student Guide

33

Solution:

(5 x 2 + 10 x 7) (3 x2 4 x + 2) = (5 x 2 + 10 x 7) + (3 x2 + 4 x 2) = 5 x 2 + 10 x 7 3 x2 + 4 x 2 = 2 x 2 + 14 x 9
To multiply monomials, first multiply their coefficients, and then multiply their variables by adding the exponents. Example: What is the product of Solution:

3x 2 yz

from

2 x 2 y 2 ?

(3 x 2 yz )( 2 x 2 y 2 ) = (3 2)( x 2 x 2 )( y y 2 )( z ) = 6 x 4 y 3 z
To multiply a monomial by a polynomial, just multiply each term of the polynomial by the monomial.

Example: What is the product of Solution:

3x

from

3x 2 6 xy 2 + 2 ?

(3 x)(3x 2 6 xy 2 + 2) = (3x 3x 2 ) (3 x 6 xy 2 ) + (3x 2) = 9 x 3 18 x 2 y 2 + 6 x


To multiply two binomials, multiply each term of first binomial by each term of second binomial, then add the results. Example: What is the product of Solution:

3x + y

from

3x 2 6 xy 2 ?

(3 x + y)(3 x 2 6 xy 2 ) = (3x 3x 2 ) + (3x ( 6 xy 2 )) + ( y 3x 2 ) + ( y ( 6 xy 2 )) = (9 x 2 ) + ( 18 x 2 y 2 ) + (3 x 2 y) + ( 6 xy 3 ) = 9 x 2 18 x 2 y 2 + 3x 2 y 6 xy 3


T h e t h r e e m o s t i m p o rt a n t b i n o m i a l p r o d u c t s a r e :

Student Guide

34

o o o

( x y )( x + y) = x2 + xy xy + y 2 = x 2 + y 2 ( x + y )( x + y) = x2 + xy + xy + y 2 = x 2 + 2 xy + y 2 ( x y )( x y ) = x 2 xy xy + y 2 = x 2 2 xy + y 2
Memorizing these can save a lot of calculation time during the test.

To divide a polynomial by a monomial, divide each term of the polynomial by the monomial. Example: What is the quotient if Solution:

32 x 2 y + 12 xy 3z

is divided by

8xy ?

32 x 2 y + 12 xy3 z 32 x 2 y 12 xy 3 z 3 = + = 4 x + y 2 z (by reducing theterms) 8 xy 8 xy 8 xy 2

Student Guide

35

Solving Equations and Inequalities The basic principle in solving equations and inequalities is that you can manipulate them in any way as long as you d o t h e s a m e t h i n g t o b o t h s i d e s. F o r e x a m p le you may add a number to both sides, or you may divide or multiply both sides with same number etc. B y u s i n g t h e f o l l o w i n g s i x -s t e p m e t h o d , y o u c a n s o l v e m o s t o f t h e equations and inequalities. The method is explained with the help of an example. E x a m p le: if

1 x + 3( x 2) = 2( x + 1) + 1 , 2
What to do

what

is

the

value

of

x?

Solution: Step 1 Example

2 3 4

5 6

Get rid of fractions and Multiply each side by 2 to get: decimals by multiplying x + 6( x 2) = 4( x + 1) + 2 both sides by the LCD. Get rid of all parentheses x + 6 x 12 = 4 x + 4 + 2 by solving them. Combine like terms on 7 x 12 = 4 x + 6 each side. By adding and subtracting 4 x fro m e a c h s i d e t o g e t : Subtract get all the variables on 3x 12 = 6 one side (mostly left). By adding or subtracting Add 12 to each side to get: get all plain numbers on 3 x = 18 the other side. Divide both sides by the D i v i d e both sides by 3 to get: coefficient of the variable. x=6 (If you are dealing with an inequality and you divide with a negative number, remember to reverse the inequality.) When you have to solve one variable and the equation/inequality involve m o r e t h a n o n e v a r i a b l e , t r e a t al l o t h e r v a r i a b l e s a s p l a i n n u m b e r s a n d a p p l y t h e s i x -s t e p m e t h o d . Example:

a = 3b c , if Solution:
Step 1 2 3 4

what

is

the

value

of

in

terms

of

and

c?

What to do There are no fractions and decimals. There are no parentheses. There are no like terms. By adding and subtracting get all the variables on one side. By adding or subtracting get all plain numbers on the other side.

Example

Remember there is only one variable b , which is on one s ide only. Remember we are a c considering and as plain number. A d d c to each side to get:

a + c = 3b

Student Guide

36

Divide both sides by the coefficient of the variable.

Divide get:

both

sides

by

to

a +c =b 3

It is not necessary to follow these steps in the order specified. Some times it makes the problem much easier, if you change the order of these steps. Example: If

x 4 = 11 ,

what is the value of

x -8 ?
x = 15 .
Now

Solution: Going immediately to step 5, add 4 to each side to get: subtract 8 from both sides to get:

x 8 = 7 .

Doing the same thing on each side of an equation does not mean doing the s a m e t h i n g t o e a c h t e r m o f t h e e q u a t i o n . T h i s i s v e r y i m p o r t a n t if y o u a r e doing divisions, or dealing with exponents and roots. Example: If

a>0

and

a 2 + b2 = c 2 ,

what is the value of

in terms of

and

c.

Solution:

a 2 + b2 = c 2
to get
2 2

a =c b .
2

a 2 = c 2 b2 .

Now you cant take a square root of each term

You must take the square root of each side:

a = c b

a = c 2 b2

Another type of equation is that in which the variable appears in exponent. These equations are basically solved by inception. Example:

= 32 , w h a t i s t h e v a l u e o f If 2 Solution:

x +3

3x + 2 ?

2 x + 3 = 32 2 x+ 3 = 25
Now as

x +3 = 5 x = 2. x=2 x + 2 = 4 3x + 2 = 34 = 81

x =2,

you can get

A system of equations is a set of two or more equations having two or more variables. To solve such equations, you must find the value of each variable that will make each equation true. To solve a system of equations, add or subtract them to get a third e q u a t i o n . I f t h e r e a r e m o r e t h a n t w o e q u a t i o n s y o u c a n j u s t a d d them. Example: If

x + y = 10

and

x y = 10

what is the value of

y?

Solution:

Student Guide

37

x + y = 10
Add two equations: Now replacing

x y = 2 2x = 12 x = 6
6 + y = 10 y= 4

with 6 in t h e f i r s t e q u a t io n :

If you know the value of one variable in a system of two equations, you can u s e t h i s v a l u e t o g e t t h e v a l u e o f the o t h e r v a r i a b l e . A s i t i s d o n e i n the previous question.

Word problems T o s o l v e w o r d p r o b l e m s , f i r s t t r a n s la t e t h e p r o b l e m f r o m E n g l i s h t o Algebra. While translating, use variables to represent unknowns. Once translated, it is easy to solve them using the techniques you have learned in previous sections. F o l l o w i n g E n g l i s h t o A l g e b r a d i c t i o n a r y w i l l b e h e l p f u l in t r a n s l a t i n g w o r d problems to algebraic expressions. English words Is, was, will be, had, has, will have, is equal to, is the same as Plus, more than, sum, increased by, added to, exceeds, received, got, older than, farther than, greater than Minus, fewer, less than, difference, decreased by, subtracted from, younger than, gave, lost Times, of, product, multiplied b y Divided by, quotient, per, for More than, greater than At least Fewer than, less than At most What, how many, etc. Mathematical meaning Equals Addition Symbol

Subtraction

Multiplication Division Inequality Inequality I n e q uality Inequality Unknown quantity

or
> < x

a b

(Some variable) Examples: o The sum of 5 and some number is 13.

o
o o

Javed was two years younger than Saleem. Bilal has at most Rs.10,000. The product of 2 and a number exceeds that number by 5 (is 5 more than that number).

5 + x = 13 J = S 2 B 10000 2N = N + 5

I n w o r d p r o b l e m s , y o u m u s t b e s u r e a b o u t w h a t y o u a r e answering. Do not answer the wrong question. In problems involving ages, remember that years ago means you need to subtract, and years from now means you need to add.

Student Guide

38

Distance problems all depend on three variations of the same formula:

o
o

distance = speed time distance speed = time distance time = speed

Example: How much longer, in seconds, is required to drive 1 mile at 40 miles per hour than at 60 miles per hour? Solution: The time to drive at 40 miles p er hour can be calculated as

time1

1 hours 40

1 60 40 2

minutes

30 3 60 s e c o n d s 2

90 s e c o n d s
The time to drive at 60 miles per hour can be calculated as

time 2

1 hours 60

1 60 60

minutes

1 60 s e c o n d s

60 s e c o n d s

difference = time1 time2 = 90 60 = 30


2.1.3

seconds.

Geometry

L i n e s a n d A n g les
An a n g l e i s f o r m e d a t t h e i n t e r s e c t i o n o f t w o l i n e s e g m e n t s , r a y s o r l i n e s . T h e p o i n t o f i n t e r s e c t i o n i s c a l l e d t h e vertex . A n g l e s a r e m e a s u r e d i n degrees. Angles are classified according to their degree measures. An acute a n g l e m e a s u r e s l e s s t h a n

90

A right a n g l e m e a s u r e s

90 90
but less than

An o b t u s e a n g l e m e a s u r e s m o r e t h a n A straight angle measures

180

180

I f t w o o r mo r e a n g l e s c o m b i n e t o g e t h e r t o f o r m a s t r a i g h t a n g l e , t h e s u m of their measures is

180 .

a + b+ c + d = 180

Student Guide

39

The sum of all the measures of all the angles around a point is

360

a + b+ c + d + e = 360
When two lines intersect, four angles are formed, two angles in each pair of o p p o s i t e a n g l e s a r e c a l l e d vertical angles . V e r t i c a l a n g l e s , f o r m e d b y t h e intersection of two lines, have equal measures.

a=c

and

b=d

If one of the angles formed by the intersection of two lines is a right angle, t h e n a l l f o u r a n g l e s a r e r i g h t a n g l e s . S u c h l i n e s a r e c a l l e d perpendicular lines

a = b = c = 90
In the figure below a line l divides the angle in two equal parts. This line is s a i d t o bisect t h e a n g l e . T h e o t h e r l i n e k b i s e c t s a n o t h e r l i n e i n t o t w o equal parts. This line is said to bisect a line.

T w o l i n e s a r e s a i d t o b e parallel, i f t h e y n e v e r i n t e r s e c t e a c h o t h e r . However, if a third line, c a l l e d a transversal , i n t e r s e c t s a p a i r o f p a r a l l e l lines, eight angles are formed. And the relationship among theses angles is shown in the following diagram. - All four acute angles are equal

a = c = e= g
All

Triangles

b =d = f =h
sum

four

obtuse of any

angles pair of

a re

equal and e.g. etc.

- The

180 , obtuse angle is a + d = 180, d + e = 180, b + g = 180

acute

Student Guide

40

In any triangle, the sum of the measures of the three angles is

180

x + y + z = 180
In any triangle: o The longest side of triangle is opposite the largest angle. o The shortest side is opposite the smallest angle. o Sides with the same length are opposite the angles with the same measure. T r i a n g l e s a r e c l a s s if i e d i n t o t h r e e d i f f e r e n t k i n d s w i t h r e s p e c t t o t h e lengths of sides. o Scalene : i n w h i c h a l l t h r e e s i d e s a r e o f d i f f e r e n t l e n g t h s . o Isosceles : i n w h i c h t w o o f t h e s i d e s o f t r i a n g l e a r e e q u a l i n length, the third is different. o Equilateral : i n w h i c h a l l t h r e e s i d e s a r e e q u a l i n l e n g t h . T r i a n g l e s a r e a l s o c l a s s if i e d w i t h r e s p e c t t o t h e a n g l e s . o Acute triangle : i n w h i c h a l l t h r e e a n g l e s a r e a c u t e . o Obtuse triangle : i n w h i c h o n e a n g l e i s o b t u s e a n d t w o a r e a c u t e . o Right triangle : T h i s h a s o n e r i g h t a n d t w o a c u t e a n g l e s . I n a r i g h t t r i a n g l e , t h e o p p o s i t e t o t h e r i g h t a n g l e i s k n o w n a s hypotenuse a n d i s t h e l o n g e s t s i d e . T h e o t h e r t w o s i d e s a r e c a l l e d l e g s. I n a n y r i g h t t r i a n g l e , t h e s u m o f t h e m e a s u r e s o f the t w o a c u t e a n g l e s i s

90 .

B y P yt h a g o r e a n T h e o r e m, t h e s u m o f s q u a r e s o f t h e l e n g t h s o f l e g s o f a right triangle is always equal to the square of length of hypotenuse.

In any triangle, the sum of any two sides is always greater than the third o n e . And the difference of any two sides is always less than the third one.

a 2 + b2 = c 2

a +b>c

and

a b < c

T h e perimeter o f a t r i a n g l e i s c a l c u l a t e d b y a d d i n g t h e l e n g t h s o f a l l t h e sides of that triangle.

Student Guide

41

perimeter = a + b + c
The area of a triangle is calculated by the formula:

area =

1 bh 2

where

is

t h e b a s e o f t h e t r i a n g l e a n d h i s t h e height o f t h e t r i a n g l e . o Any side of triangle can be taken as the base. o H e ight is the altitude (perpendicular) drawn to the base from its opposite vertex. o I n a r i g h t t r i a n g l e a n y l e g c o u l d b e t a k e n a s the b a s e , t h e o t h e r will be the altitude.

Quadrilateral and other Polygons


A polygon i s a c l o s e d g e o m e t r i c f i g u r e , m a d e u p of line se gme nt s. The l i n e s e g m e n t s a r e c a l l e d sides a n d t h e e n d p o i n t s o f l i n e s a r e c a l l e d vertices ( p l u r a l o f v e r t e x ) . L i n e s , i n s i d e t h e p o l y g o n , d r a w n f r o m o n e v e r t e x t o t h e o t h e r , a r e c a l l e d d i a g o n a l s.

The sum of the measures of the always

( n 2) 180 .

angles in a polygon with

sides is

In any quadrilateral, the sum of the measures of the four angles is

360 .

A r e g u l a r p o l y g o n i s a p o l y g o n i n w h i c h a l l o f t h e s i d e s a r e o f the same length. In any regular polygon, the measure of each interior angle is

(n 2) 180 n

and the measure of each exterior angle is

360 . n

Student Guide

42

A p a r a l l e l o g r a m i s a s p e c i a l q u a d r i l a t e ra l , i n w h i c h b o t h p a i r s o f o p p o s i t e sides are parallel. The Following are some properties of parallelogram.

o o o o o

AB = CD a n d AD = BC Measures of opposite angles are equal. a = c and b = d C o n s e c u t i v e a n g l e s a d d u p t o 180 . a + b = 180 , b + c = 180


Lengths of opposite sides are equal.

etc.

T h e t w o d i a g o n a l s b i s e c t e a c h o t h e r . AE = EC a n d BE = ED A diagonal divides the parallelogram into two triangles that are congruent.

A rectangle i s a p a r a l l e l o g r a m i n w h i c h a l l f o u r a n g l e s a r e r i g h t a n g l e s . I t has all the properties of a parallelogram. In addition it has the following properties: o o T h e m e a s u r e o f e a c h a n g l e i n a r e c t a n g l e i s 90 . The diagonals of a rectangle are equal in length.

A square i s a r e c t a n g l e t h a t h a s t h e f o l l o w i n g a d d i t i o n a l p r o p e r t i e s : o A square has all its sides equal in length. o I n a s q u a r e , d i a g o n a ls a r e p e r p e n d i c u l a r t o e a c h o t h e r .

To calculate the area, the following formulas are required: o o o For a parallelogram, height. F o r a r e c t a n gle, width. For a square,

Area = bh ,

where

is the b a s e a n d

is the i s the

Area = lw ,

where

is the l e n g t h a n d

Area = s 2 ,

where

is the side of the square.

Perimeter for any polygon is the sum of lengths, of all its sides.

Student Guide

43

Circles
A circle consists of all the points that are the same distance from one fixed point called the center. That distance is called the radius o f a circle. T h e w o r d r a d i u s i s a l s o u s e d t o r e p r e s e n t a n y o f the line segments joining the c e n t e r a n d a p o int on the circle. The p l u r a l o f r a d i u s i s radii .

CED in the figure, formed by connecting the end Any triangle, such as V p o i n t s o f t w o r a d ii, is a n i s o s c e l e s .
A l i n e s e g m e n t , s u c h a s ED i n t h e d i a g r a m a b o v e , b o t h o f w h o s e e n d p o i n t s a r e o n a c i r c l e i s c a l l e d a chord . A c h o r d t h a t p a s s e s t h r o u g h t h e c e n t e r o f t h e c i r c l e i s c a l l e d t h e diameter o f t h e circle. The lengt h o f t h e d i a m e t e r i s a l w a y s d o u b l e t h e r a d i u s o f the circle. The d i a m e t e r i s t h e l o n g e s t c o r d t h a t c a n b e d r a w n i n a c i r c l e . The total length around a circle is known as circle. the circumference o f t h e

T h e r a t i o o f the circumference to the diameter is a l w a y s the s a m e f o r a n y circle. T h i s r a t i o i s d e n o t e d b y t h e s y m b o l ( p r o n o u n c e d a s p i ) .

a n d r i s t h e r a d i u s o f t h e circle. Value of

C C = d C = 2 r d

w h e r e C i s t h e circumference, d is t h e d i a m e t e r

is approximately

3.14

A n a r c c o n s i s t s o f t w o p o i n t s in a c i r c l e a n d a l l t h e p o i n t s b e t w e e n t h e m . E.g. PQ i s a n a r c i n t h e d i a g r a m .

A n a n g l e w h o s e v e r t e x i s a t t h e c e n t e r o f t h e c i r c l e i s c a l l e d t h e central angle .

PCQ

i n t h e d i a g r a m a b o v e i s a central angle .

The degree measure of a complete circle is

360 .

The degree measure of an arc is the measure of the central angle that intercepts it. E.g. the degree measure of

PQ

is equal to the measure of

PCQ

in the diagram above.

Student Guide

44

If

i s t h e d e g r e e m e a su r e o f a n a r c , i t s l e n g t h c a n b e c a l c u l a t e d a s

where C is the circumference.

x C, 360

r2 . The area of a sector formed by the arc and two radii can be x A , where A is the area of a circle. calculated as 360
The area of a circle can be calculated as

2.2

Discrete Quantitative Questions

These are standard multiple-choice questions. Most of such questions require you to do some computations and you have to choose exactly one of the available choices based upon those computations. This section will teach you the basic tactics to attempt such questions.

2.2.1

Question format

Each question will consist of a question statement and the choices labeled from A to E. The number of choices may vary from 2 to 5, but exactly one choice will be correct for each question.

2.2.2

How to attempt ?

Following are some tactics, which will lead you to the correct answer. Whenever you know how to answer a question directly, just do it. The t a c t i c s s h o u l d b e u s e d o n l y w h e n y o u d o not know the exact solution, and you just want to eliminate the choices. R e m e m b e r t h a t n o p r o b l e m r e q u i r e s l e n g t h y o r d i f f i c u l t c o m p u t a t i o n s . If y o u f i n d y o u r s e l f d o i n g a l o t o f c o m p l e x a r i t h m e t i c , t h i n k a g a i n . You may be going in the wrong direction. W h e n e v e r t h e r e i s a q u e s t i o n w i t h some unknowns (variables), replace them with the appropriate numeric values for ease of calculation. W h e n y o u n e e d t o r e p l a c e v a r i a b l e s w i t h v a l u e s , c h o o s e e a s y-t o -use n u m b e r s , e . g . t h e n u m b e r 1 0 0 i s a p p r o p r i a t e i n m o s t p e r c e n t -related problems and the LCD (least common denominator) is best suited in questions that involve fractions. Apply back-solving whenever you know what to do to answer the question b u t y o u w a n t t o a v o i d d o i n g a l g e b r a . To understand this tactic re ad the following example: O n M o n d a y , a s t o r e o w n e r r e c e i v e d a s h i p m e n t o f b o o k s . On Tuesday, she sold half of them. On Wednesday after two more were sold, she had exactly 2/5 of the books left. How many were in the shipment? (A) 10 (B) 20 (C) 30 (D) 4 0 (E) 50

now by this tactic: Assume that (A) is the correct answer, if so; she must have 3 books on W e d n e s d a y . B u t 2 / 5 o f 1 0 are 4 , s o , ( A ) i s i n c o r r e c t ; Assume that (B) is the correct answer, if so; she must have 8 books on Wednesday. 2/5 of 20 are 8 , so, (B) is the correct choice, and as there may

Student Guide

45

be only one correct choice, there is no need to check for remaining choices. This tactic is very h e l p f u l w h e n a n o r m a l a l g e b r a i c s o l u t i o n fo r the problem involves complex or lengthy calculations. If you a r e n o t s u r e h o w t o a n s w e r t h e q u e s t i o n , d o n o t l e a v e it unanswered. Try t o e l i m i n a t e a b s u r d c h o i c e s a n d g u e s s f r o m t h e re m a i n i n g o n e s . M o s t o f t h e times four of the choices are absurd and your answer is no longer a guess. M a n y things may help you to realiz e t h a t a p a r t i c u l a r c h o i c e i s a b s u r d . Some of them are listed below. o o o The answer must be positive but some of the choices are negative so eliminate all the negative ones. The answer must be even but some of the choices are odd so e l i m i n a t e a l l t h e odd ch o i c e s . The answer must be less then 100, but some of the choices are greater than 100 (or any other value) so eliminate all choices that are out of range. The answer must be a whole number, but some of the choices are fractions so eliminate all fractions. These are some examples. There may be numerous situations where you can apply this tactic and find the correct answer even if you do not know the right way to solve the problem.

o o

Example questions with solutions


T h e fo l l o w i n g a r e s o m e e x a m p l e s , w h i c h w i l l h e l p y o u t o m a s t e r t h e s e types of questions.

Example
If 25% of 220 equals 5.5% of X, what is X? (A) 10 (B) 55 (C) 100 (D) 110 (E) 1000

Solution:
Since 5.5% of X equals 25% of 220, X is much greater than 220. So, choices A, B, C, and D are immedia t e l y e l i m i n a t e d b e c a u s e t h e s e a r e n o t l a r g e r t h a n 220. And the correct answer is choice E.

(Note: An important point here is that, even if you know how to solve a problem, if you immediately see that four of the five choices are absurd, just pick the remaining choice and move on.)

Example
Sc i e n c e s t u d e n t s c h o o s e e x a c t l y o n e o f t h r e e f i e l d s ( i . e . m e d i c a l s c i e n c e s , e n g i n e e r i n g s c i e n c e s a n d c o m p u t e r s c i e n c e s ) . I f , i n a c o l l e g e , t h r e e -fifths of t h e s t u d e n t s c h o o s e m e d i c a l s c i e n c e s , o n e -forth of the remainin g s t u d e n t s t a k e computer sciences, what percent of the students take engineering sciences? (A) 10 (B) 15 (C) 20 (D) 25 (E) 30

Solution:

T h e l e a s t c o m m o n d e n o m i n a t o r o f 3 / 5 a n d 1 / 4 is 2 0 , s o a s s u m e t h a t t h e r e a r e 2 0 s t u d e n t s i n t h a t c o l l e g e . T h e n t he n u m b e r o f s t u d e n t s c h o o s i n g m e d i c a l sciences is 12 (3/4 of 20). Of the remaining 8 students, 2 (1/4 of 8) choose computer sciences. The remaining 6 choose engineering sciences. As 6 is 30% of 20, the answer is E.

Student Guide

46

Example
If a school cafeteria needs C cans of soup each week for each student and there are S students, for how many weeks will X cans of soup last? (A) CX/S (B) XS/C (C) S/CX (D) X/CS (E) CSX

Solution:

Replace C, S and X with three easy to use numbers. Let C=2, S=5 and X=20. N o w e a c h s t u d e n t will n e e d 2 c a n s p e r w e e k a n d t h e r e a r e 5 s t u d e n t s , s o 1 0 cans are needed per week and 20 cans will last for 2 weeks. Now put these values in choices to find the correct one. The choices A, B, C, D and E become 8, 50, 1/8, 2 and 200 respectively. So the choice D represents the correct answer.

2.3

Quantitative Comparison Questions

S o m e o f t h e q u e s t i o n s i n the Q u a n t i t a t i v e s e c t i o n o f t h e t e s t m a y b e q u a n t i t a t i v e c o m p a r i s o n q u e s t i o n s . T h e Following text will explain you the format and techniques u need to attempt the questions of this format.

2.3.1 Question format


Such questions consist of two quantities, one in column A and the other in column B. You have to compare the two quantities. The information concerning one or both quantities is presented be fore them. Only the following four choices will be given: A. B. C. D. The The The The quantity in column A is greater quantity in column B is greater two quantities in both columns are equal relationship cannot be determined from the information given

And as it is clear from the choices, only one will be correct at one time. Your job is to choose one of them after careful comparison. The following text explains some simple tactics to attempt such questions.

2.3.2 How to attempt


W h e n e v e r y o u e n c o u n t e r a q u a n t i t a tive comparison question, the following guidelines will help you to find the correct answer quickly. If the question involves some variables, replace them with appropriate numbers. Here are some guidelines in choosing an appropriate number: o The very best numbers to use are 1, 0 and 1. o Often fractions between 0 and 1 are useful (e.g. 1/2, 3/4 etc.). o Occasionally, large numbers such as 10 or 100 can be used. o If there is more than one variable, it is permissible to replace each with the same number. o D o n o t i m p o s e a n y u n -s p e c i f i e d c o n d i t i o n s o n n u m b e r s . C h o o s e them randomly. Eliminate the choices and choose from the remaining ones. For example If you found the quantities ever equal, the correct choice could never be A or B, so, eliminate A and B. A quantitative comparison question can be treated as an equation or inequality. Either: Column A < Column B, or Column A = Column B, or

Student Guide

47

Column A > Column B So, you can perform similar operation on both columns to simplify the problem just as in equations (or inequalities).

Example:

m > 0 and m 1

m2

m3
change simple, So the m can

In this example divide both the quantities by m2. This will column A to 1 and column B to m. Now the comparison is very a s w e k n o w t h a t m is greater than 0 and cannot be 1. relationship is not determinable using the current information. b e both greater than 1 or between 0 and less than 1.

2.3.3 Example Explanations


Example 1:

questions

with

Answers

and

A student earned a 75 on each of her first three math tests and an 80 on her fourth and fifth tests. A B Average after 4 tests A. B. C. D. The The The The Average after 5 tests

quantity in column A is greater quantity in column B is greater two quantities in both columns are equal relationship cannot be determined from the information given

Remember you want to know which average is higher, not what the averages are. After 4 tests, the average is clearly less than 80, so an 80 on the fifth test had to raise the average. So the answer is choice (B).

Example 2:
A B T h e t im e i t t a k e s t o The time it takes to drive 40 miles at 35 d r i v e 3 5 m i l e s a t 4 0 mph mph quantity in column A is greater quantity in column B is greater two quantities in both columns are equal relationship cannot be determined from the information given

A. B. C. D.

The The The The

Once again there is no need for calculation, as the speed in column B is higher than that in column A. It is obvious that it will take less time to travel shorter d i s t a n c e a t a g r e a t e r s p e e d . S o t h e v a l u e i n c o l u m n A i s l a r g e r . T h e a n s w e r is option (A).

Example 3:
A B

Student Guide

48

20 2
A. B. C. D. The The The The

5 5

quantity in column A is greater quantity in column B is greater two quantities in both columns are equal relationship cannot be determined from the in formation given

Square each column: equal and the answer is choice (C).

20 20 = =5 2 4

and

5 25 = 5 =5 5 .

So both columns are

Example 4:
A B

13 y
To solve this question, subtract

15 y 13 y
from both columns to get

13 y 13 y = 0

column A and for column B. As there are no restrictions, be greater than, less than or equal to 0. So the correct choice is (D).

15 y 13 y = 2 y

for

2 y can

2.4

Data Interpretation Questions

These questions are based on the information that is presented in the form of a graph, chart or table. Most of the data is presented graphically. The most common types of graphs are line graphs, bar graphs and circle graphs. The objective of such questions is to test your ability to understand and analyze statistical data.

2.4.1

Data interpretation questions always appear in sets, you are presented with some data in any format (chart, graph or table), and you will then be asked with some questions about that data. The following example explains the format of such questions.

Question Format

Example: Question 1:
W h a t i s t h e a v e r a g e s a l e , i n m i l l i o n R s . , f o r t h e p e r i o d 1 9 9 4 -2 0 0 0 ? (A) 5.5 (B) 6.0 (C) 7.0 (D) 8.0 (E) 8.5

Question 2:
For which year, the percentage increase in sales from the previous year is the greatest. (A) 1995 (B) 1996 (C) 1999 (D) 2000 (E) 2001

2.4.2 How to attempt

Student Guide

49

Do not try to answer such questions immediately, first of all read the presented data carefully. You must be very clear about the data and its meanings even before reading the first question. Do not confuse numbers with percents. This confusion is most likely to occur when data is presented in pie graphs. F o r e x a mple in the following graph

Sales in million Rs.

12 10 8 6 4 2 0 1994 1995 1996 1997 1998 1999 2000 2001 Years

Now it would be a great mistake here to think that sales of TVs & VCRs is 15% more than the sales of Computers in 2001 by XYZ Corporation. To know this you have to calculate it as T r y t o a v o i d u n -n e c e s s a r y c a l c u l a t i o n s . M o s t o f t h e q u e s t i o n s c o u l d easily be solved by observation and estimation. Use estimation to eliminate the choices, if you are not able to find the correct answer w i t h o u t c a l c u la t i o n . F o r e x a m p l e t o s o l v e Q u e s t i o n 1 p r e s e n t e d i n t h e e x a m p l e a t t h e s t a r t o f t h i s s e c t i o n , i f y o u a r e n o t s u r e o f t h e correct answer, you can then try to cut down the number of possible choices by observation. You are being asked to tell the percentage increase. W h e r e as, in year 2000, the sale is decreasing instead of increasing, so, you can immediately eliminate choice (D) in that question.

15 100 = 60% 25 .

Student Guide

50

Your answers must be based upon the information presented in the g i v e n c h a r t s a n d g r a p h s . I f y o u r k n o w l e d g e c o n t r a d i c t s a n y o f the data presented, ignore what you know and stick to the presented data. The presented data should be the only base for your calculations and estimations. Always use the proper units, there may be some questions that ask you t o c o m p a r e d i f f e r e n t d a t a items possibly from different data sets. Be careful about the units used to represent the data. Because graphs and charts present data in a form that enables you to readily see the relationships among values and to make quick c o m p a r i s o n s , y o u s h o u l d a l w a ys try to visualize you answer in the same format as the original data was presented. Be sure that your answer is reasonable. For example, the profit could never increase the actual sales, or the expenses could never be negative etc. While answering the question, first of all eliminate such unreasonable choices, and then choose from the remaining ones.

2.5
1

Practice exercise
(A) 49 ( B ) 49.5 (C) 50 ( D ) 50.5 (E) 5 1

What is the average of positive integers from 1 to 100 inclusive?

If

x + y = 6, y + z = 7 ,

and

x+ z =9,

what is the average of

x, y

and (E)

z?

11 (A) 3
3 In the

11 (B) 2
diagram below,

(C) lines

22 3
l and

( D ) 11 m are not

22

parallel.

If A represents the average measure of all the eight angles, what is the value of A?

A = 45 45 < A < 90 ( C ) A = 90 ( D ) 90 < A < 180 (E) A = 180


(A) (B)

Aslam has 4 times as many books as Salman and 5 times as many as Javed. If Javed has more than 40 books, what is the least number of books that Aslam could have? (A) 200 (B) 205 (C) 210 (D)220 (E) 2 4

Student Guide

51

Aslam is now 3 times as old as Javed, but 5 years ago, he was Javed was. How old is Aslam now? (A) 10 (B) 12 is 10, what is y? (C) 24 (D)30

5 times as (E) 3 6

If

x% o f y 10 (A) x 100 (B) x 1000 (C) x x (D) 100 x (E) 10

Answer Key
1 2 3 4 5 6 D A A D D C

Student Guide

52

Analytical Ability

3.1.1 Question format Each analytical reasoning question is a logical puzzle, based on a given set of conditions. Like mathematical questio ns, these questions have exactly one correct answer, which is what you need to select. Analytical reasoning questions are presented in groups of four or five questions. Each group is based on a short passage followed by a set of conditions. Occasionally, t h e r e a r e g r a p h s a n d t a b l e s i n s t e a d o f a p a s s a g e . To understand the general format of the question, consider the following example. Question 1 - 4 : A s p a r t o f t h e i r s p o r t s p h y s i c a l , s e v e n c o l l e g e a t h l e t e s F, G, H, I, J, K and L a r e b e i n g w e i g h e d . I n announcing the results of the physical exams, the coach has given the following information. i. None of the athletes is exactly the same weight as another athlete. ii. K is heavier than L, but lighter than H. iii. I is heavier than J iv. Both F and G are heavier than H. 1 Each of the following could be true EXCEPT A. F is the heaviest. B. G is the heaviest. C. I is the heaviest. D. More than three athletes are heavier than K. E. More than three athletes are lighter than K. Which of the following, if true, would be sufficient to determin e w h i c h athlete is the lightest? A. I is the heaviest B. I is lighter than K C. K is heavier than J D. J is heavier than K E. Exactly five students are lighter than F.

3 If J is heavier than F, how many different rankings by weight, of the athletes are possible? A. 1 B. 2 C. 3 D. 4 E. 5 4 If H is heavier than I, which A. Is weight is equal to the B. Is weight is equal to the C. Js weight is equal to the D. J is the second lightest. of the following CANNOT be true? average of Fs weight and Gs weight. average of Ks weight and Ls weight average of Ks weight and Ls weight

Answers:

1. E
3.1.2 How to attempt

2. D

3. C

4. A

Simplify the information by using abbreviations and symbols. The first step is to strip away all of the excess verbiage from the given passage and then to abbreviate the remaining key words with single letters. For example, in the question statement five musicians a bassist, a drummer, a guitarist, a pianist, and a trumpeter are performing in a t a l e n t s h o w , y o u s h o u l d i m m e d i a t e l y a b b r e v i a t e t h e m B , D , G , P a n d T.

Student Guide

53

You can use abbreviated letters to represent a whole sentence also. You should use symbols to represent conditions. You may develop your own symbolic conventions for this. The objective is to convert the problem i n t o n o t a t i o n s , s o t h a t , i t i s e a s i l y un d e r s t a n d a b l e . T h e fo l l o w i n g i s a basic set of symbols, which are most commonly used. A represents the statement Akbar is going. B represents the statement Babur is going. Symbo l ~ Meaning Not And ~A Examples Akbar is not going. Or you can say, it i s n o t t h e c a s e t h a t A k b a r i s going. Akbar and Babur are going. Akbar is going and Babur is not going. Akbar or Babur is going. Akbar is going or Babur is not going. If Akbar is going then Babur is going. If Akbar and Babur are going, then Saleem is going.

A A

B ~B

Or If, then

AB A ~B

Babur is going, if and only if Akbar is going. Before learning the tactics to attempt an analytical reasoning question, you must be familiar with some ba sic logic facts, which are explained in the following text. Consider A and B are two statements. o A is true means ~A is false. o ~A is true means A is false. o (A B ) i s t r u e m e a n s b o t h A a n d B a r e t r u e . o (A B) is t r u e m e a n s e i t h e r A o r B o r b o t h a r e f a l s e . o (A B ) i s t r u e m e a n s e i t h e r A o r B o r b o t h a r e t r u e . o (A B ) i s f a l s e m e a n s b o t h A a n d B a r e f a l s e . o ~(A B ) i s e q u i v a l e n t t o ( ~ A ~ B ) . o ~(A B ) i s e q u i v a l e n t t o ( ~ A ~ B ) . o If (A B ) i s t r u e t h e n If A is true B is also true. If A is false B may be true or false. o If (A B ) i s f a l s e t h e n A i s t r u e a n d B i s f a l s e . o (A B ) i s e q u i v a l e n t t o ( ~ B ~A) o (A B ) i s t r u e m e a n s : If A is true B is true. If A is false B is false. o (A B) is false means: If A is true B is false. If A is f a l s e B i s t r u e . o (A B ) i s e q u i v a l e n t t o [ ( A B) ( B A)]. You must be familiar with the most common types of analytical r e a s o n i n g q u e s t i o n s . T h e f o l l o w i n g f o u r t y p e s occur more frequently than the others, and when you see them, you should immediately know what you need to do to answer them. o Which of the following could be true? I f o n l y o n e o f t h e answer choices could be true, then each of the other four choices m u s t b e false; that is, each one must violate at least one of the given conditions.

If and only if

A B (A B) S A B

Student Guide

54

Which of the following must be true? S i n c e o n l y o n e o f t h e answer choices must be true, then for each of the choices, either it is false or it is possibly (but not definitely) true. You have to choose only that choice which is definitely true. Which of the following cannot be true? Since only one of the answer choices cannot be true, then each of the other choices could be true. The correct answer is the only choice, which v i o la t e s a t l e a s t o n e o f t h e g i v e n c o n d i t i o n s o r i s o t h e r w i s e inconsistent with what you know must be true. How many possibilities are there? T h i s q u e s t i o n a s k s , H o w many different ways are there to satisfy all of the given c o n d i t i o n s ? H e r e , y o u m u s t s y s t e ma t i c a l l y c o u n t o r l i s t a l l o f t h e possibilities that do not violate any of the conditions.

Identify the key words that serve to limit the situation. Certain words are critical to your understanding of the situation. Be sure to i n c o r p o r a t e y o u r s y m b o l s . Some frequently used key words are listed below: All But Directly Possible Fewer Impossible Same Smallest Always Can be Each Entire First Last Some Greatest At least Cannot be No fewer than Every If Least The Least None At most Consecutive No more than Exactly If and only if Most The Most

After Before Different Only Except Imme d i a t e l y Must be Unless

Note that certain key words have only one function, to rule out a potential ambiguity. E l i m i n a t i n g t h e c h o i c e s i s a l w a y s a g o o d s t r a t e g y . W h i l e e l i minating the choices, first of all, eliminate those which are ruled out by individual conditions: Then work through the remaining choices. Study conditions, not merely for what they state but also for what they imply. C e r t a i n a n a l y t i c a l r e a s o n i n g q u e s t ions resemble the inference questions you find in the reading comprehension section. To answer them correctly, you must understand not only what the conditions state explicitly, but also what they imply. O f t e n t h e k e y t o a n s w e r i n g a n a l y t i c a l r e a s o n i n g q u e stions is to organize the given information in a list or table. On some analytical reasoning questions, an excellent way to deal with the information is to draw a simple diagram, picture, or map. This is p a r t i c u l a r l y h e l p f u l w h e n y o u a r e d e a l i n g w i t h t h e physical or temporal order of things. It is much easier to tell whether person A can be seated opposite person B if you have sketched a diagram of the table; it is easier to know whether person C is older or younger than person D if y o u h a v e e n t e r e d a l l of t h e g i v e n i n f o r m a t i o n o n a t i m e l i n e ; a n d i t i s easier to determine whether town E is east of town W if you have drawn a simple map.

Student Guide

55

3.1.3 Example Explanations


Questions 1-5:

questions

with

Answers

and

Six actors - - - - Bob, Carol, Dave Ed, Frank, and Gra ce audition for a part in an off-B r o a d w a y p l a y . T h e a u d i t i o n s w i l l t a k e p l a c e o v e r f o u r c o n s e c u t i v e d a y s , starting on a Thursday. Each actor will have one audition; the days on which the different actors will audition must conform to the following condit ions. i. ii. iii. iv. v. vi. vii. 1 At least one audition will take place each day. No more than two auditions will take place on any day. No more than three auditions will take place on any two consecutive days. Bobs audition must take place on Saturday. Carols audition must take p lace on the same day as another audition. Franks audit ions must take place on the d a y b e f o r e G r a c e s audition. Daves audition must take place on a day after Eds audition.

If only one audition takes place on Thursday which actor could have that audition? (A) Bob (B) Carol ( C ) Dave (D)Frank (E) Grace If Bobs and Franks auditions are on the same day, which of the following must be true (A) Daves audition will take place on Thursday (B) Daves audition will take place on Friday (C) Graces audition will take place on Thursday (D)Carols audition will take place on Sunday (E) E d s a u d i t i o n w i l l t a k e p l a c e o n S u n d a y If the director decides to hold two auditions on Thursday and two on Sunday, how many actors would be eligible to audition on Friday? (A) 1 (B) 2 (C) 3 (D)4 (E) 5 If Ed and Grace have their audit ions on the same day which of the following must be true? (A) Eds audition will take place on Thursday. (B) Franks audition will take place on Friday. (C) Carols audition will take place on Saturday. (D)Graces audition will take place on Saturday. (E) C a r o l s a u d i t i o n w il l t a k e p l a c e o n S u n d a y . If Eds audition is on Saturday, which of audition on the same day as any other actor? (A) Bob (C) Ed (E) Gr (B) Carol (D)Frank ace the following actors cannot

Questions 6-10:
During the first half of the year, from January through June, the chairperson of the mathematics department will be on sabbatical. The dean of the college has a s k e d e a c h o f t h e s i x p r o f e s s o r s i n t h e d e p a r t m e n t --- Arkes, Borofsky, Chang, D e n t u r e , H o b b e s , a n d L e e --- t o s e r v e a s a c t i n g c h a i r p e r s o n d u r i n g o n e o f

Student Guide

56

those months. Th e m a t h e m a t i c i a n s c a n d e c i d e t h e o r d e r i n w h i c h t h e y w i l l serve, subject only to the following criteria established by the dean. i. Chang will serve as chairperson in February. ii. Arkes will serve as chairperson before Hobbes does. iii. B o r o f s k y a n d D e x t e r w i l l s e r v e as chairpersons in consecutive months. 6 Which of the following professors could serve as chairperson in January? ( A ) Bo r o d f s k y 7 ( B ) Chang (C) Dexter (D)Hobbes (E) L e e

In how many ways can the schedule be made up if Lee has to serve as chairperson in May? (A) 1 (B) 2 (C) 3 (D)4 (E) 6 If Lee serves in April, all of the following could be true EXCEPT (A) Arkes serves in January ( B ) Hobbes serves in march (C) Borofsky serves in may (D)Borofsky serves in June (E) H o b b e s s e r v e s i n J u n e

Student Guide

57

If Borofsky serves in May, what is the latest month in which Arkes could serve? (A) January (B) February ( C ) March ( D ) April (E) J u n e

10 Which of the following CANNOT be true? (A) Arkes and Lee serve in consecutive months. (B) Lee and Hobbes serve in consecutive months. (C) Hobbes and Dexter serve in consecutive months. (D)Arkes and Chang serve in consecutive months. (E) B o ro f s k y a n d C h a n g s e r v e i n c o n s e c u t i v e m o n t h s .

Solutions 1-5:
First express each of the conditions symbolically: B, C, D, E, F, and G : 1 audition each Days: Thu, Fri, Sat, Sun Each day: 1 or 2 auditions 2 consecutive days: 2 or 3 auditions B=Sat C x F<G E<D 1 A violates the condition that Bobs audition will take place on Saturday (B=Sat). B violates the condition that Carols audition cannot be the only audition on a particular day (C x ) . C h o i c e s C a n d E a r e i m p o s s i b l e . S i n c e D a v e s a u d i t i o n m u s t t a k e p la c e o n a d a y a f t e r E d s a u d i t i o n ( E < D ) a n d G r a c e s audition must take place on a day after Franks audition (F<G) neither can take place on Thursday. Only choice D does not violate any of the given conditions, so this is the correct answer. 2 The condition that Bobs and Franks auditions are on the same day completely d e t e r m i n e s t h e s c h e d u l e . They must take place on Saturday (B=Sat). To avoid having more than three auditions on two consecutive days, there can be o n l y o n e a u d i t i o n o n F r i d a y a n d o n e o n S u n day, which means there will be two on Thursday. Since Frank must have to precede Grace (F<G), Graces audition will take place on Sunday. Since Ed must precede Dave, Eds audition will take p l a c e o n T h u r s d a y a n d D a v e s a u d i t i o n o n F r i d a y . F i n a l l y , C a r o l s a u d i t i o n w i l l be the second audition on Thursday. The final schedule is C and E on Thursday, D on Friday, B and F on Saturday and G on Sunday. Only choice B is consistent with this schedule, so B is the correct choice. S i n c e o n l y o n e a u d i t i o n c a n take place on Friday, it cannot be Carols (C x); and, of course, it cannot be Bobs (B = Sat). Any of the other four actors could audition on Friday as indicated in the following schedules: E/F on Thu, D on Fri, B on Sat, C/G on Sun C/F on Thu, E on Fri , B on Sat, D/G on Sun C/E on Thu, E on Fri, B on Sat, D/G on Sun E/F on Thu, G on Fri, B on Sat, C/D on Sun So the correct choice is D. T h e o n l y s c h e d u l e that fulfils the conditions is F on Thu, E/G on Fri, B on Sat, and C /D on Sun. Only choice E is consistent with this schedule. Since Ed and Bobs auditions are both taking place on Saturday, eliminate choices A and C. Since Carole must audition on the same day as another actor, eliminate B. F i n a l l y , s i n c e D a v e s a u d i t i o n m u s t t a k e p l a c e o n S u n d a y (E < D), Franks audition must take place on Thursday and Graces audition on Friday (F < G). E l i m i n a t e c h o i c e D . T h e c o m p l e t e s c h e d u l e i s : C / F o n T h u , G o n F r i , B / F o n S a t , a n d D on Sun.

4 5

Solutions 6-10:

Let A, B, C, D, H, L represents professor names. C =February, A<H, B < < D a n d D < < B

Student Guide

58

6 7

Only choice E is there, which does not violate any of the conditions, so is the correct choice. W i t h C s e r v i n g i n F e b r u a r y a n d L i n M a y , t h e o n l y c o nsecutive months a v a i l a b l e f o r B a n d D a r e M a r c h a n d A p r i l . Then since A mu s t s e r v e is b e f o r e H i n J u n e . There are two possible schedules, depending on the order of B and D, so the correct choice is B. If L serves in April, the consecutive months available for B and D are May and June; so choices C and D could be true. Since A must serve before H, choices A and B must be true, only choice E cannot be true. S i n c e A m u s t s e r v e b e f o r e H d o e s , A c a n n o t s e r v e i n J u n e . Can A serve in April? No, because then, D would serve in June (B<<D or D<<B), and again A would not precede H. Th e l a t e s t t h a t A c o u l d s e r v e i n M a r c h , w h i c h c o u ld o c c u r i n t h e f i n a l o r d e r: L , C , A , D , B a n d H .

1 0 The only professors that can serve in January are A and L, so, one of them m u s t s e r v e i n J a n u a r y , a n d n e i t h e r s e r v e s i n F e b r u a r y . S o choice A cannot be true.

3.2

Logical Reasoning

Each logical reasoning question requires you to analyze an argument presented in a short passage. Often you are asked either to find a conclusion that is a logical consequence of the passage, or to choose a statement that, if true, strengthen or weakens the argument.

3.2.1 Question format


Logical reasoning questions are based upon a passage called argument. You have to analyze the argument presented in the passage. The passage is followed by a question. Occasionally, there is m o re t h a n o n e q u e s t i o n r e l a t e d be the same p as s ag e. N o m a t t e r w h a t t h e n u m b e r i s , t h e q u e s t i o ns always aim at your ability to understand the conclusion reached by the author of the passage, and to give a r g u m e n t a n d c o n t r a a r g u m e n t s . Logical reasoning questions are a lot like reading comprehension questions in a v e r b a l s e c t i o n . For each logical reasoning question, the argument is followed by a multi choice question. The choices are simple statements. Mostly the question statement b e g i n s w i t h t h e p h r a s e w h ic h o f t h e f o l l o w i n g s t a t e m e n t s . H e r e a r e a few examples: Which of the following statements is an assumption on which the conclusion of this argument is based? W h i c h o f t h e f o l l o w i n g s t a t e m e n t s i d e n t i f i e s a f l a w i n t h e r e a s o n i n g o f this argument? W h i c h of the following statements can be most reasonably inferred, from the statements in the given passage? Which of the following statements, if true, would most seriously, weaken the argument offered? W h i c h o f t h e f o l l o w i n g s t a t e m e n t s , i f t r u e , w o u l d s t r e n gthen the conclusion in the preceding argument? Which of the following statements would be the most important to know to evaluate the argument given in the preceding paragraph? Every logical reasoning question does not fit this mold, but you should try.

Student Guide

59

3.2.2 How to attempt


While attempting logical reasoning questions, you should read the question statement before reading the argument. Then you should analyze the argument presented in the passage. You must know what aspect of the a r g u m e n t y o u a r e t o c o n c e n t r a t e o n , a n d f o c u s o n i t . By this, you will n o t b e unnecessarily wasting your time . You must be able to spot the question type by reading the question statement. If you do this, you will be better able to approach the argument in hand. Th e f o l l o w ing six categories are t h o s e w h i c h m o s t c o m m o n ly occur:

Assumption :

Q u e s t i o n s t h a t t e s t y o u r ability to recognize the premises on which an argument is based, often take the following forms: o The conclusion above depends on which of the following assumptio n s ? o The author of the passage above makes which of the following assumptions? o In the passage above, the author assumes which of the following statement to be true?

Inference:

Questions, which test your ability to go beyond the authors e x p l i c i t s t a t e m e n ts and see what these statements imply, may be worded like these. o It can be inferred from the passage above that the author believes that o Which of the following is implied by the passage above? o From the information above, which of the following is the mos t reasonable inference?

Conclusion:

Questions that test your ability to determine what claim can logically be made on the basis of evidence in the passage above? o If the statements above are true, which of the following in a conclusion that can be properly drawn? o The statements in the passage, if true, best supports which of the following conclusions?

Central Point:

Questions that test your ability to understand the thrust of an argument. o The statement sited above conveys which of the following propositio n s ? o The author of the passage above argues that o W h i c h o f t h e f o l l o w i n g e x p r e s s e s t h e p o i n t t h e a u t h o r o f the passage above makes?

Support:

Questions that test your ability to recognize whether an assertion supports or undermines an argument. o Which of the following, if true, best supports the authors conclusion? o Which of the following, if true, most weakens the authors conclusion?

6 Argument Evaluation: an argument.


o o

Questions that test your ability to judge

W h i c h o f t h e f o l l o w i n g i d e n t i f i e s a f l a w i n t he speakers reasoning? Which of the following would be most important to know when evaluating the accuracy of the argument above?

Student Guide

60

Do not try to skim the passage, read each argument carefully. It is not e n o u g h t o h a v e a g e n e r a l i d e a a b o u t t h e a r g u m e n t ; you must be able to analyze it very carefully. You must find the conclusion of the argument, which the author claims to h a v e r e a c h e d . Th a t m o s t c o m m o n s i t u a t i o n s a r e a s f o l l o w s : o T h e c o n c l u s i o n i s t h e l a s t s e n t e n c e o f the passage, often starting b y words s uch as so, therefore, thus, hence, consequently etc. o The conclusion is the first sentence of the passage followed by the supporting evidence. o Occasionally, the conclusion is not present in the passage; in this case, the question asks you to identify the co nclusion. Pay particular attention to signal words such as accordingly, for this reason, hence, although, but, except, in contrast, nevertheless, unlike etc. E l i m i n a t i n g t h e c h o i c e s i s a l w a y s the best stra t e g y i f y o u d o n o t k n o w w h a t t h e c o r r e c t a n s w e r is . T h i s p r o c e s s w i l l e l i m i n a t e s o m e o b v i o u s w r o n g choices. A n d y o u w i l l b e a b l e t o m a k e a n e d u c a t e d g u e s s from the remaining ones. Every argument is based upon certain assumptions made by the author. If a n a r g u m e n t s b a s i c p r e m i s e s a r e s o u n d , t h e a r g u m e n t i s s t r e n g t h e n e d . If a n a r g u m e n t s b a s i c p r e m i s e s a r e f l a w e d , t h e a r g u m e n t i s w e a k e n e d . In support questions, where you have to decide about weakening or strengthening the question, pinpoint what the argument assumes. Then compare that assumption with the answer choices. If the question asks you to find the choice, which most strengthens the argument, look for the c h o i c e t h a t i s m o s t i n k e e p i n g w i t h the arguments basic assumption. If the question asks you to choose the choice that most weakens the a rgument, look for the answer that casts the most doubt on that assumption. Some logical reasoning questions are essentially mini analytical reasoning questions, so, be familiar with all of the important logica l f a c t s a n d a p p l y whenever needed. Example questions with Answers and Explanations

3.2.3 Example Questions with Answers and Explanations Questions 1-2:


The microwave oven has become a standard appliance in many kitchens, mainly b e c a u s e i t o f f e r s a f a s t w a y o f c o o k i n g f o o d . Y e t , s o m e h o m e o w n e r s b e lieve that the ovens are still not completely safe. Microwaves, therefore, should not be a standard appliance until they have been carefully researched and tested. 1 Which of the following, if true, would most weaken the conclusion of the passage above? ( A ) H o me o w n e r s , o f t e n p u r c h a s e i t e m s d e s p i t e k n o w i n g t h e y m a y b e u n s a f e . (B) Those homeowners in doubt about microwave safety ought not to purchase microwaves. (C) Research and testing of home appliances seldom reveals safety hazards. ( D ) M i c r o w a v e s a r e n o t a s d a n g e r o u s a s steam irons, which are used in almost every home. (E) H o m e o w n e r s o f t e n p u r c h a s e i t e m s t h a t t h e y d o n o t n e e d . 2 Which one of the following, if true, would most strengthen the conclusion of the passage above? (A) Homeowners often doubt the advertised safety of all new appliances.

Student Guide

61

(B) Speed of food preparation is not the only concern of todays homeowner. (C) Modern homeowners have more free time than ever before. (D)Food preparation has become almost a science, with more complicated and involved recipes. (E) M a n y m i c r o w a v e o v e n s h a ve b e e n f o u n d t o l e a k r a d i o a c t i v e e l e m e n t s . 3 Years ago, a poll concluded that there are more televisions than there are b a t h t u b s i n A m e r i c a n h o m e s . No doubt that fact remains today, especially in light of the growing popularity of home computers. Now, in addition to owning televisions for entertainment, more and more families are purchasing TV monitors for use with a personal computer. We can safely guess that there are still many more people staring at a picture tube than singing in the shower. Which of the following statements can be inferred from this passage? (A) Personal computers probably cost less than installing a shower or bathtub. (B) People can wash themselves without a tub or shower, but they cannot watch television unless they own a television set. ( C ) TV monitors will work with personal computers in place of regular computer monitors. (D)As many computers are sold today as television sets a few years ago. (E) M o r e t e l e v i s i o n m o n i t o r s a r e n o w u s e d w i t h p e r s o n a l c o m p u t e r s t h a n a r e u s e d t o w a t c h c o m m e r c i a l t e l e v i s io n b r o a d c a s t s . Some scientists have proposed that, over two hundred million years ago, one giant land mass, rather than various continents and islands, covers one third of the earth. Long before there was any human life, and over vast periods of time, is l a n d s a n d c o n t i n e n t s d r i f t e d a p a r t . A u s t r a l i a w a s t h e f i r s t t o s e p a r a t e , while South America and Africa were late in splitting apart. Some islands, of course, were formed by volcanoes and were never part of the great land mass. All the following would s upport the authors claim EXCEPT ( A ) Many of the plants of the South American rain forests are markedly similar to those of African rain forests . ( B ) A u s t r a l i a h a s m o r e a n i m a l s t h a t a r e n o t f o u n d in a n y o t h e r c o n t i n e n t t h a n have several of the much larger continents. (C) Volcanic islands like Hawaii have ecosystems very different from those of continental lands with the same average temperature. (D)The plants of similar conditions in South America have less in common with t h o s e o f A u s t r a l i a t h a n w i t h t h o s e o f A s i a , A f r i ca o r E u r o p e . (E) T h e p r i m i t i v e l a n g u a g e s o f A u s t r a l i a a r e u n l i k e t h o s e o f A f r i c a , w h i c h resembles those of South America. 5 Every Saturday, Amir has pizza for lunch and then goes to the movies. I f t h e s t a t e m e n t a b o v e i s t r u e , w h i c h o f t h e f o l l o w i n g s t a t e m e n t s must also be true? 1 If it is not Saturday, than Amir is not having pizza for lunch and is not going to the movies. 2 If Amir has pizza for lunch and then goes to the movies, it is Saturday. 3 I f A m i r h a s p i z z a f o r l u n c h , b u t d o e s n o t g o t o t h e m o vi e s , i t i s n o t a Saturday. (A) 1 (B) 2 (C) 3 (D)1 (E) 2 6 only only only and 2 only and 3 only

Antifreeze lowers the melting point of any liquid to which it is added so that t h e l i q u i d w i l l n o t f r e e z e i n c o l d w e a t h e r . It is commonly used to maintain the c o o l i n g s y s t e m i n a u t o m o b i l e r a d i a to r s . O f c o u r s e , t h e w e a t h e r m a y b e c o m e so cold that even antifreeze is not effective, but such a severe climatic c o n d i t i o n r a r e l y o c c u r s i n w e l l -t r a v e l e d p l a c e s .

Student Guide

62

Which of the following can be deduced from the passage? ( A ) Well-t r a v e l e d p l a c e s h a v e m e a n s o f t r a n s p o r t a t i o n o t h e r t h a n a u t o m o b i l e s . ( B ) Antifreeze does not lower the melting point of certain liquids in extreme conditions. (C) Severe climatic conditions rarely occur. (D)It is not often that many travelers who use antifreeze have their cooling system freeze. (E) An t i f r e e z e r a i s e s t h e m e l t i n g p o i n t o f s o m e l i q u i d s .

Solutions:
1 The conclusion of the passage is that, because of safety concerns, more research and testing ought to be done before microwaves become standard household appliances. I f , h o w e v e r , r e s e a r c h a n d testing are ineffective means of discerning safety problems (as choice C says), then research and testing would be irrelevant. This criticism seriously weakens the conclusion. So choice C is the correct answer. If many microwave o v e n s h a v e b e e n f o u n d t o leak radioactive elements (as choice E says), then the conclusion that microwaves should not be standard a p p l i a n c e s u n t i l t h e y a r e m o r e c a r e f u l l y r e s e a r c h e d a n d t e s t e d is further s t r e n g t h e n e d b e c a u s e m o r e s a f e t y c o n c e r n s n e e d t o b e a d d r e s s e d . So, choice E is the correct answer. Though Choices A and B may well be true, they cannot be inferred from the information in the passage. But choice C can be inferred since, more and more families are purchasing TV monitors for use with a personal computer. TV monitors m u s t w o r k w i t h t h e s e compute rs, ot h e r w i s e , p e o p l e w o u l d n o t b u y them for that purpose. Choices D and E may or may not true, but they are not i n f e r e n c e s f r o m t h e p a s s a g e , s i m p l y a d d i t i o n a l i n f o r m a t i o n . So , t h e c o r r e c t choice is C. If Australia w a s t h e f i r s t c o n t i n e n t t o s e p a r a t e , i t w o u l d f o l l o w t h a t i t s f l o r a and fauna would develop in isolation over a longer period of time. Similarly, we may expect the plants and animal of South America and Africa that separated later, to be more alike. Choices A, B, and D support these ideas. T h e s e p a r a t e l y d e v e l o p e d i s l a n d s a r e d i f f e r e n t at i s a l s o i n a c c o r d w i t h t h e p as s ag e . However the languages of all the continents would have developed in i s o l a t i o n , s i n c e m a n d i d n o t e v o l v e u n t i l a f t e r t h e b r e a k-u p o f the landmass, and it is surprising that African and South American languages are similar. Human likeness or differences are irrelevant to the claims of the passage. So choice E is the correct answer. This logical reasoning question is very easy as soon as you express the given statement symbolically. If it is Saturday, then Amir has Pizza and goes to Movies translates as

which is equivalent to . So if either P or M is false, then S is false. Therefore, 3 is true, neither 1 nor 2 are true. So, the correct choice is C.

(~ P ~ M ) ~ S

S (P M ) .

This is equivalent to

~ ( P M ) ~ S ,

C h o i c e D i s t h e c o r r e c t a n s w e r . Since severe climatic conditions rarely occur in well-t r a v e l e d p l a c e s , i t i s n o t n e c e s s a r i l y t r u e t h a t I t is not often that m a n y t r a v e l e r s w h o u s e a n t i f r e e z e h a v e t h e i r c o o l i n g s y s t e m s f r e e z e . Choice A mentions other means of transportation, which is not addressed in the p as s ag e . C h o i c e B r e f e r s t o c e r t a i n l i q u i d s .

Student Guide

63

Verbal Ability

T h e p u r p o s e o f the Ve rb a l Tes t i s t o e v a l u a t e a n d a n a l y z e y o u r English comprehension and understanding towards the language. The questions will be basically asked to judge the sentence completion, analogy and critical reading skills. T h e q u e s t i o n s o f d i f f e r e n t t y p e s i . e . a b o u t sentence completion and analogy testing will be asked randomly. The questions about the critical reading however will be asked separately.

4.1

A s a lready discussed, this section will consist of the following types of questions i.e . s e n t e n c e c o m p l e t i o n , a n a l o g y a n d t h e c r i t i c a l r e a d i n g . T h e d e t a i l a b o u t e a c h s e c t i o n i s a s below ;

About the Verbal Questions

4.1.1

Sentence Completion

The questions that come under this category are provided with various choices. You are asked to complete the sentences by filling in the blanks with the most suitable choice. The questions for sentence completion can be related to any of the other areas of study i.e. science, literature, history, geography etc but the subject matter would not hinder your language abilities. Y o u are asked to complete the sentence with the use of correct grammar or vocabulary. These questions try to determine your ability to recognize the correct sentence s t r u c t u r e , r i g h t g r a m m a r a n d h o w y o u m a k e t h e c o r r e ct c h o i c e o f v o c a b u l a r y .

Techniques fo r Sentence Completion


For the sentence completion a few choices are given that could be selected for c o m p l e t i n g t h e s e n t e n c e s . Only one choice is correct out of the several choices. Y o u h a v e t o c o m p l e t e t h e s e n t e n c e b y s e l e c t i n g t h e c o r r e c t c h o i c e a c c o rding to the grammar or vocabulary. For making the right choice you can benefit from the following techniques; After you read the incomplete sentence do not look at the choices. Try to think about the correct answer yourself. If you think that you have completed the sentence and found the correct choice you can consult your list of choices. If the answer you thought matches one of the choices mentioned in the list that is most probably the right choice to be marked. If it does not match with the choice you can look for a synonym replacement. This tactic is very help f u l i n f i n d i n g t h e r i g h t a n s w e r , i t prevents you from confusing yourself with the wrong choices. Do not select the choice hastily. E v e n i f y o u a r e s a t i s f i e d w i t h y o u r c h o i c e t r y t o s u b s t i t u te it with the other choices so that you are more satisfied w i t h y o u r d e c i s i o n . Sometimes the other choice fits more appropriately to the sentence. When you are asked to complete a sentence, which has two spaces to be filled in, try to put the first word of every choice in the first blank. Note d o w n the choice that you find best. Now for the second blank try every second choice of all choices. Note the choice that you think is most appropriate. Check if the two selected choices are matching one of the given pair of choices. If it does then select it as your correct choice, if not then consider this pair as a wrong choice and try with the other choices. If you find difficulty in making sense out of certain words and you are not v e r y f a m i l i a r w i t h t h e m you can try to make a guess with reference to the

Student Guide

64

context of the sentence. Try to break the word into various parts and analyze its meaning e.g. if you do not know the meaning of the word c i v i l i z a t i o n b r e a k i t i n t o t w o i . e . c i v i l i z e a n d a t i o n n o w you may know the meaning of civilize and through the term ation you can make out that the word is a noun of civilize. If you find the word unfamiliar with prefixes and suffixes divide the word into its parts e.g. prerecording. This word c o n s i s t s o f b o th prefix and suffix. Y o u c a n b r e a k t h e w o r d l i k e p r e -record i n g . H e r e y o u k n o w t h a t p r e m e a n s b e f o r e , r e c o r d m e a n s t o s t o r e a n d -ing is a term of continuous tense. So you can find this break up of words quite helpful in making out the right sense. I f none of the technique works try making a guess with reference to the context. When long and complex sentences confuse you t h e n try to break that sentence into smaller more sentences by rephrasing it. After you divide it c o m p a r e w i t h t h e o r i g i n a l s e n t e n c e t o a v o i d a n y m i s i n t e r p r e t a t i o n . If you are satisfied read the smaller sentences to get the idea more clearly.

Example Questions
1. Multan ___________ a very hot climate. A. H a s B. Have C. Has been D. W i t h 2. One of the least effective ways of sorting _________ it. A. Repeat B. Repeating C. To repeat D. how repeat

information

is

le arning

3. Salman finished__________ two of his published compositions before his twelfth birthday. A. Written B. Writing C. To write D. Wrote 4. Sofia __________ collect stamps, but now she has other interests. A. Used to B. Was used to C. Used to be D. Using to 5 . A f t e r p a s s i n g t h r o u g h a g r e a t t r a u m a o f h e r h u s b a n d s d e a t h , she __________ hard to achieve mental relaxation. A. Struggled B. Struggling C. Struggle D. To struggle 6. In partnership with Pakistan, South Korea ____________on Motor way. A. Helped worked B. Helping work C. Helped working D. To help working 7. We will wait if you __________ go. A. Wanted to B. Want C. Want to

Student Guide

65

D.

Wanting to

8. If I had more time I ____________ checked my paper. A. Would have B. Would C. Would had D. Will have 9. I thought that he___________ coming today. A. Has been B. Is C. Was D. Has 10.That professor enjoys teaching and ___________. A. Writing B. Written C. To write D. Write 11.Just __________the files on my table. A. Let B. Leaves C. Stay D. Leave 12.Thank you A. B. C. D. for __________ me your book. Borrowing Lending Borrowed Had lent of insulin, it was not possible to treat

1 3 . _ _ _ _ _ _ _ _ _ _ _ _ discovery diabetes. A. Prior B. Before to the C. Prior to the D. To prior the

14.Distribute the handouts ___________ the candidates. A. Between B. Among C. To D. In 15.Only _________ were present at the seminar. A. a few people B. a little people C. a few peoples D. the little people

Answer Key
1. 2. 3. 4. 5. 6. 7. 8. A C B A A C C A 9. C 10. 11. 12. 13. 14. 15. A D B C B A

Student Guide

66

4.1.2 Analogy Questions


Analogy means similarity in examples or describing likeness between two or more words. These questions ask the reader to analyze the relationship between two words in a pair and look for another similar or equivalent pair of words. You are provided with five other pairs of words. You are expected to match the original p a i r , w h i c h i s g i v e n i n t h e q u e s t i o n w i t h o n e o f t h e p a i r s in the given choices on the bases of simila r r e l a t i o n s h i p s b e t w e e n t h e m . T h i s e x e r c i s e o r s u c h q u e s t i o n s try to determine your basic understanding towards vocabulary and your ability to recognize the relationship between words. Some questions may also ask you to s e l e c t a s u i t a b l e a n t o n y m f o r a g iv e n w o r d .

Techniques for Analogy Questions


For the analogy questions you can follow the guidelines mentioned below; Do not read the choices before you have analyzed the relationship between the pair of words, yourself. Try to understand the words more appropriately and think on which basis the relationship between the words is formed. After you reach a conclusion read the given choices afterwards to get a proper match with another pair having the same relationship. When you find yourself stuck with a word of difficult vocabulary, do not feel confused. Try to understand its meaning reference to the context or if it is somewhat familiar try to remember where and when you heard the word before. It can be a great help. Sometimes you find that there is more than one pair that fits well to the question and is appropriate for the choice, give the original pair a little more thought so that you can further study the relationship between the w o r d s a n d n a r r o w i t d o w n t o a m o r e d i s t i n c t o n e . A f t e r y o u h a v e b e en successful in finding a closer relationship you can now scrutinize the two other pairs that confused you earlier. Repeating the same procedure with these words would prove useful. Do not get caught up by the tricks of the test makers. Sometimes the q u estions are provided with very tricky and dodging choices that misguide greatly. Try to think of every choice more specifically and narrowly. If you are familiar with the parts of speech and their nature, it can be beneficial in making a more sensible ch oice. Remember if the words in the original pair are a noun and an adjective, the correct choice you make should also contain the words in the same grammatical order. Otherwise, your choice is wrong. So, if you are confused with two pairs and cannot c h o o se the correct choice you can easily look at their grammatical order and give preference to the one, which matches the original one.

Exclude the choice from your consideration that you think is incorrect, e.g. the choices that do not have the same grammat i c a l unit as of the original pair cannot match the original pair in anyway. Spend more time on considering the more possible choices. You should know about the various kinds of analogies that are more frequently asked. S o m e o f t h e c o m m o n a n a l o g y t y p e s are as follows; Synonyms Some words are linked together in a pair which means the same or h a s a s i m i l a r d i c t i o n a r y d e f i n i t i o n . e . g P r e t t y- Beautiful

i.

Student Guide

67

ii.

Describing Qualities Some pairs have some words in which one word describes the other w o r d . H e a v y- R a i n Class and Member Some pairs have words which are based on class and member basis e . g . E l e c t r o n i c s- Radio Antonyms Some pairs consist of the words that are opposite to each other e.g. Love- H a t e Describing Intensity Some pairs consist of the intensity of the other e.g. Smile - L a u g h t e r

iii.

iv.

v.

words

in

whic h

one

describes

the

vi.

Function In some pairs a word describes the function of the other word e.g. Pen- W r i t e Manners Some words in a speech describe the manners and behavior e.g. P o l i t e- S p e e c h W o r k e r- W o r k p l a c e So m e p a i r s i n a w o r d d e s c r i b e t h e p r o f e s s i o n a n d i t s w o r k p l a c e e . g . D o c t o r- Clinic

vii.

viii.

Example Questions 1.
HEIGHT: MOUNTAIN Depth : Shade : Weight Speed: Mineral : Trench Tree : Age Highway Mine

(A) (B) (C) (D) (E)

2.
(A ) (B) (C) (D) (F)

OBLIVIOUS : AWARENESS Comatose Serene Erudite Adroit Invigorate : Consciousness : Composure : Knowledge : Skill : Energy

3.

BELLWETHER : BAROMETER

(A) Proselyte : Spark plug (B) Panhandler : Kill (C) Embezzler : Abduct (D) Cy n o s u r e : M a g n e t (F) Morass : Catalyst Student Guide

68

4.
(A) (B) (C) (D) (E)

ACT : ACTION Therapy Oblivion Liturgy Image Bowl : Thermometer : Obvious : Literature : Imagine : Bowdlerize

5.
(A) ( B) (C) (D) (E)

BIBULOUS : DRINK Rapacious : Clothing Gluttonous : Food Altruistic : Money Vegetarian : Meat Controversy : Reconcile SONG : RECITAL : Bibliography : Series : Team : Agile : Prose

6.

(A) Author (B) Episode (C) Coach (D) Dancer (E) Poetry

7.
(A) (B) (C) (D)

HOUSE : BIG Home : Live School : Daily Water : Cold Clothes : Socks

8.
(A) (B) (C) (D)

ANIMAL

: MONKEY

Zebra : Giraffe Stationery: Pencil Book : Cap Tree : Wood HEAVEY : LIGHT

9.
(A) (B) (C) (D)

Fat : Thin Stupid : Idiot Rough : Surface B e a u t i f u l : Diary

Choose the lettered word or phrase that is most nearly opposite in meaning to the word in capital letters.

10.

DETER

(A) Twist (B) Intimidate Student Guide

69

(C) E n c o u r a g e (D) Straighten (E) Supply

11.

INDIGENOUS

(A) Modern (B) Alien (C) Ornamental (D) Natural (E) Conceal

12.
(A) (B) (C) (D) (E)

THERAPEUTIC Curative Injurious Medicinal Practical Slander QUIZZICAL

13.

(A) Amused (B) Unequivocal (C) Contorted (D) Dissimilar (E) Lax

14.

ANCILLARY

(A) Dependent (B) Concomitant (C) Appendix (D) Primary (E) Buffet

15.

VIRTUOSO

(A) Wicked (B) Dazzling (C) Mediocre (D) Honorable (E) Bawdy

Answer Key 1. A 2. A 3. A 4. D 5. B 6. B 7. C 8. B 9. A 10. C 11. B 12. B 13. B 14. D 15. E

4.1.3

Critical Reading Questions

Student Guide

70

Questions related to critical reading try to judge your reading skills and how you understand and interpret what you read. The paper includes a few passages that ask answering questions related to the passage.

Techniques for Critical Reading Exercises


There are a few techniques related to the Critical Reading Questions that prove to be a good guideline for solving such questions. Do not read the questions before read i n g t h e w h o l e p a s s a g e . T r y t o skim through the whole passage and then read the questions to look for a more specific answer. Read the passage quickly with understanding but do not panic. Try to analyze what the whole passage is about and what the author really intends to convey. While reading mark the lines where you think the passage carries the most important points. These strategies would definitely help you find the answers. When you find yourself stuck with a question, do not waste your time on it and go ahead for the next questions. Sometimes, answering other questions guide you about the earlier question. But, if you still do not find the answer mark it for doing in the end more calmly, having enough time to think. T r y t o f a m i l i a r i z e y o u r s e lf w i t h t h e t y p e s o f c r i t i c a l r e a d i n g questions. Once you know the nature of such questions, you will be able to find the answers more quickly even when you are reading the passage. The examples of some commonly asked questions are as follows: o Central Ide a Mostly, questions are asked to explain the central idea or main theme of the whole passage, which analyzes how you skim through it. Sometimes, the opening and closing lines can give you a better clue about answering such questions properly. Specific Details Sometimes to analyze your scanning abilities you are asked to answer some specific details about the passage. Such questions are about when, where, which and who. You can get the answers of this kind of questions from the area of the passag e which you marked in the first reading, where you think the most important and informational remarks of the author lies. Making Inferences Most of the questions ask you to infer from the passages, making y o u r o p i n i o n a b o u t w h a t i s s a i d i n t h e p a r a g r a p h, implying meaning and making your own point of view. These questions try to assess your judgment; you must be clear in your mind about what the author is referring to and then make your own opinion according to your understanding and comprehension. R e ad a n d think about all the choices and analyze each of it logically according to your comprehension rather than the authors point of view.

Student Guide

71

Meaning in Context Some selected words from the passage are pointed out to explain t h e m w i t h r e f e r e n c e t o t h e c o n t ex t t o c h e c k y o u r r e a d i n g comprehension. Sometimes the word that describes something in a dictionary p o r t r a y s i t t h e o t h e r w a y w h e n i t a p p e a r s i n t h e context. The test tries to judge your ability to make sense of the word in the context. Authors Approach Some questions ask you to explain the mood in which the author is writing whether it is sarcastic, humorous, witty, sad etc. When you are asked questions like these you can look for certain expressions, words, phrases or exclamations, which describe the tone, mood or style of the author. The feelings of the writer are mostly exhibited through choice of words. While answering these questions read the message carefully observing particularly the use of words. Title Selection Some passages ask for selec ting a title that best suits the passage. Remember that the chosen title should not be narrowly or broadly selected. Try to avoid choosing those titles that describes only one or two paragraphs but the one, which is a p p l i c a b l e t o t h e w h o l e p a s s a g e a n d p o rt r a y s i t b e s t .

Example Questions Passage I:


We are profoundly ignorant about the origins of language and have to content ourselves with more or less plausible speculations. We do not even know for certain when language arose, but it seems likely that i t g o e s back to the earliest history of man, perhaps half a million years. We have no direct evidence, but it seems probable that speech arose at the same time as tool making and the earliest forms of specifically human cooperation. In the great Ice Ages of the Pleistocene period, our earliest human ancestors established the Old Stone Age culture; they made flint tools and later tools of bone, ivory, and antler; they made fire and cooked their food; they hunted big game, often by methods that called for c onsiderable cooperation and coordination. As their material culture gradually improved, they became artists and made carvings and engravings on bones and pebbles, and wonderful paintings of animals on the walls of caves. It is difficult to believe that the makers of these Paleolithic cultures lacked the power of speech. It is a long step Admittedly, from the earliest flint weapons to the splendid art of the late Old Stone Age: the first crude flints date back perhaps to 500,000 B.C., while the finest ac hievements of Old Stone Age man are later than 100,000 B.C.; and, in this period, we can envisage a corresponding development of language, from the most primitive and limited language of the earliest human groups to a fully developed language in the flowering time of Old Stone Age culture. How did language arise in the first place? There are many theories about this, based on various types of indirect evidence, such as the language of children, the language of primitive societies, the kinds of changes that have taken place in languages in the course of recorded history, the Student Guide

72

behavior of higher animals like chimpanzees, and the behavior of people suffering from speech defects. These types of evidence may provide us with useful pointers, but they all suffer f r o m l i m i t a t i o n s , a n d m u s t b e treated with caution. When we consider the language of children, we have to remember that their situations are quite different from that of our earliest human ancestors, because the child is growing up in an environment where t h e r e i s a l r e a d y a f u l l y d e v e l o p e d l a n g u a g e , a n d i s surrounded by adults who use that language and are teaching it to him. For example, it has been shown that the earliest words used by children are mainly the names of things and people (Doll, Spoon, Mummy): but, this does not prove that the earliest words of primitive man were also the names of things and people. When the child learns the name of an object, he may then use it to express his wishes or demands: Doll!: often means Give me my doll! Or Ive dropped my doll: pick it up for me!; the child is using language to get things done, and it is almost an accident of adult teaching that the words used to formulate the childs demands are mainly nouns, instead of words like Bring! Pick up!; and so on. 1 The main idea of this excerpt is A. to provide evidence of the origin of language. B. t o p r e s e n t t h e n e e d f o r l a n g u a g e . C. t o d i s c u s s h o w e a r l y m a n c o m m u n i c a t e d . D. to present the culture of early man. E. to narrate the story of English. T h e o r i e s o f t he o r i g i n o f l a n g u a g e i n c l u d e a l l o f t h e f o l l o w i n g EXCEPT A. Ch a n g e s o c c u r r i n g t h r o u g h t h e y e a r s . B. T h e n e e d t o c o m m u n i c a t e . C. La n g u a g e o f c h i l d r e n . D. T he first mans extensive vocabulary. E. Co m m u n i c a t i o n a m o n g p r i m i t i v e m e n . T h e p u r p o s e o f t h e d i s c u s s i o n o f t h e w o rd , D o l l , i s i n t e n d e d t o A. Trace the evolution of a noun. B. S u p p o r t t h e f a c t t h a t n a m i n g t h i n g s i s m o s t i m p o r t a n t . C. I n d i c a t e h o w a d u l t s t e a c h l a n g u a g e t o c h i l d r e n . D. S h o w t h e e v o l u t i o n o f m a n y m e a n i n g s f o r o n e w o r d . E. Ev i n c e m a n s m u l t i p l e u s e s o f s i n g l e w o r d s The i m p l i c a t i o n o f t h e a u t h o r r e g a r d i n g t h e e a r l y e l e m e n t s o f language is that A. T h e r e w e r e s p e c i f i c r e a l s t e p s f o l l o w e d t o d e v e l o p o u r l a n g u a g e . B. C a r e m u s t b e e x e r c i s e d w h e n e x h u m i n g w h a t w e c o n s i d e r t h e r o o t s of language. C. W e o w e a d e b t o f g r a t i t u d e t o t h e c h i m p a n z e e c o n t r i b u t i o n . D. Adults created language in order to instruct their children. E. Language was fully developed by primitive man. If we accept that primitive man existed for a very long period of time without language, then we may assume that A. La n g u a g e i s n o t n e c e s s a r y t o m a n s e x i s t e n c e . B. La n g u a g e d e v e l o p e d w i t h t h e d e v e l o p i n g c u l t u r e o f p r i m i t i v e s . C. Primitives existed in total isolation from one another.

F.

Student Guide

73

D. E.
6

Children brought about a need for language. Ma n k i n d w a s n o t i n t e n d e d t o c o m m u n i c a t e .

A f t e r a r e a di n g o f t h i s a r t i c l e , o n e m i g h t i n f e r t h a t A. So c i e t y c r e a t e s p r o b l e m s w i t h l a n g u a g e . B. La n g u a g e i s f o r a d u l t s t o i n s t r u c t c h i l d r e n . C. Society uses language to improve itself. D. With the evolution of language came wisdom. E. La n g u a g e b r i n g s p o w e r . Answer Key 1. A 2. D 3. C 4. B 5. B 6. E

Student Guide

74

COMMON QUERIES

Student Guide

75

5
Q

About the General Test


W h a t i s t h e G e n e r a l T e s t generally a b o u t ?

The NTS G e n e r a l Test is generally divided into three sections i.e. T h e Verbal Section, Quantitative Section and the s e c t i o n o f A n a l y t i c a l Reasoning. T h e d u r a t i o n o f t h e s e t e s t s is 120 minutes w h i c h y o u c a n yourself manage for each section, as there is no specific time distribution for any.

T h e f o l l o w i n g t i m e t a b l e informs of the number of questions in each section as well as the marks given for that section: Test Sections Verbal Analytical Reasoning Quantitative No Questions 25 35 40 of Marks 25 35 40

W h a t K i n d o f Q u e s t i o n s a r e asked ?

T he nature of the test is obje c t i v e t y p e , e ach section is b a s e d o n M u l t i p le Choice Questions (MCQs), and every question will have at least two choices or five choices at the most.

What are the Contents of the Test?

T h e General Test i s g e n e r a l l y d i v i d e d i n t o t h r e e s e c t i o n s i . e . T h e V e r b a l, Q u a n t i t a t i v e a n d A n a l y t i c a l R e a s o n i n g. A l l t h e q u e s t i o n s a r e b a s e d o n Multiple - C h o i c e Q u e s t i o n f o r m a t . The Verbal section of the test is based on completing sentences with the appropriate words, c o m p r e h e n s i o n e x e r c i s e s t o a s s e s s t h e b a s i c k n o w l e d g e a b o u t t h e v o c a b u l a r y a n d g r a m m a r o f t h e l a nguage. Q u e s t i o n s b a s i c a l l y about sentence completion, analogy, finding antonyms, and critical reading are a s k e d . In the Q u a n t i t a t i v e S e c t i o n the NTS tries t o m e a s u r e y o u r b a s i c mathematical skills and check your understanding of the elementary level m a t h e m a t i c s . T h e M C Q s i n t h i s s e c t i o n a r e b a s e d o n a r e a s l i k e A rithmetic, Algebra and Geometry, data analysis etc. In the Analytical R e a s o n i n g Section variou s o b j e c t i v e t y p e q u e s t i o n s a r e asked to analyze the capability of every student with regard to t heir problem solving aptitude in the i r d a i l y l i f e a c t i v i t i e s a n d a l s o t h e a c a d e m i c interests. Q ue s t i o n s a s k e d a r e b a s i c a l l y o n A n a l y t i c a l R e a s o n i n g a n d L o g i c a l Re a s o n i n g .

From Where Can I Get Help to Prepare for the Test?

Student Guide

76

NTS does not refer to one spec i f i c b o o k f o r c o n s u l t a t i o n d u r i n g t h e t e s t p r e p a r a t i o n . H o w e v e r , t h e c a n d i d a t e s c a n g o t h r o u g h t h is S t u d e n t G u i d e , published by NTS, which is easily available in the market. It contains s a m p l e q u e s t i o n p a p e r s as well as e x a m p l e s o f t h e types of q u e s t i o ns y o u may find in each segment . You can also practice by solving the sample tests and the other questions f o u n d i n t h is g u i d e a n d t h e N T S w e b s i t e . It would help you in becoming familiar with the nature of the test. Try practicing the tests by keeping the allotted time in mind. Give yourself an average of one minute to complete each question. It would help you pace yourself t h r o u g h o u t t h e t e s t . D o n o t l e a v e a n y q u e s t i o n u n s o l v e d . F o r f u r t h e r m e a s u r e s y o u c a n c o n s u l t m a t e r i a l s e a s i l y a v a i l a b l e a t a n y go o d b o o k s t o r e f o r T O E F L , S A T , GRE e t c .

What are the General Guidelines to Attempt the Test?

T his S t u d e n t G u i d e b y t h e N T S i s t h e r i g h t s o u r c e t o b e c o n s u l t e d . T h e sample question papers f o r a n y o f t h e t h r e e a r e a s o f s t u d y a r e a g r e a t h e l p for you. T h e t e s t is c o m p o s e d o n t h e s a m e t e s t p a t t e r n a n d t h e q u e s t i o n s are s e l e c t e d o n t h e s a m e l e v e l . You may also use additional books for your preparation. If you answer 75% of the questions correctly, you will receive an excellent score.

Can I Get My Registrat ion Cancelled?

If you cannot take the test and want to cancel your registration, you should inform the NTS Office in writing by sending an application or email at least one week before the test date. Q How Can I Get My Test Schedule or Test Centre Changed? Once you have been allotted a Test Center and the time of the test, no changes c a n be made p o s s i b l e t o t h a t s c h e d u l e . H o w e v e r , y o u c a n a s k N T S by email and NTS may try to help you, if it finds it administratively feasible.

C a n I A p p e a r f o r t h e T e s t M o re T h a n O n e T i m e ?

Yes, if you want to improve your score and ranking, you can reappear in the test whenever the new schedule is announced. However you have to r e g i s t e r a g a i n w i t h f u l l f e e p a y m e n t . You also have to wait when the next s c h e d u l e o f t h e t e s t is announced.

From Where Can I Get the Registration Forms for the General Test (NAT)?

For the paper based tests, you can get a free of charge NTS Registration F o r m , e n c l o s e d i n i t s b r o c h u r e , w h i c h i s m a d e a v a i l a b l e i n t h e N T S a llied bank b r a n c h e s , t h e Front Of f i c e o r R e c e p t i o n o f t h e N T S o r t h e a l l i e d institutes of NTS. H o w e v e r , p l e a s e e n s u r e t h a t y o u g e t t h e s e b e f o r e t h e last date of registration.

H o w C a n I R e g i s t e r f o r N T S General Test ?

A f t e r r e c e i v i n g t h e f o r m , f i l l i t a p p r o p r i a t e l y . S u b m i t it at t h e N T S A l l i e d Institute while submitting the NTS Registration Forms NTS Test Fee is Student Guide 77

charged by the NTS Bank announced specifications.

Branches / A l l i e d I n s t i t u t e s a c c o r d i n g t o t h e

F i l l t h e f o r m a c c o r d i n g t o t h e i n s t r u c t i o n s g i v e n a t t h e b a c k of it . Submi t i t by paying the T e s t F e e i n a n y b r a n c h o f N T S a l l i e d ba n k / in s t i t u t e s / o r g a n i z a t i o n s / N T S b y e m a i l o r p o s t . You get a Test Entry Coupon from the bank/ i n s t i t u t e, w h i c h s h o w s y o u y o u r R o l l N u m b e r on it.

How Shall I Fill the Forms?

A few things must be clear to you before you fill in the Form. Any incomplete or ambiguous information in the form may cause delay in the registration process and you may not be able to take the test. For filling in the forms correctly it is necessary to;

1. 2. 3. 4. 5.

Use black ink and BLOCK LETTERS to fill in the form. Write the appropriate test title, fo r which you want to register i.e. General Test Be careful while spelling your name. Write the mailing address correctly where you can be contacted in future. E n s u r e t h a t t h e T e s t F e e is s u b m i t t e d i n t h e b a n k / I n s t i t u t e t h r o u g h Challan Form attached with the Registration Form. After submitting the fee, one copy of t h e Cha llan Form must be attached t o this Registration Form and y o u w i l l n e e d t o k e e p t h e o t h e r f o r yourself . D o n o t f o rget to fill in the bank Challan details in the a p p r o p r i a t e c o l u m n s . If t h e C h a l l a n F o r m i s n o t a t t a c h e d w i t h y o u r form, the Registration Form will not be accepted and no further processing will be made on your request for registration. A l s o i n d i c a t e y o u r c h o i c e of c i t y , w h e r e y o u w a n t y o u r t e s t c e n t e r t o be. Your group and Test Center information will be intimated to you well before the test day. Specify the institute that you want your test score to be forwarded t o. You can get the list of Allied Ins t i t u t e s f r o m t h e N T S w e b s i t e . If hande d over to the same bank where you have submitted the fee. The bank will give you one part of this form as a Test Entry Coupon. You have to show this coupon at the time of test. Without this , you may n o t b e a l l o w e d t o e n t e r t h e T e s t C e n t e r .

6. 7. 8.

NOTE: These instructions are also given on the back of the Registration Form. The General Test for some allied institutes/organizations may not need NTS Registration Form.

How Can Testing?

Pay

the

Registration

Fee

for

the

P a p e r- b a s e d

You can submit the Registration F e e t h r o u g h a B a n k V o u c h e r i n a n N T S Authorized Bank along with your Registration Form. However the fee and fee deposition procedure is subject to change from time to time. The e xact particulars are c o m m u n i c a t e d o n t h e a n n o u n c e m e n t o f t h e t e s t s c h e d u l e . O n o c c a s i o n s th e T e s t F e e c a n a l s o be s u b m i t t e d a t t h e i n s t i t u t e s w h e r e t h e t e s t i s being c o n d u c t e d . Student Guide

78

H o w t o S u b m i t t h e F o r m f o r t h e P a p e r- b a s e d T e s t i n g ?

After you h a v e c o m p l e t e d t h e f o r m a c c o r d i n g t o t h e i n s t r uc t i o n s , g e t i t s u b m i t t e d a t t h e s a m e b r a n c h o f t h e b a n k/ i n s t i t u t e / o r g a n i z a t i o n f r o m w h e r e y o u c o l l e c t e d y o u r r e g i s t r a t i o n f orm. Do not forget to c o l l e c t a stamped and photographed Test Entry Coupon from the bank, which e n s u r e s y o u r i d e n t i f i c a t i o n o n t h e T est Day. A f t e r r e c e i v i n g t h e f o r m s , t h e NTS Head office checks the essential requirements for NTS registration i.e.

i ii iii iv v

Three copies of the recent photographs Candidate's name c l e a r l y w r i t t e n in the block letters Ca n d i d a t e ' s f a t h e rs c l e a r l y w r i t t e n name in block letters. Ca n d i d a t e s p o s t a l a d d r e s s c l e a r l y w r i t t e n in block letters Phone number c l e a r l y w r i t t e n e t c .

If any essential requirement is missing f r o m t h e R e g i s t r a t i o n F o r m, it is clearly marked as rejected. Note: The Registration Forms are at times differently printed for different institutes/ o r g a n i z a t i o n s . T h e i n s t r u c t i o n s f o r t h o s e f o r m s a r e a v a i l a b l e a t t h e b a c k o f t h e R e g i s t r a t i o n F o r m.

How does NTS Confirm My Registration for the P a p e r- b a s e d Testing?

A f t e r s u b m i t t i n g t h e R e g i s t r a t i o n F o r m a t th e b a n k/ in s t i t u t e/ o r g a n i z a t i o n , you get your Test Entry Coupon, which indicates your Roll Number and also serves as your Identification Slip. This slip confirms your registration. In case of any confusion or problems with the registration process you may contact NTS.

May I Register on the Day of Examination f o r the T e s t?

General

The registration of candidates on the day of the NTS Examination cannot be entertained. The candidate should have got oneset r e g i s t e r e d b e f o r e the deadline, announced by NTS fo r the close of t h e registration process.

How C an I Get Help Registration Process?

in

Case

of

Any

Problem

During

the

If you have any confusion about the registration process you can clarify this by calling the NTS Office or by sending an email.

How Can I Get the Test Schedule for the Test ?

If you have any confusion about the Test Schedule you can obtain details from the news media a s w e l l a s NTS w e b s i t e .

How Can I Get My Test Entry Coupon for the Testing?

P a p e r- b a s e d

Student Guide

79

You are given your Test Entry Coupo n when you submit your form. It serves as your Identification Slip and also informs you about the location of the center, Roll Number and other details.

What do I have to do with the Test Entry Coupon?

Take the Test Entry Coupon with you on the Test Day. At the Test Center you are asked to show the Test Entry Coupon that was given to you after you submitted your form there. The staff at the Test Center identifies you with the photograph attached on the Coupon. Remember to attach a photograph, which is n o t m o r e t h a n s i x m o n t h s o l d . If you fail to show your Test Entry Coupon to the administration on the test day or if your photograph does not match your appearance, you may not be allowed to enter the Test Center.

What Should I Take With Me to the Test Center for t h e P a p e rbased Testing?

You a r e r e q u i r e d t o b r i n g y o u r o w n s h a r p e n e d p e n c i l s a n d e r a s e r . B e s i d e s this , t h e c a n d i d a t e i s a l s o a s k e d t o s h o w h i s / h e r T e s t E n t r y C o u p o n .

W h a t Things Should I not b r i n g w i t h m e i n t h e T e s t C e n t e r f o r P a p e r- b a s e d T e s t i n g ?

Calculators, cell phones, books, booklets, any kind of paper or cheating m a t e r i a l a r e s t r i c t l y prohibited inside of the Test Center. If a n y o f t h e s e i t e m s a r e f o u n d t o b e i n y o u r p o s s e s s i o n d u r i n g t h e t e s t y o u will be disqualified f r o m t h e e x a m i n a t i o n.

What are the Rules and Regulations that are a p p l i e d t o inside t h e T e s t C e n t e r f o r t h e P a p e r- based Testing?

me

The test is taken on the day and at the time scheduled. You are asked to observe punctuality. 1. You are assigned a seat by the invigilation s t a f f . 2. Testing aids are not permitted to be taken along to the Test Center. Items like calculators, pagers, cell phones, headphones, and any paper or writing material, dictionaries etc are prohibited in the test center. 3. Y o u a r e n o t a l l o w e d t o e a t , d r i n k, o r u s e t o b a c c o d u r i n g t h e t e s t . 4. I f y o u d o n o t s h o w t h e T e s t E n t r y C o u p o n a n d y o u r p h o t o g r a p h d o e s not match, you will not be p e r m i t t e d t o t a k e t h e t e s t . 5. No discussion or communication with y o u r f e l l o w c a n d i d a t e s i s allowed during the test session. 6. You are n o t a l l o w e d t o l e a v e t h e t e s t c e n t e r w i t h o u t t h e p e r m i s s i o n of the supervisor. 7. Under no circumstances c a n the test questions or any part of a test be removed, reproduced or disclosed verbally, electronically, or physically or by any other means to any person or entity. 8. T e s t c e n t e r s d o n o t h a v e l a r g e w a i t i n g a r e a s . F r i e n d s o r r e l a t i v e s who accompany you to the test center are not permitted to wait in the near vicinity of the test center or contact you while you are taking the test.

Student Guide

80

9. 10. 11. 12. 13. 14. 15.

If you need to l e a v e y o u r s e a t a t a n y t i m e d u r i n g t h e t e s t , r a i s e your hand and ask the invigilator. T i m i n g o f t h e s e c t i o n d o e s n o t stop. Repeated unscheduled breaks will be documented and reported to NTS. NTS reserves the right to videotape any or all of the testing sessions. Dress in such a way that you can adapt to any room temperature. You cannot communicate with anyone (other than the test center staff) while the test session is in progress or even during the breaks. You are not permitted to leave the test center vicinity during the test session or breaks. If at any time during the test you believe that you have a problem with your test, or need the Invigilation Staff for any reason, raise your hand to notify the Invigilation Staff. On What Basis Can I be disqua lified from the Test? you will be

If you are found indulged in any of the following activities, disqualified from the test:

Arriving late t o t h e t e s t c e n t e r . Taking the test for someone else. Failing to provide the Test Entry Coupon. Using any objectionable material during the test. Trying to create a disturbance inside the test center. Showing ill behavior towards the invigilation staff. Trying to communicate with fellow candidates while the test is in progress. 8. T r y i n g t o t e a r p a g e s f r o m t h e q u e s t i o n p aper. 9. S h o w i n g i n d i s c i p l i n e i . e . e a t i n g , s m o k i n g e t c . 10. L e a v i n g t h e t e s t c e n t e r w i t h o u t p e r m i s s i o n .

1. 2. 3. 4. 5. 6. 7.

Q 1. 2. 3.

How Shall I Mark the Answer Sheet? Fill in the particulars carefully like y our Name, Fathers Name, Enrolment Number and the Test Name i.e. General Test and other particulars as mentioned o n t h e a n s w e r s h e e t . Read the directions carefully before you begin your test. The invig i l a t o r a n n o u n c e s t h e p r o c e d u r e f o r m a r k i n g t h e a n s w e r sheets. Listen carefully to the instructions. When you mark the a n s w e r s h e e t s , m a ke s u r e t h a t t h e s p a c e i s c o m p l e t e l y f i l l e d i n . Y o u c a n c h a n g e t h e a n s w e r, b y e r a s i n g t h e e a r l i e r c h o i c e n e a t l y . If a question is found with no answer or it has mo re than one m a r k e d c h o i c e , i t w i l l not be considered for marking. Completely fil l in the corr e s p o n d i n g c i r c l e s on t h e a n s w e r s h e e t u s i n g a l e a d p e n c i l . D o n o t u s e i n k , b a l l- p e n o r a n y o t h e r m a r k i n g material to mark the oval. Filling more than one oval will render your answer incorrect. Please fill in the oval completely and make no other marks on i t . Incorrect

4. 5. 6. 7.

Student Guide

81

Correct

Q 1.

What are the Other Test Taking Strategies T e s t?

for the

General

2. 3. 4. 5. 6.

7. 8. Q

The test is divided into three sections i.e. Verbal, Analytical Reasoning and Quantitative. The directions given at the beginning of every section will tell you the total number of questions and the recommended time. You are provided with a few blank pages in the test paper. Y o u c a n use them for your rough work. Some formulas and helping tables are given at the end of the question paper. You can take help from those for solving the questions. Keep a very careful track of time. Do not panic while solving your paper. G i v e e a c h s e c t i o n a s p e c i f i c time period and try to complete every section on time. If you find difficulty with some questions, do not waste your time by thinking about them for a long time. Try finishing the easier q u e s t i o n s f i r s t a n d you c a n g o b a c k a n d f i n i s h t h e d i f f i c u l t q u e s t i o n s l a t e r. Do not l e a v e a n y q u e s t i o n u n a n s w e r e d , if y o u a r e c o n f u s e d o r h a v e no idea a b o u t a c e r t a i n q u e s t i o n , t r y m a k i n g a g u e s s f o r t h e r i g h t answer. Do not make any extra marks on the answer sheet. Co m p u t e r m i g h t register these and consider them as another marked choice. If I am d i s q u a l i f i e d f r o m t h e T e s t , C a n I apply for it a t a L a te r Time?

If you are dismissed from the test session, you will be given a second chance and will be allowed to reappear in the next testing session.

How can I Ask for Result Reporting?

You can ask for your result by requesting it from N T S, through a phone c a l l , f a x o r b y s e n d i n g a n e - mail. Y o u c a n a l s o v i s i t t h e w e b s i t e o f N T S t o check your result s. The result s w i l l b e s e n t t o y o u b y p o s t o n r e q u e s t . A proper result would also be prepared for the institutes and the candidates c a n c h e c k u p t h e i r p e r f o rm a n c e s b y c o n t a c t i n g t h e i r r e s p e c t i v e i n s t i t u t e .

How is My NTS Test Scored?

The papers are marked according to the international marking standards. Each question carries one mark. Every question which is correctly answered will be given one mark unless t h e c o m p u t e r f i n d s a n e r r o r w i t h your marking.

Is There Any Negative Marking While Scoring?

If the answer is wrong you will lose one mark but there is no n e g a t i v e m a r k i n g f o r a n i n c o r r e c t answer.

If I ha v e a l r e a d y a p p e a r e d S c o r e Affect M y N e w S c o r e ?

in the Test, How Will the Fir s t

Student Guide

82

If the candidate has appeared more than two times in a n NTS t e s t , his/her highest score would be considered to be valid.

Is There Any Chance to Improve My Score?

I f y o u a r e n o t s a t i s f i e d w i t h y o u r t e s t s c o r e a n d y o u w a n t to improve your grade you will be allowed to apply and appear in the test again.

W h a t S h o u l d I d o , i f I R e g i s t e r a n d T h e n Ca n Not Take the Test?

If you do not appear to take the test you will be given another chance to take the test o n t h e n e x t s c h e d u l e d t e s t , but no refunds c a n be made.

Who Shall I Contact in Case of any Confusion?

If you have any confusions about the testing service, registration, test execution process and the announcement of results etc you can call, mail or fax the NTS Head office for the details. Queries for Online General Test Q. In What Ways is NTS General Test Different from Other NTS Tests? The NTS General Tests are different from its other tests in many ways. Unlike the NTS Subject tests, the General Tests aims to assess the verbal, quantitative and analytical abilities that have been attained over a period of time and that are not necessarily related to any specific field of study. Q. How Can I Register for NTS-General Tests? For the NTS, General Test Registration, you have to fill the NTS Online Registration Form on the NTS website. Before you get registered online you have to pay the registration fee as per the specified procedure for that test. You can fill the form online at our present website and submit it. You will get e-mail from NTS after the submission of the form, which indicates that your form has been received by NTS. NTS will confirm your registration by sending you an email once the bank confirms the receipt of your fee. This form is available when the NTS has announced its online test schedule. Q. What happens when I submit my Online Registration Form? After you submit your Online Registration Form you receive an email from NTS which confirms the receipt of your registration request. NTS takes 3-5 days time to check up the valid Bank Challan No from the bank. When NTS confirms the deposition of your fee from the bank, it sends you the Admission Ticket which gives you your User ID, location of the Test Center, Test Time etc. Q. When do I receive confirmation of my final Registration Status? After submitting your Online Registration Form, you receive the confirmation of your Test Registration and some other information about the test in the form of an Admission Ticket within 3-5 days, through an email by NTS. If your registration is not confirmed Student Guide

83

during the given time you can Contact Us for inquiring about the reasons. Q. How Can I Pay the Registration Fee? For the NTS General Test, the Test Fee is to be deposited in the prescribed Bank Branches or a draft to be sent to NTS or the allied institute as prescribed. The reference of your Bank Deposit Slip Number (that you received from the bank after depositing the fee) is necessary to be endorsed on your Online Registration Form for getting your registration confirmed. Q. Will NTS Confirm my Registration? Yes, after receiving a completely and properly filled NTS Online Registration form, NTS confirms your registration by sending you an email as soon as the bank confirms the receipt of your registration. Q. May I Register on the Day of Online Test? No, for the NTS Online General Test, you have to get registered by the due date, intimated on the NTS website or advertised through various newspapers prior to the test. Q. Where Shall I Contact if I Do Not Get my Registration Confirmed? If NTS does not confirm your registration within a weeks time after you submit your form, you may contact support@nts.org.pk by giving the reference of your Bank Deposit Receipt Number and your other personal particulars. Q. Can I Get My Registration Cancelled? If you cannot take the test and want to cancel your registration, you should inform the NTS Office in writing by sending an application or email at least one week before the test date. Q. How Can I Get the Test Schedule? The Test Schedule that includes the Test Date, Test Time and the Venue of the test, is sent to you by NTS through an email. Q. How Can I Get My Test Schedule or Test Centre Changed? Once you have been allotted a Test Center and the time of the test, no changes might be possible to that schedule. However, you can ask NTS by email and NTS may try to help you, if it finds feasible administratively. Q. Can I Appear for the Test More Than One Time? Yes, if you want to improve your score and ranking, you can reappear in the test whenever the new schedule is announced. However you have to register again with full fee payment. You also have to wait when the next schedule of the test is announced.

Q. Is There Any Chance to Improve My Score? If you want to improve your score and ranking, you are allowed to apply and appear in Student Guide 84

the test again. However, you have to register again, once the schedule is announced. Q. What Computer Skills are Necessary for NTS Online General Tests? For the NTS computer-based online General Tests, minimal computer skills are required. The skills that are required include, using a mouse, entering and confirming a response, changing a response, accessing the help function, and moving to the next question although a help function is available throughout the test. These Test Instructions will also be given to you before the test is given. Q. How Often is the NTS Online General Test Conducted? A series of computer-based General Tests is planned. NTS announces the Schedule from time to time on its website www.nts.org.pk and through newspaper advertisements. Q. When Should I Arrive at the Test Center? Try to arrive at the testing center at least 45 minutes before your scheduled Test Time so that no delays are made during the check -in procedures and your identification. Q. What Should I Take With Me to the Test Center? Bring your ID card with you for your identification. No other items e.g. stationary, books, calculators are required nor allowed to be taken inside the Test Centre. Q. How Long Will I be at the Test Center? Although the actual test takes 3 hours, you should be prepared to spend up to 4 hours at the center. You will spend extra time on taking Test Instructions, getting seated, and other administrative arrangements, etc. Q. What are the important Things to be careful about for Online General Tests? You should be particularly careful about a few things while taking the Online General Test: Online testing does not allow you to come back and reattempt the question that you have already marked. If you face any difficulty in logging on to your system or face any other related problem you can ask for help from any lab assistant or invigilator.

Q. How to take NTS Online Test? You will go through the following screen shots to learn how to take NTS Online test. Step: 1 This is the first screen named as Candidate Login Screen. You will enter your Candidate ID and Password provided to you by NTS

Student Guide

85

Student Guide

86

Step: 2 You will see Test Instructions Screen after you successfully login. Please read the instructions carefully to avoid any confusion during the test. After reading the instructions, press Start Test button on center bottom of the page.

Student Guide

87

Step: 3 After you click on the Start Test button your test starts and a page appears which shows your first question of the test. Each question has various choices, if you know the answer you can select the correct choice for your answer and press Next Question button. You can also add this question in the Pass Box to answer it at some another stage. You will also find some additional information about your test on this page. This information includes: Total Number of Questions in the Test Total Number of Questions Answered by you Total Number of Questions in the Pass box to be attempted later

Student Guide

88

Step: 4 If you place a question in Pass Box you will notice that the Questions in Pass Box field will increase by one.

Student Guide

89

Step: 5 If you want to answer the questions in Pass Box, simply click on Questions in Pass Box link at the top. It will take you on the following screen. Now click Answer this Question button for the question you want to answer.

Student Guide

90

Step: 6 You have selected this question from Pass Box now select its answer and proceed for next question.

Student Guide

91

Step: 7 You have attempted all questions and now this last screen will show your detailed result.

Wish You Good Luck with your Test. National Testing Service

Q. How is NTS Online Test different from paper-based Test? For the Paper-based Test, A pencil, eraser and a sharpener is required to attempt the paper-based test. You can make the changes in the answer that you have already marked. The announcement of results is delayed than the online tests. You have to fill in a separate answer sheet to mark your answers.

Where as for the Online Test, No stationery items are required to attempt online test. If you want to attempt a question at the end, you can place it in the Pass Box for attempting later. You can not reattempt a question that you have already answered. As soon as you finish the test, the result is displayed on the screen. In case of a system failure during the test, you will have to log-in again and the test will start from the same question where you had left. No information will be lost while the system was unavailable.

Student Guide

92

Q. How Can I Ask for Result Reporting? Your result is reported to you right after you finish your test. You are given the certificate fifteen days after the conduct of the test of the last batch. If you still do not get the result you can Contact Us. You can also visit the website of NTS to check your result. The final result is sent to you by email. Q. Is There Any Negative Marking While Scoring? There is no negative marking for wrong answers. However the negative marking may be activated if it is required by the allied institute or organization.

Q. What are the Rules and Regulations that apply to me in the Test Center when taking the Online General Test? Rules and Regulations: If you do not appear with the Identity Card (NIC) on the Test Center, you will not be allowed to take the test. The test will be given on the day and at the scheduled time. You are asked to observe punctuality. Arriving late at the center may disqualify you from taking the test. You are not allowed to bring any testing aids inside the test center. Nothing, except the original Identity Card is required to be taken along. You are not allowed to smoke, eat or drink inside the test center. No discussion or any form of communication with the fellow candidates is allowed during the testing session. You will also not be allowed to leave the test center without the permission of the supervisor. Test centers do not have large waiting areas. Friends or relatives who accompany you to the test center will not be permitted to wait in the test center or contact you while you are taking the test. You will be required to sign the attendance sheet before and after the test session and any time you leave or enter the premises where the test is being conducted. If you need to leave your seat at any time during the test (which shall only be allowed in case of serious illness), raise your hand and ask the invigilator. Repeated unscheduled breaks will be documented and reported to NTS. NTS reserves the right to videotape all or any of Testing Sessions and use it to determine any misconduct, etc. If at any time during the test you believe that you have a problem with your test, or need the Invigilation Staff for any reason, raise your hand to notify the Invigilation Staff.

Student Guide

93

NOTE: The rest of the queries regarding the test format, contents and other procedures have almost the same answers as of the paper based tests, given above.

Student Guide

94

DRILL TESTS General

Note: The sample papers do not include quantitatively the same number of questions as there would be in the actual papers. They are merely meant to provide conceptual guidance to the users or prospective candidates.

Student Guide

95

Drill Test I

Student Guide

96

I Quantitative Section 12 minutes is A. B. C. D. E. A 66 72 732 723 None of these B C D E No. Of Questions 10

Choose the correct answer for each question and shade the corresponding CIRCLE on the answer sheet

1. The number of degrees through which the hour hand of a clock moves in 2 hours and

2. A cylindrical container has a diameter of 14 inches and a height of 6 inches. one gallon equals 231 cubic inches, the capacity of the tank is approximately A. B. C. D. E. A 2-2/7 gallons 4 gallons 1-1/7 gallons 2-2/7 gallons None of these B C D E

Since

3. A train running between two towns arrives at its destination 10 minutes late when it goes 40 miles per hour and 16 minutes late when it goes 30 miles per hour. The distance between the two towns is A. B. C. D. E. A 720 miles 12 miles 8-6/7 miles 12-7/7 miles None of these B C D E

4. If the base of a rectangle is increased by 30% and the altitude is decreased by 20% the area is increased by A. B. C. D. E. A 25% 10% 5% 1% 4% B C D E

5. If the sum of the edges of a cube is 48 inches, the volume of the cube is A. 512 inches B. 96 cubic inches C. 64 cubic inches Student Guide

97

D. 698 cubic inches E. None of these A B C D E

6. A certain triangle has sides, which are, respectively, 6 inches, 8 inches, and 10 inches long. A rectangle equal in area to that of the triangle has a width of 3 inches. The Perimeter of the rectangle, expressed in inches, is A. B. C. D. E. A 11 16 22 23 24 B C D E

7. The cube of 1/3 is A. B. C. D. E. A 3/9 3/27 1/81 1/27 1/9 B C D E

8. In general, the sum of the squares of two numbers is greater than twice the product of the numbers. The pair of numbers for which this generalization is not valid is A. B. C. D. E. A 8,9 9,9 9,10 9,8 8,10 B C D E

9. A piece of wire 132 inches long is bent successively in the shape of an equilateral triangle, a square, a regular hexagon, a circle. The plane surface of the largest area is included when the wire is bent into the shape of a A. B. C. D. E. A Circle Square Hexagon Triangle Line B C D E

10. If pencils are bought at 35 cents per dozen and sold at 3 for 10 cents the total profit on 5 1/2 dozen is Student Guide

98

A. B. C. D. E. A 0

25 35 27 28 31 B

cents cents 1/2 cents 1/2 cents 1/2 cents C D E

Student Guide

99

II ANALYTICAL Section For question 1 to 4 No. Of Questions 20

Choose the correct answer for each question and shade the corresponding CIRCLE in the answer sheet

Three adultsR, S, and Vwill be traveling in a van with five childrenF, H, J, L, and M. The van has a driver's seat and one passenger seat in the front, and two benches behind the front seats, one bench behind the first. Each bench has room for exactly three people. Everyone must sit in a seat or on a bench, and seating is subject to the following restrictions: An adult must sit on each bench. Either R or S must sit in the driver's seat. J must sit immediately beside M. 1. Which of the following can sit in the front passenger seat? A: J B: L C: R D: S E: V A B C D E

2. Which of the following groups of three can sit together on a bench? A: B: C: D: E: A F, J, and M F, J, and V F, S, and V H, L, and S L, M, and R B C D E

3. If F sits immediately beside V, which of the following CANNOT be true? A: B: C: D: E: A J sits immediately beside S. L sits immediately beside V. H sits in the front passenger seat. F sits on the same bench as H. H sits on the same bench as R. B C D E

4. If S sits on a bench that is behind where J is sitting, which of the following must be true? A: B: C: D: H sits in a seat or on a bench that is in front of where M is sitting. L sits in a seat or on a bench that is in front of where F is sitting. F sits on the same bench as H. L sits on the same bench as S

Student Guide

100

E: M sits on the same bench as V. A B C D E

For question 5 to 9 The principal of a school is forming a committee. There are to be five members: three teachers, chosen from Mr. J, Ms. K, Ms. L, Mr. M, and Mr. N; and two students, chosen from O, P, Q, and R. The composition of the committee must conform to the following conditions: Ms. J will serve only if R is also on the committee. Ms. L will not serve unless Ms. K and O also serve. Neither Mr. M nor Mr. N will serve without the other. If P serves, neither Q nor R can serve. 5. Which of the following is an acceptable committee? A: J, L, M, N, O B: K, L, N, O, P C: K, M, N, O, R D: L, M, N, O, R E: M, N, O, P, Q A B C D E

6. How many different committees could include Mr. J and Q? A: 1 B: 2 C: 3 D: 4 E: 5 A B C D E

7. If Q and R are both on the committee, who else must be on the committee? A: J B: K C: L D: M E: O A B C D E

8. If M is not on the committee, each of the following must be on the committee EXCEPT A: J B: L C: O D: Q E: R B C D E 9. In how many different ways can the principal select an acceptable committee? A: Fewer than 3 B: 3 Student Guide A

101

C: 5 D: 7 E: More than 7 A B C D E

For question 10 to 13 A contractor will build five houses in a certain town on a street that currently has n o houses on it. The contractor will select from seven different models of housesT, U, V, W, X, Y, and Z. The town's planning board has placed the following restrictions on the contractor: No model can, be selected for more than one house. Either model W must be selected or model Z must be selected, but both cannot be selected. If model Y is selected, then model V must also be selected. If model U is selected, then model W cannot be selected. 10. If model U is one of the models selected for the street, then which of the following models must also be selected? A: B: C: D: E: A T W X Y Z B C D E

11. If T, U, and X are three of the models selected for the street, then which of the following must be the other two models selected? A: B: C: D: E: A V and W V and Y V and Z W and Y Y and Z B C D E

Student Guide

102

12. Which of the following is an acceptable combination of models that can be selected for the street? A: T, U, V, X, Y B: T, U, X, Y, Z C: T, V, X, Y, Z D: U, V, W. X, Y E: V, W, X, Y, Z A B C D E

13. If model Z is one m odel not selected for the street, then the other model NOT selected must be which of the following? A: B: C: D: E: A T U V W X B C D E

For question 14 to 16 Seven childrenF, J, K, M, R, S, and Tare eligible to enter a spelling contest. From these seven, two teams must be formed, a red team and a green team, each team consisting of exactly three of the children. No child can be selected for more than one team. Team selection is subject to the following restrictions: If M is on the red team, K must be selected for the green team. If F is on the red team, R, if selected, must be on the green team. R cannot be on the same team as S. J cannot be on the same team as K. 14. A: B: C: D: E: A Which of the following can be the three members of the Red team? F, J, and K F, R, and T J, K, and T K, M, and R M, R, and T B C D E

15. If M and F are both on the red team, the green team can consist of which of the following? A: B: C: D: E: A J, K, and R J, S, and T K, R, and S K, R, and T R, S, and T B C D E

Student Guide

103

16. If M is on the red team, which of the following, if selected, must also be on the red team? A: B: C: D: E: A F J R S T B C D E

For question 17 to 20 A mail carrier must deliver mail by making a stop at each of six buildings: K, L, M, O, P, and S. Mail to be delivered are of two types, ordinary mail and priority mail. The delivery of both types of mail is subject to the following conditions: Regardless of the type of mail to be delivered, mail to P and mail to S must be delivered before mail to M is delivered. Regardless of the type of mail to be delivered, mail to L and mail to K must be delivered before mail to S is delivered. Mail to buildings receiving some priority mail must be delivered, as far as the above conditions permit, before mail to buildings receiving only ordinary mail. 17. If K is the only building receiving priority mail, which of the following lists the buildings in an order, from first through sixth, in which they can receive their mail? A: B: C: D: E: A L, K, P, S, O, M L, K, S, P, M, O K, L, P, M, S, O K, P, L, S, O, M O, K, L, P, S, M B C D E

18. If L, M, and S are each receiving priority mail, which of the following lists the buildings in an order, from first to sixth, in which they must receive their mail? A: K, L, P, S, O, M B: L, K, O, P, S, M C: L, K, S, P, M, O D: M, L, S, P, K, O E: S, L, M, P, K, O A B C D E

Student Guide

104

19. If the sequence of buildings to which mail is delivered is O, P, L, K, S, M and if S is receiving priority mail, which of the following is a complete and accurate list of buildings that must also be receiving priority mail? A: O, L B: O, P C: P, L D: P, M E: O, P, L, K A B C D E

20. If only one building is to receive priority mail, and, as a result, O can be no earlier than fourth in the order of buildings, which of the following must be the building receiving priority mail that day? A: K B: L C: M D: P E: S A B C D E

Student Guide

105

III VERBAL Section No. Of Questions 20

Choose the correct answer for each question and shade the corresponding CIRCLE in the answer sheet

Each sentence below has one or two blanks, each blank indicating that something has been omitted. Beneath the sentence are five lettered words or sets of words. Choose the word or set of words that, when inserted in the sentence, best fits the meaning of the sentence as a whole. 1. Florence Nightingale was ___ in the development of modern medicine, ___ such practices as sanitization of hospital wards and isolation of actively infected patients. A. B. C. D. E. A a collaboratorrejecting a maverickprotesting an innovatorinitiating a pioneercriticizing an individualiststandardizing B C D E

2. As a journalist who works to overturn erroneous convictions, Griffin Nicholson was opposed to the court ruling ___ appeals for inmates who might be ___ . A. B. C. D. E. A barringculpable curbingexonerated encouraginginnocent scrutinizingeligible shieldingesteemed B C D E

3. Linda Greenhouse's articles for the New York Times are an outstanding example of ___, capsulizing prose into a necessarily limited space. A. B. C. D. E. A Callousness Brevity Intuition Propriety Fortitude B C D E

Student Guide

106

4. Roberto Clement was seen as ___ during his life because of both his selflessness on the baseball field and his humanitarian work in his native Nicaragua. A. B. C. D. E. A An individualist a grandstander a sybarite an altruist an opportunist B C D E

5. His habit of spending more than he earned left him in a state of perpetual-----but he------------hoping to see a more affluent day. A. B. C. D. A indigence: persevered in confusion: compromised by enervation: retaliated by motion: responded B C D E

6. Known for his commitment to numerous worthy causes, the philanthropist deserved------ for his-----------. A. recognition: folly B. blame: hypocrisy C. reward: modesty D. credit: altruism A B C D E

7. You should --------this paragraph in order to make your essay more-----. A. B. C. D. A Delete, succinct Enlarge, redundant Remove, discursive Revise, abstruse B C D E

8. A------glance pays-----attention to details. A. B. C. D. A Furtive: meticulous Cursory: little Cryptic: close Keen: scanty B C D E

Each question below consists of a related pair of words or phrases, followed by five lettered pairs of words or phrases. Select the lettered pair that best expresses a relationship similar to that expressed in the original pair. Student Guide

107

9. TIRADE: ABUSIVE A. B. C. D. A Diatribe: familial Satire: pungent Panegyric: laudatory Eulogy: regretful B C D E

10. SOLDIER: REGIMENT A. B. C. D. A Colonel: martinet Dancer: balletomane Singer: chorus Trooper: rifle B C D E

11. ASYLUM: SHELTER A. B. C. D. A Harbor: concealment Palisade: display Stronghold: defense Cloister: storage B C D E

12. STATIC: MOVEMENT A. B. C. D. A Humdrum: excitement Chronic: timeliness Ecstatic: decay Diligent: industry B C D E

13. INTEREST: FASCINATE A. Vex: enrage B. Vindicate: condemn C. Regret: rue D. Appall: bother A B C D E

Student Guide

108

14. LAUREL: VICTOR A. Chevrons: army B. Oscar: movie star C. Power: glory D. blue ribbon: cooking A B C D E

15. PRECEDENT: JUSTIFICATION A. B. C. D. A Kindness: obedience Authority: sanction Usage: submission Tradition: novelty B C D E

16. IMPLICATE: COMPLICATE A. Vitality: inevitable B. Empathy: sympathy C. Importune: construct D. Imply: simplify A B C D E

17. PERMEATE: RUEFUL A. B. C. D. A Truculent: merciful Sadden: pitiful Evaporate: mournful Penetrate: sorrowful B C D E

18. ENERVATE: STRENGTHEN A. B. C. D. A Aver: attribute Divert: turn Apprise: appraise Stultify: enliven B C D E

Read the passages and answer the questions given at the end: Recent technological advances in manned undersea vehicles have overcome some of the limitations of divers and diving equipment. Without vehicles, divers often become sluggish and their mental concentration was limited. Because of undersea pressure that affected their speech organs, communication among divers was difficult or impossible. But today, most oceanographers make observations by the means of instruments that are lowered into the ocean or from samples taken from the water direct observations of the ocean floor are made not only by divers of more than seven miles and cruise at the depth of fifteen thousand feet. Radio equipment buoys can be operated by remote Student Guide 109

control in order to transmit information back to land based laboratories, including data about water temp erature, current and weather. Some of mankinds most serous problems, especially those concerning energy and food, may be solved with the help of observations made by these undersea vehicles. 19. With what topic is the passage primarily concerned? A. Recent technological advances. B. Communication among divers. C. Direct observation of the ocean floor D. Undersea vehicles A B C D E

20. Divers have problems in communicating underwater because? A. B. C. D. A The pressure affected their speech organs The vehicles they used have not been perfected. They did not pronounce clearly The water destroyed their speech organs. B C D E

Student Guide

110

Drill Test II

Student Guide

111

I Quantitative Section

No of Questions

10

Choose the correct answer for each question and shade the corresponding CIRC LE in the answer sheet

1. A piece of wood 35 feet, 6 inches long was used to make 4 shelves of equal length. The length of each shelf was A. B. C. D. E. 9 8 7 7 6 feet, 1 1/2 inches feet, 10 1/2 inches feet, 10 1/2 inches feet, 1 1/2 inches feet, 8 1/2 inches

2. The tiles in the floor of a bathroom are 15/16 inch squares. The cement between the tiles is 1/16 inch. There are 3240 individual tiles in this floor. The area of the floor is A. B. C. D. E. 225 sq. yds. 2.5 sq. yds. 250 sq. ft. 22.5 sq. yds 225 sq. ft.

3. A man bought a TV set that was listed at $160. He was given successive discounts of 20% and 10%. The price he paid was A. B. C. D. E. $129.60 $119.60 $118.20 $115.20 $112.00

A
4.

Mr. Jones' income for a year is $15,000. He pays 15% of this in federal taxes and 10% of the remainder in state taxes. How much is left? A. B. C. D. E. $12,750 $9,750 $14,125 $13,500 $11,475

5. The radius of a circle which has a circumference equal to the perimeter of a hexagon whose sides are each 22 inches long is closest in length to which one of the following? A. B. C. D. E. 7 21 14 28 24

Student Guide

112

6. If a, is a multiple of 5 and b = 5a, which of the following could be the value of a + b? I. A. B. C. D. E. 60 I only III only I and III only II and III only None of these II. 100 III. 150

7. Which of the following expressions has the greatest value? A. B. C. D. E. 4 4 4 4 2 44+4 44+4 4 44 4 + 44 2 + 22

8. If (a + 3) / 5 is an integer, what is remainder when a is divided by 5? A. B. C. D. E. 1 2 3 4 5

9. The integral part of logarithm is called A. Characteristic B. Mantissa C. Solution D. Root E. None of these

A
A. B. C. D. E.

B
1 zero - 1

10. On the y-axis, the x-coordinate is

Student Guide

113

II ANALYTICAL Section No. Of Questions 20

Choose the correct answer for each question and shade the corresponding CIRCLE in the answer sheet

For question 1 to 3 A volunteer uses a truck to pick up donations of unsold food and clothing from stores and to deliver them to locations where they can be distributed. He drives only along a certain network of roads. In the network there are two-way roads connecting each of the following pairs of points: 1 with 2, 1 with 3, 1 with 5, 2 with 6, 3 with 7, 5 with 6, and 6 with 7. There are also one-way roads going from 2 to 4, from 3 to 2, and from 4 to 3. There are no other roads in the network, and the roads in the network do not intersect. To make a trip involving pickups and deliveries, the volunteer always takes a route that for the whole trip passes through the fewest of the points 1 through 7, counting a point twice if the volunteer passes through it twice. The volunteer's home is at point 3. Donations can be picked up at a supermarket at point 1, a clothing store at point 5, and a bakery at point 4. Deliveries can be made as needed to a tutoring center at point 2, a distribution center at point'6, and a shelter at point 7. 1. If the volunteer starts at the supermarket and next goes to the shelter, the first intermediate point his route passes through must be A: 2 B: 3 C: 5 D: 6 E: 7 A B C D E

2. If, starting from home, the volunteer is then to make pickups for the shelter at the supermarket and the bakery (in either order), the first two intermediate points on his route, beginning with the first, must be A: 1 and 2 B: 1 and 3 C: 2 and 1 D: 2 and 4 E: 4 and 2 A B C D E

Student Guide

114

3. If, starting from the clothing store, the volunteer next is to pick up bread at either the supermarket or the bakery (whichever stop makes his route go through the fewest of the points) and then is to go to the shelter, the first two points he reaches after the clothing store, beginning with the first, must be A: 1 and 2 B: 1 and 3 C: 4 and 2 D: 6 and 2 E: 6 and 4 A B C D E

For question 4 to 5 There are seven cages next to each other in a zoo. The following is known about the cages. Each cage has only one animal, which is either a monkey or a bear. There is a monkey in each of the first and last cages. The cage in the middle has a bear. No two adjacent cages have bears in them. The bears cage in the middle has two monkey cages on either side. Each of the two other bear cages are between and next to two monkey cages 4. How many cages have monkeys in them? A: 2 B: 3 C: 4 D: 5 E: 6 A B C D E

5. The bear cage in the middle must have A: B: C: D: E: A No other bear cage to its left No monkey cage on its right. A bear cage to its left and to its right Other bear cages next to it. No monkey cage to its left. B C D E

For question 6 to 8 A nursery class in a school has a circular table with eleven seats around it. Five girls (Kiran, Lado, Maryam, Omera and Parveen) and five boys (Farhan, Ghaus, Haris, Imdad and Jahangir) are seated around the table. None of the girls are seated in a seat adjacent to another girl. Kiran sits between Farhan and Ghaus, and next to each of them. Jahangir does not sit next to Imdad. 6. Which of the following is a possible seating order around the table? A: Empty seat, Farhan, Kiran, Ghaus, Lado, Omera, Haris, Imdad, Parveen, Jahangir, and Maryam. Student Guide 115

B: Empty seat, Farhan, Kiran, Ghaus, Lado, Jahangir, Parveen, Omera, Imdad, Maryam, Haris. C: Empty seat, Farhan, kiran, Ghaus, Omera, Jahangir, Parveen, Imdad, Maryam, Haris, Lado. D: Empty seat, Omera, Farhan, Kiran, Ghaus, Lado, Jahangir, Imdad, Parveen, Haris, Maryam. E: Empty seat, Maryam, Farhan, Kiran, Ghaus, Lado, Jahangir, Perveen, Imdad, Omera, Haris. A B C D E

7. If Lado, Haris, Maryam, Jahangir, and Ghaus are seated in that order, which of the following is a correct completion of the seating order after Ghaus? A : Kiran, Farhan, Omera, Imdad, Parveen, empty seat. B : Kiran, Farhan, Imdad, Omera, empty seat, Parveen C : Farhan, Parveen, Kiran, Imdad, Omera, empty seat. D : Kiran, Farhan, Parveen, Imdad, empty seat, Omera. E : Kiran, Farhan, Omera, empty seats, Parveen, Imdad. A B C D E

8. If Jahangir leaves his seat and occupies the empty seat, his new seating position would be between: A : Farhan and Kiran B : Maryam and Ghaus C : Kiran and Ghaus D : Imdad and Lado E : Parveen and Lado A B C D E

For question 9 to 11 Four telephone operators (Abid, Baqir, Chauhan, and Daud) each have to perform duties at the telephone exchange on four different days, Thursday through Sunday. The following is known about their duty schedule: Chauhan has his duty day before Abid. Daud has his duty day later than Baqir. 9. Which of the following is a possible order of duty days for the four operators? A: B: C: D: E: A Chauhan, Daud, Abid and Baqir. Daud, Chauhan, Abid, and Baqir. Baqir, Chauhan, Daud and Abid. Abid, Chauhan, Daud and Baqir. Abid, Baqir, Daud and Chauhan. B C D E

10. If Chauhan has his duty day on Saturday, who must have his duty day on Thursday? A: Either Abid or Daud. Student Guide

116

B: C: D: E: A

Daud Abid Either Baqir or Daud. Baqir. B C D E

11. Each of the following is possible EXCEPT: A: B: C: D: E: A Chauhan has his duty on Thursday. Baqir has his duty on Thursday. Daud has his duty on Saturday. Baqir has his duty on Sunday Abid has his duty on Sunday. B C D E

For question 12 to 13 There are 12 seats facing the blackboard in a classroom, four seats (A1, A2, A3 & A4) in that order are in row A, the first row from the blackboard. Immediately behind row A is row B with four seats (B1, B2, B3 & B4) in that order. Immediately behind row B, is the last row C with four seats (C1, C2, C3 & C4) in that order. Six students attend the class the following is known about there seating arrangement: Ejaz sits exactly in front of Comil, Seat A2 is always unoccupied Daud does not sit next to Farhat, Gharuy sits in seat A4 Hamid does not sit in seat B4 All the seats in row C always remain empty 12. If Daud sits in seat B3, then Farhat must sit in seat: A: A3 B: A1 C: B4 D: B2 E: C2 A B C D E

13. Suppose that Hamid and Ejaz are sitting in seats A1 and A3 respectively, then it CANNOT be true that seat: A: B: C: D: E: A B1 is occupied by Daud. B2 is empty B1 is empty B3 is OCCUPIED BY Comil B4 is empty B C D E

For question 14 to 17 The principal of a school is forming a committee. There are to be five members: three teachers, chosen from Mr. J, Ms. K, Ms. L, Mr. M, and Mr. N; and two students, chosen from O, P, Q, and R. The composition of the committee must conform to the following conditions: Ms. J will serve only if R is also on the committee. Ms. L will Student Guide 117

not serve unless Ms. K and O also serve. Neither Mr. M nor Mr. N will serve without the other. If P serves, neither Q nor R can serve. 14. Which of the following is an acceptable committee? A: B: C: D: E: A J, L, M, N, O K, L, N, O, P K, M, N, O, R L, M, N, O, R M, N, O, P, Q B C D E

15. How many different committees could include Mr. J and Q? A: 1 B: 2 C: 3 D: 4 E: 5 A B C D E

16. If Q and R are both on the committee, who else must be on the committee? A: J B: K C: L D: M E: O A B C D E

17. In how many different ways can the principal select an acceptable committee? A: Fewer than 3 B: 3 C: 5 D: 7 E: More than 7 A B C D E

Student Guide

118

For question 18 to 20 Three adultsR, S, and Vwill be traveling in a van with five childrenF, H, J, L, and M. The van has a driver's seat and one passenger seat in the front, and two benches behind the front seats, one bench behind the other. Each bench has room for exactly three people. Everyone must at in a seat or on a bench, and seating is subject to the following restrictions: An adult must sit on each bench. Either R or S must sit in the driver's seat. J must sit immediately beside M. 18. Which of the following can sit in the front passenger seat? A: J B: L C: R D: S E: V A B C D E

19. Which of the following groups of three can sit together on a bench? A: F, J, and M B: F, J, and V C: F, S, and V D: H, L, and S E: L, M, and R A B C D E

20. If F sits immediately beside V, which of the following CANNOT be true? A: J sits immediately beside S. B: L sits immediately beside V. C: H sits in the front passenger seat. D: F sits on the same bench as H. E: H sits on the same bench as R. A B C D E

Student Guide

119

III VERBAL Section No. Of Questions 20

Choose the correct answer for each question and shade the corresponding CIRCLE in the answer sheet

Each sentence below has one or two blanks; each blank indicates that something has been omitted. Beneath the sentence are five lettered words or sets of words. Choose the word or set of words that, when inserted in the sentence, best fits the meaning of the sentence as a whole. 1. Surprisingly enough, it is more difficult to write about the--------than about the--and strange. A. B. C. D. A specific, foreign abstract, prosaic commonplace, exotic simple, routine B C D E

2. A-----response is one that is made with----------. A. B. C. D. A stupid, fear speedy, alacrity sure, slowness harmful, grimaces B C D E

3. A----is a-------. A. B. C. D. A norm, standard criterion, mistake discipline, school doctrine, follower B C D E

4. It is widely believed that a nuclear war could ____ enough smoke and dust to block out the sun and freeze the earth. A. B. C. D. E. A billow extinguish generate duplicate decimate B C D E

Student Guide

120

5. Consumption of red meat has ____ because its fat content has become a worrisome and ____ matter. A. B. C. D. E. A abated ... dubious skyrocketed ... stressful abounded ... divisive stabilized ... newsworthy declined ... controversial B C D E

6. It takes ____ character to ____ the extremities of the arctic region. A. B. C. D. E. A an unflappable ... sustain a dictatorial ... brook a Spartan ... negotiate an inimitable ... resist a nomadic ... espouse B C D E

7. Consumers refused to buy meat products from the company because of rumors that the water supply at the meat processing plant was ______; the rumors, however, were quite ______, with no hard evidence to back them up. A. B. C. D. E. A uninspected .. reckless contaminated .. unsubstantiated impure .. damaging misdirected .. scandalous unscrupulous .. vicious B C D E

8. Many kinds of harmful viruses are unhindered when passing through different parts of the host organism; indeed, there are few organic substances which such viruses cannot______. A. B. C. D. E. A undermine disseminate aerate exterminate perforate B C D E

9. Their conversation was unsettling, for the gravity of their topic contrasted so oddly with the ______ of their tone A. B. C. D. E. A uniqueness rapidity lightness precision reverence B C D E

Student Guide

121

10. Throughout the animal kingdom, ____ bigger than the elephant. A. B. C. D. E. A whale is only the only the whale is is the whale only only whale is the whale is only B C D E

Each question below consists of a related pair of words or phrases, followed by five lettered pairs of words or phrases. Select the lettered pair that best expresses a relationship similar to that expressed in the original pair. 11. YAWN: BOREDOM :: A. dream : sleep B. anger : madness C. smile : amusement D. face : expression E. impatience : rebellion A B C D E

12. OBSTRUCTION : BUOY :: A. B. C. D. E. A construction : building boy : girl danger : red light iceberg : titanic arise : lay down B C D E

13. CONCERT : MUSIC :: A. B. C. D. E. A performance : artist exhibition : art play : actor operetta : singer rock : role B C D E

14. TEAMMATE : ADVERSARY :: A. B. C. D. E. A felon : criminal enemy : associate pacifier : agitator winner : loser friend : foe B C D E

Student Guide

122

For Question 15-20 read the following passage: A popular theory explaining the evolution of the universe is known as the Big Bang Model. According to the model at some time between twenty billion years ago, all present matter and energy were compressed into a small ball only a few kilometers in diameter. It was, in effect, an atom that contained in the form of pure energy all of the components of the entire universe. Then, at a moment in time that astronomers refer to as T = 0, the ball exploded, hurling the energy into space. Expansion occurred. As the energy cooled most of it became ma tter in the form of protons, neutrons and electrons. These original particles combined to form hydrogen and helium and continued to expand. Matter formed into galaxies with stars and planets.

15. Which sentence best summarizes this passage? A. The big band theory does not account for the evolution of the universe B. According to the Big Bang Model, an explosion caused the formation of the universe C. The universe is made of hydrogen and helium D. The universe is more than ten billion years old A B C D E

16. According to this passage when were the galaxies formed? A. B. C. D. A Ten Billion Years ago Fifteen billion Years ago At T = 0 Twenty billion years ago B C D E

17. The word compressed in the passage could best be replaced by A. B. C. D. A Excited Balanced Reduced Controlled B C D E

18. It may be inferred that A. B. C. D. A Energy and matter are the same Protons, neutrons, and electrons are not matter Energy may be converted into matter The galaxies stopped expanding as energy cooled B C D E

19. The word it in the passage refers to A. B. C. D. Energy Space Expansion Matter C D E

B Student A Guide

123

20. The environment before the Big Bang is described as all the following EXCEPT A. B. C. D. A Compressed matter Energy All the components of the universe Protons, electrons and neutrons B C D E

Student Guide

124

Drill Test III

Student Guide

125

I Quantitative Section No. Of Questions 20

Choose the correct answer for each question and shade the corresponding OVAL in the answer sheet

1. If the pattern of dots shown above is continued so that each row after Row One contains 1 dot more than the row immediately above it, which row will contain 12 dots? A. B. C. D. E. A Seven Eight Nine Ten Twelve B C D E

2. Each of Steve's buckets has a capacity of 11 gallons, while each of Mark's buckets can hold 8 gallons. How much more water in gallons can 7 of Steve's bucket's hold compared to 7 of Mark's buckets? A. B. C. D. E. A 3 7 21 24 56 B C D E The greater of the two

3. Two integers have a sum of 42 and a difference of 22. integers is A. B. C. D. E. A 22 25 28 31 32 B C D E

4. The average of five numbers is 34. If three of the numbers are 28, 30 and 32, what is the average of the other two? A. 40 B. 50 C. 60 Student Guide

126

D. 70 E. 80 A B C D E

5. In the figure above, what is the value of x? A. B. C. D. E. A 30 40 50 80 100 B C D E

6. In a certain cake, two straight cuts (made along two different radii) succeed in removing 4/15 of the total cake. What is the central angle in degrees of the piece cut? A. B. C. D. E. A 26 60 85 92 96 B C D E

7. If an equilateral triangle and a square have the same perimeter, what is the ratio of the length of the sides of the equilateral triangle to the lengths of the sides of the square? A. B. C. D. E. A 3:4 4:3 1:4 1:3 3:1 B C D E

Student Guide

127

8. If 2 and 4 each divide q without rema inder, which of the following must q divide without remainder. A. B. C. D. E. A 1 2 4 8 It cannot be determined from the information given. B C D E

9. The ratio of boys to girls in a certain classroom was 2 : 3. If boys represented five more than one third of the class, how many people were there in the classroom? A. B. C. D. E. A 15 25 30 45 75 B C D E

10. Let xy = z, where x,y,z and nonzero numbers. If x is multiplied by 3 and z is divided by 3, this is equivalent to multiplying y by A. B. C. D. E. A 1/9 1/3 1 3 9 B C D E

11. If 5x = 3, then (5x + 3) = A. B. C. D. E. A 0 9 18 36 81 B C D E

Student Guide

128

12. If the postal charges for a package are 62 cents for the first five ounces and 8 cents for each additional ounce, what is the weight of a package for which the charges are $1.66? (Assume there are 16 ounces in one pound) A. B. C. D. E. A 1.05 pounds 1.1 pounds 1.125 pounds 1.25 pounds 1.5 pounds B C D E

13. If m/n = .75, then what is 3m + 2n? A. B. C. D. E. A 0 8 14 17 24 B C D E

14. Which is greater? Column A (10/4) / (3/2) * (3/7) A. B. C. D. E. A if if if if if Column B (3/4) * (10/7) / (3/2)

the quantity in Column A is greater the quantity in Column B is greater the two quantities are equal there is no relationship between these two quantities the relationship cannot be determined from the information given C D E

15. Which of the following has a graph that is symmetric to the x-axis A. B. C. D. E. A y=x y = x2 + 3 y2 = x y = x3 x y != x B C D E

Student Guide

129

16. The prime factors of 96 are: A. B. C. D. E. A 2 and 3 6 and 8 2, 3 and 4 8 and 12 3 and 9 B C D E

17. 8 are what percent of the 6? A. B. C. D. E. A 1.25 75 125 133.333 150 B C D E

18. 2- 3 = A. B. C. D. E. A -8 8 -6 -1/8 1/8 B C D E

19. If x + 1 < 3x + 5, then A. B. C. D. E. A X > -2 x < -2 x = 0v v v v x < 2 x > 2 B C D E

20. Which of the numbers cannot be represented by a repeating decimal? A. B. C. D. E. A 11/9 23/7 v3 4 1/3 2 B C D E

Student Guide

130

II Analytical Reasoning

No. of Questions

15

Choose the correct answer for each question and shade the corresponding CIRCLE in the answer sheet

The office staff of the XYZ Corporation presently consists of three bookkeepers (L, M and N ) and five secretaries (O, P, Q, R and S). Management is planning to open a new office in another city sending three secretaries and two bookkeepers from the present staff. To do so they plan to separate certain individuals who do not function well together. The following guidelines were established to set up the new office: (a) Bookkeepers L and N are constantly finding faults with one another therefore should not be sent together to the new office. (b) N and P function well alone but not as a team. They should be separated. (c) O and R have not been on speaking terms for many months. They should not go together. (d) Since O and Q have been competing for a promotion, they should not be in one team. Based on the information given above find the correct answers to the following Questions: 1. If M insists on staying back then how many combinations are possible? A. B. C. D. 1 2 3 None

2. If L is to be moved as one of the bookkeepers, which of the following CANNOT be a possible working unit? A. B. C. D. LMOPS LMPQS LMORS LMPRS

3. If N is sent to the new Office which member of the staff CANNOT be sent? A. B. C. D. O M Q R

4. If O is sent to the new office then which of the following is a possible team? A. B. C. D. LMOPR MNOQS MNOPS LMOPS

5. If both N and Q are moved to the new office, how many combinations are possible? A. 2 B. 3

Student Guide

131

C. 4 D. 1

E
Adjacent countries

6. A map representing countries R, S, W, X, Y and Z is to be drawn. cannot have the same color in the map. The countries adjacent to each other are as follows: Each of R, S, X and Y is adjacent to W. X is adjacent to Y. Each of R and S is adjacent to Z. If X is the same color as Z then it must be true that A. B. C. D. W is a different color from any other country. S is a different color from any other country. X is the same color as Y. S is the same color as X.

Two statements, labeled I. & II, follow each of the following questions. The statements contain certain information. In the questions you do not actually have to compute an answer, rather you have to decide whether the information given in the statements I. and II. is sufficient to find a correct answer by using basic mathematics and every day facts?
7. A long distance runner has just completed running 28 miles. How long did it take him to finish the journey? I. His record speed is 8.25 miles per hour. II. His average speed through the journey was 8 miles per hour. A. Statement I. ALONE is sufficient but II. ALONE is not sufficient to answer question. B. Statement II. ALONE is sufficient but I. ALONE is not sufficient to answer question. C. Statements I. and II. TOGETHER are sufficient to answer the question NEITHER of them is sufficient ALONE. D. Statements I. and II. COMBINED are NOT sufficient to answer the question additional information is needed to find the correct answer. this this but and

8. Captain of the national cricket team has to be the most popular member of the team. Who is the captain of Pakistans national cricket team? I. Waqar is the best player on the team. II. Waseem is the senior-most member. A. Statement I. ALONE is sufficient but II. ALONE is not sufficient to answer this question. B. Statement II. ALONE is sufficient but I. ALONE is not sufficient to answer this question. C. Statements I. and II. TOGETHER are sufficient to answer the question but NEITHER of them is sufficient ALONE. D. Statements I. and II. COMBINED are NOT sufficient to answer the question and additional information is needed to find the correct answer.

Student Guide

132

E
How many boys

9. In a BCE class at CIIT, 30 boys and 10 girls registered Calculus II. passed the course? I. 5 students could not pass. II. There were 2 girls who obtained A grade.

A. Statement I. ALONE is sufficient but II. ALONE i s not sufficient to answer question. B. Statement II. ALONE is sufficient but I. ALONE is not sufficient to answer question. C. Statements I. and II. TOGETHER are sufficient to answer the question NEITHER of them is sufficient ALONE. D. Statements I. and II. COMBINED are NOT sufficient to answer the question additional information is needed to find the correct answer.

this this but and

10. A horse ran 100 miles without stopping. What was its average speed in miles per hour? I. The journey started at 8 PM and ended at 4 AM the following day. II. The horse ran 20 miles per hour for the first 50 miles. A. Statement I. ALONE is sufficient but II. ALONE is not sufficient to answer question. B. Statement II. ALONE is sufficient but I. ALONE is not sufficient to answer question. C. Statements I. and II. TOGETHER are sufficient to answer the question NEITHER of them is sufficient ALONE. D. Statements I. and II. COMBINED are NOT sufficient to answer the question additional information is needed to find the correct answer. this this but and

Student Guide

133

11. How much time will computer a need to solve 150 problems? I. The computer needs 50 seconds to solve one problem. II. Computer never takes more than 60 seconds to solve a problem. A. Statement I. ALONE is sufficient but II. ALONE is not sufficient to answer question. B. Statement II. ALONE is sufficient but I. ALONE is not sufficient to answer question. C. Statements I. and II. TOGETHER are sufficient to answer the question NEITHER of them is sufficient ALONE. D. Statements I. and II. COMBINED are NOT sufficient to answer the question additional information is needed to find the correct answer. this this but and

12. How many pencils does Raheel have? I. He bought two boxes each containing 10 pencils. II. He lent two pencils to Khaleel A. Statement I. ALONE is sufficient but II. ALONE is not sufficient to answer this question. B. Statement II. ALONE is sufficient but I. ALONE is not sufficient to answer this question. C. Statements I. and II. TOGETHER are sufficient to answer the question b ut NEITHER of them is sufficient ALONE. D. Statements I. and II. COMBINED are NOT sufficient to answer the question and additional information is needed to find the correct answer.

13. In a certain farm there are 47 goats. How many large brown goats are there? I. 16 of the goats are large. II. There are 18 brown goats in the farm. A. Statement I. ALONE is sufficient but II. ALONE is not sufficient to answer question. B. Statement II. ALONE is sufficient but I. ALONE is not sufficient to answer question. C. Statements I. and II. TOGETHER are sufficient to answer the question NEITHER of them is sufficient ALONE. D. Statements I. and II. COMBINED are NOT sufficient to answer the question additional information is needed to find the correct answer. this this but and

14. Can there be more than 200 pictures in a 60-page book? I. There is at least one picture in each page. II. There are no more than 3 pictures in any page. A. Statement I. ALONE is sufficient but II. ALONE is not sufficient to answer this question. B. Statement II. ALONE is sufficient but I. ALONE is not sufficient to answer this question. C. Statements I. and II. TOGETHER are sufficient to answer the question but NEITHER of them is sufficient ALONE. D. Statements I. and II. COMBINED are NOT sufficient to answer the question and additional information is needed to find the correct answer.

A Student Guide

134

15. If P > Q and R > S, then, P + R > Q + S. Is X > Y? I. X + A > Y + B II. A > B A. Statement I. ALONE is sufficient but II. ALONE is not sufficient to answer this question. B. Statement II. ALONE is sufficient but I. ALONE is not sufficient to answer this question. C. Statements I. and II. TOGETHER are sufficient to answer the question but NEITHER of them is sufficient ALONE. D. Statements I. and II. COMBINED are NOT sufficient to answer the question and additional information is needed to find the correct answer.

Student Guide

135

III VERBAL Section

Choose the correct answer for each question and shade the corresponding CIRCLE in the answer sheet No of Questions 10

Each sentence below has one or t wo blanks, each blank indicates that something has been omitted. Beneath the sentence are five lettered words or sets of words. Choose the word or set of words that, when inserted in the sentence, best fits the meaning of the s e nt e n c e a s a w h o l e . 1. Although its publicity has been ___, the film itself is intelligent, well-acted, handsomely produced and altogether ___ A. tasteless respectable B. extensive moderate C. sophisticated moderate D. risqu crude E. perfect spectacular A 2. B C D E

The Inuit natives of Alaska's North Slope worry that ___ oil exploration might ___their sensitive natural environment. A. B. C. D. E. A additionalassist currentbolster curtailedshatter unregulateddamage controlledreassess B C D E

3.

Ants live in colonies based on ___; each member contributes to the good of all by actively working with others in performing necessary tasks. A. B. C. D. E. A Heredity Individualism Cooperation Reasoning Instinct B C D E

Each question below consists of a related pair of words or phrases, followed by five lettered pairs of words or phrases. Select the lettered pair that best expresses a relationship similar to that expressed in the original pair. 4. STUDYING: LEARNING:: A. running : jumping B. investigating : discovering C. reading : writing

Student Guide

136

D. dancing : swimming E. talking : listening A 5. B C D E

AFTERNOON : DUSK :: A. B. C. D. E. A breakfast : dinner yesterday : tomorrow Sunday : Saturday night : dawn arise : lay down B C D E

6.

VIBRATION: SOUND : A. B. C. D. E. A gravity : pull watercolor : paint accident : death worm : reptile arrive : home B C D E

7.

RUN : RACE :: A. B. C. D. E. A walk : pogo stick swim : boat fly : kite sink : bottle repair : automobile B C D E

Read the passages and answer the questions asked at its end. Almost a century ago Alfred Binet, a gifted psychologist, was asked by the French Ministry of Education to help determine who would experience difficulty in school. Given the influx of provincials to the capital, along with immigrants of uncertain stock, Parisian officials believed they needed to know who might not advance smoothly through the system. Proceeding in an empirical manner, Binet posed many questions to youngsters of different ages. He ascertained which questions when answered correctly predicted success in school, and which questions when answered incorrectly foretold school difficulties. The items that discriminated most clearly between the two groups became, in effect, the first test of intelligence. Binet is a hero to many psychologists. He was a keen observer, a careful scholar, an inventive technolo gist. Perhaps even more important for his followers, he devised the instrument that is often considered psychology's greatest success story. Millions of people who have never heard Binet's name have had aspects of their fate influenced by instrumentation that the French psychologist inspired. And thousands of psychometricians specialists in the measurement of psychological variables earn their living courtesy of Binet's invention. Student Guide

137

Although it has prevailed over the long run, the psychologist's version of intelligence is now facing its biggest threat. Many scholars and observers and even some iconoclastic psychologists feel that intelligence is too important to be left to the psychometricians. Experts are extending the breadth of the concept proposing much intelligence, including emotional intelligence and moral intelligence. They are experimenting with new methods of ascertaining intelligence, including some that avoid tests altogether in favor of direct measures of brain activity. They are forcing citizens everywhere to confront a number of questions: What is intelligence? How ought it to be assessed? And how do our notions of intelligence fit with what we value about human beings? In short, experts are competing for the "ownership" of intelligence in the next century. 8. According to the passage, which of the following is most similar to the "barometer" developed by Binet? A. B. C. D. E. A 9. The S.A.T. or other standardized college admission test. The written portion of a driver's license test. Open tryouts for a varsity athletic team An electronic scan of brain-wave activity. The trivia questions of a game show. B C D E

The author suggests which of the following about "citizens everywhere"? A. They do not have a sufficiently accurate definition of intelligence to evaluate recent scientific developments. B. They stand to benefit from recent progress in the scientific assessment of intelligence. C. The experiments they are performing with new methods of intelligence measurement are valuable and interesting. D. They are at odds with the experts over who should have the right to define "intelligence." E. Traditionally they have not given careful consideration to some important issues concerning intelligence.

A B C D E 10. As used in line # 8, "discriminated" most nearly means A. B. C. D. E. A equalized predetermined showed favoritism displayed intolerance distinguished B C D E

Student Guide

138

Drill Test IV

Student Guide

139

I Quantitative Section No. Questions Of 20

Choose the correct answer for each question and shade the corresponding CIRCLE in the answer sheet

1. If the length of BC is twice the length of AC, what are the coordinates of B where A=( x,y)? A. B. C. D. E. A (x,2y) (-x,2y) (2x,y) (-2x,y) (-2x,2y) B C D E

2. The average of five numbers is 34. If three of the numbers are 28, 30 and 32, what is the average of the other two? A. 40 B. 50 C. 60 D. 70 E. 80 A B C D E

Student Guide

140

G 3. In the figure above, rectangle AEJL has been divided into 8 congruent squares with each of the 8 squares having an area of 16. What is the length of AE + MF + LG+ AL + BK + CJ + DH + EG? A. B. C. D. E. A 32 44 88 128 176 B C D E which of the following is an

4. For any positive integer x, #x = x/3 and &x = 9 /x. expression for the product of #x and &x? A. B. C. D. E. A 3x x 1 x3 /64 27 x3 B C D E

5. In a certain town, p gallons of gasoline are needed per month for each car. long will q gallons last at this rate given that there are r cars in town? A. B. C. D. E. A pr/q qr/p r/pq q/pr pqr B C D E

How

Student Guide

141

6. Let xy = z, where x,y,z are nonzero numbers. If x is multiplied by 3 and z is divided by 3, this is equivalent to multiplying y by A. B. C. D. E. A 1/9 1/3 1 3 9 B C D E

7. If x, y, and z are different positive odd integers and x + y + z = 11, what is the greatest possible value of z? A. B. C. D. E. A 10 9 8 7 6 B C D E

8. If the postal charges for a package are 62 cents for the first five ounces and 8 cents for each additional ounce, what is the weight of a package for which the charges are $1.66? (Assume there are 16 ounces in one pound) A. B. C. D. E. A 1.05 pounds 1.1 pounds 1.125 pounds 1.25 pounds 1.5 pounds B C D E

9. What fraction of two weeks is 24 minutes? A. B. C. D. E. A 1/120 1/336 1/840 1/2880 1/20160 B C D E

Student Guide

142

10. If the vertices of a triangle are at (0,0), (-3, 4) and (3, 4), what is the area of the triangle? A. B. C. D. E. A 4 6 12 14 18 B C D E

11. A water-tank has a base with dimensions 2 feet by 6 feet. If a cube with each side 1 foot is totally immersed in the water, how many inches will the water rise? (12inches = 1 foot) A. B. C. D. E. A 1 2 4 8 It cannot be determined from the information given B C D E

12. In the figure above, the quadrilateral ABCD is a trapezoid with x = 2. The diameter of each semicircle is a side of the trapezoid. What is the sum of the lengths of the four drawn semicircles? (Round to the nearest whole number.) A. B. C. D. E. A 13 16 19 22 31 B C D E

Student Guide

143

13. If n + 3 = n x 3, then n = A. B. C. D. E. A 0.5 1.5 2 2.5 3 B C D E

14. If an equilateral triangle and a square have the same perimeter, what is the ratio of the length of the sides of the equilateral triangle to the lengths of the sides of the square? A. B. C. D. E. A 3:4 4:3 1:4 1:3 3:1 B C D E

15. A restaurant has a special whereby both parents can eat for $20 and each child can eat for $5. Assuming a family group consists of both parents and at least one child, what is the maximum number of family groups that could have attended given that the restaurant took $115? A. B. C. D. E. A 6 5 4 3 2 B C D E

16. Which of the following points lays in the interior of the circle whose radius is 10 and whose center is at the origin? A. B. C. D. E. A (-9, 4) (5, -19) (0, -10) (10, -1) (0,15) B C D E

Student Guide

144

Choose the correct answer for each question and shade the corresponding CIRCLE in the answer Analytical No. Of 15 sheet Reasoning Questions 17. If the perimeter of the rectangle ABCD is 14, what is the perimeter of BCD? A. B. C. D. E. 7 12 7 + 29 86 It cannot be determined from the information given. B C D E

II

A D

B C

18. The roots of ax2 + bx + c = 0 are real only if A. B. C. D. E. A b2 b2 b2 b2 b2 B 4ac 0 4ac = 0 + 4ac = 0 4ac < 0 4bc < 0 C D E

19. The two numbers, whose sum is -13 and product -30, are A. B. C. D. E. A 2, 15 2, -15 -3, 10 3, 10 -3, -13 B C D E

20. Let A = total area of five circles of radius r and let B = total area of three circles of radius s. If A = B, then r / s = A. B. C. D. E. A 3/5 3 / 5 3 / 5 (3 ) / 5 3 B C D E

Two statements labeled I & II, follow each of the following questions. The statements contain certain information. In the questions you do not actually have to compute an answer, rather you

Student Guide

145

have to decide whether the information given in the statements I. and II. is sufficient to find a correct answer by using basic mathematics and every day facts? 1. What day of the week is today? I. Today is March 25. II. Akram left Pakistan on Wednesday. A. Statement I. ALONE is sufficient but II. ALONE is not sufficient to answer question. B. Statement II. ALONE is sufficient but I. ALONE is not sufficient to answer question. C. Statements I. and II. TOGETHER are sufficient to answer the question NEITHER of them is sufficient ALONE. D. Statements I. and II. COMBINED are NOT sufficient to answer the question additional information is needed to find the correct answer. this this but and

2. Can any of the four rivers be more than 300 meters wide? I. The narrowest of the four rivers is 240 meters wide. II. Average width of the four rivers is 300 meters. A. Statement I. ALONE is sufficient but II. ALONE is not sufficient to answer question. B. Statement II. ALONE is sufficient but I. ALONE is not sufficient to answer question. C. Statements I. and II. TOGETHER are sufficient to answer the question NEITHER of them is sufficient ALONE. D. Statements I. and II. COMBINED are NOT sufficient to answer the question additional information is needed to find the correct answer this this but and

3. If it is raining then there must be clouds. Are there clouds? I. It is not raining. II. It rained yesterday. A. Statement I. ALONE is sufficient but II. ALONE is not sufficient to answer this question. B. Statement II. ALONE is sufficient but I. ALONE is not sufficient to answer this question. C. Statements I. and II. TOGETHER are sufficient to answer the question but NEITHER of them is sufficient ALONE. D. Statements I. and II. COMBINED are NOT sufficient to answer the question and additional information is needed to find the correct answer.

Read the passage to answer the question 4-5 A map representing countries R, S, W, X, Y and Z is to be drawn. Adjacent countries cannot have the same color in the map. The countries adjacent to each other are as follows: Each of R, S, X and Y is adjacent to W. X is adjacent to Y. Student Guide

146

Each of R and S is adjacent to Z. 4. Which of the following countries can be the same color as W? A. B. C. D. A S X Y Z B C D E

5. Which of the following is a pair of countries that can be the same color? A. B. C. D. R and S S and W W and X X and Y

Questions 6 to 11 depends on the following passage A college president wishes to select four members of a faculty-student committee as special representatives to meet with the college's board of trustees. The faculty-student committee consists of eight members four of which (F, G, H and I) are faculty members whereas the other four (R, S, T and U) are students. The president can select any four of the eight committee members as long as the following rules are observed: The four representatives must consist of exactly two faculty members and two students. Either F or G must be one of the representatives but F and G both cannot be the representatives. If R is a representative then H must also be a representative. If T is a representative then G cannot be a representative. 6. If T is a representative but H is not a representative then the whole group can be determined if it were also true that: A. B. C. D. F is a representa tive. I is a representative. R is not a representative. U is not a representative.

7. If R is a representative then which of the following CANNOT be a representative? A. B. C. D. H I S T

8. If G is a representative then which of the following can be the o ther three representatives? A. B. C. D. F, S and U H, I and R H, R and S I, R and U

Student Guide

147

9. If neither S nor U is a representative then which of the following is the pair of facultymember representatives? A. B. C. D. F and G F and H F and I G and H

10. If G, I and S are representatives then which of the following must also be a representative? A. B. C. D. H R T U

11. If F and I are representatives then which of the following is not a representative? A. B. C. D. I S U R

Questions 12 to 14 depends on the following passage

At a congress of the Ruling Party, the seven top party leaders, who are all cabinet ministers, are seated on a platform in order of rank the Prime Minister being in the center. The closer a person is to the Prime Minister; the higher is his/her rank. Moreover, a person sitting on the right of the PM outranks the one sitting equidistant on the left of the PM. The seven leaders are T, U, V, W, X, Y, and Z. Y is four places to the left of the Minister of Agriculture, who is two places to the right of V. Us neighbors are T and the Minister of Agriculture. Z is two places to the left of W. The Ministers of Education, Mining and Culture are seated together, in order, from left to right. The remaining Ministers are those of Social Welfare and Defense. 12. The fifth ranking person in the party hierarchy is: A. Z, the Minister of Mining B. Y, the Minister of Culture C. W, the Prime Minister. D. X, the minister of Defense. A B C D E

13. How many of the seven party leaders outrank the Minister of Education? Student Guide

148

A. B. C. D. A

3 4 5 6 B C D E

14. The lowest ranking Minister is A. Minister of Social Welfare. B. Minister of Defense. C. Minister of Education. D. Minister of Mining. A B C D E

15. A meadow in springtime is beautiful, even if no one is there to appreciate it. This statement would be a logical opposite to which of the following claims? A. People will see only what they want to see. B. Beauty exits only in the eyes of the beholder. C. Beauty does not depend on seasons. D. The greatest pleasure available to mankind is the contemplation of beauty. A B C D E

Student Guide

149

III VERBAL Section No. Of Questions 10

Choose the correct answer for each question and shade the corresponding CIRCLE in the answer sheet

Each sentence below has one or two blanks, each blank indicates that something has been omitted. Beneath the sentence are five lettered words or sets of words. Choose the word or set of words that, when inserted in the sentence, best fits the meaning of the sentence as a whole. 1. Some illnesses such as smallpox, which have been almost eliminated in the United States are still ____ in many places abroad. A. discussed B. prevalent C. scarce D. unknown E. hospitalized A 2. B C D E

A recent study indicates that the crime rate in the United States remains ____ and that one in three households ____ some form of major crime in any year A. incredible ... witnesses B. astronomical ... experiences C. simultaneous ... perpetrates D. unsuccessful ... initiates E. defeated ... prosecutes A B C D E

Each question below consists of a related pair of words or phrases, followed by five lettered pairs of words or phrases. Select the lettered pair that best expresses a relationship similar to that expressed in the original pair. 3. SALVAGE : TREASURE A. settle : argument B. incorporate : company C. send : correspondence D. rescue : victim E. recycle : newspaper A 4. B C D E

CONTROVERSY : ARBITRATOR A. peacemaker : conflict B. artifact : anthropologist C. game : referee D. dispute : mediator E. disease : pathologist A B C D E

Read the passages and answer the questions given at its end: Student Guide

150

We are profoundly ignorant about the origins of language and have to content ourselves with more or less plausible speculations. We do not even know for certain when language arose, but it seems likely that it goes back to the earliest history of man, perhaps half a million years. We have no direct evidence, but it seems probable that speech arose at the same time as tool making and the earliest forms of specifically human cooperation. In the great Ice Ages of the Pleistocene period, our earliest human ancestors established the Old Stone Age culture; they made flint tools and later tools of bone, ivory, and antler; they made fire and cooked their food; they hunted big game, often by methods that called for considerable cooperation and coordination. As their material culture gradually improved, they became artists and made carvings and engravings on bones and pebbles, and wonderful paintings of animals on the walls of caves. It is difficult to believe that the makers of these Paleolithic cultures lacked the power of speech. It is a long step Admittedly, from the earliest flint weapons to the splendid art of the late Old Stone Age: the first crude flints date back perhaps to 500,000 B.C., while the finest achievements of Old Stone Age man are later than 100,000 B.C.; and, in this period, we can envisage a corresponding development of language, from the most primitive and limited language of the earliest human groups to a fully developed language in the flowering time of Old Stone Age culture. How did language arise in the first place? There are many theories about this, based on various types of indirect evidence, such as the language of children, the language of primitive societies, the kinds o f changes that have taken place in languages in the course of recorded history, the behavior of higher animals like chimpanzees, and the behavior of people suffering from speech defects. These types of evidence may provide us with useful pointers, but they all suffer from limitations, and must be treated with caution. When we consider the language of children, we have to remember that their situations are quite different from that of our earliest human ancestors, because the child is growing up in an environment where there is already a fully developed language, and is surrounded by adults who use that language and are teaching it to him. For example, it has been shown that the earliest words used by children are mainly the names of things and people (Do ll, Spoon, Mummy): but, this does not prove that the earliest words of primitive man were also the names of things and people. When the child learns the name of an object, he may then use it to express his wishes or demands: Doll!: often means Give me my doll! Or Ive dropped my doll: pick it up for me!; the child is using language to get things done, and it is almost an accident of adult teaching that the words used to formulate the childs demands are mainly nouns, instead of words like Bring! Pick up!; and so on. 5. The main idea of this excerpt is (A) to provide evidence of the origin of language. (B) to present the need for language. (C) to discuss how early man communicated. (D) to present the culture of early man. (E) to narrate the story of English. A 6. B C D E

Theories of the origin of language include all of the following EXCEPT (A) changes occurring through the years. (B) the need to communicate. (C) language of children. (D) the first mans extensive vocabulary. (E) communication among primitive men.

Student Guide

151

A 7.

The purpose of the discussion of the word, Doll, is intended to (A) Trace the evolution of a noun. (B) Support the fact that naming things is most important. (C) Indicate how adults teach language to children. (D) Show the evolution of many meanings for one word. (E) Evince mans multiple uses of single words A B C D E

8.

The implication of the author regarding the early elements of language is that (A) There were specific real steps followed to develop our language. (B) Care must be exercised when exhuming what we consider the roots of language. (C) We owe a debt of gratitude to the chimpanzee contribution. (D) Adults created language in order to instruct their children. (E) Language was fully developed by primitive man. A B C D E

9.

If we accept that primitive man existed for a very long period of time without language, then we may assume that (A) language is not necessary to mans existence. (B) language developed with the developing culture of primitives. (C) primitives existed in total isolation from one another. (D) children brought about a need for language. (E) mankind was not intended to communicate. A B C D E

10. After a (A) (B) (C) (D) (E) A

reading of this article, one might infer that society creates problems with language. language is for adults to instruct children. society uses language to improve itself. with the evolution of language came wisdom. language brings power. B C D E

Student Guide

152

Answer Keys to Drill Tests

Student Guide

153

DRILL TEST I - ANSWER KEY Section- I Quantitative 1. 2. 3. 4. 5. 6. 7. 8. 9. 10. A A A A A A A A B B B B B B C C C D D D B B C C C C E E E

D D D D D D

15. 16. 17. 18.

A A A A A A

C C D

E E E D E E

B B

E E E E E E

19. 20.

C B C

D D

Section- III 1. 2. A A A A B A B B B B

Verbal D D C C C C C C D D D D D E E E E E E E E E D D C C C C C D D D D D E E E E E E E E E E D E

Section- II Analytical Reasoning 1. 2. 3. 4. 5. 6. 7. 8. 9. A A A B B B B B C B C

3. 4. 5. 6. 7. 8.

A A A A A B B B B B

C C C C

E E

D D D E E E E E D D D D D E E E

A A A A B B B B B A A A A A A B B B B

9. 10. 11. 12. 13. 14. 15. 16. 17. 18. 19. 20.

C C C C C

D D

10. A 11. A 12. A 13. A 14. A

C C C C

Student Guide

154

DRILL TEST II - ANSWER KEY Section- I Quantitative 1. 2. 3. 4. 5. 6. 7. 8. 9. 10. A A A A A A A A A A B B C B B C C D D B B C C C C C D D D E E E E E E Section- III 1. 2. 3. 4. 1. 2. 3. 4. 5. 6. 7. 8. 9. 10. 11. 12. 13. 14. A A A A A A A A A A B B B B B B B B B B B C C C C D D D E C C C A B C C C D D D D D D D D D D E E E E E E E E E 5. 6. 7. 8. 9. 10. 11. 12. 13. 14. 15. 16. 17. 18. 19. 20. A A A A A A A A A A A A A A A A B B B B B C C B B B C C C B B C C A A B B B B C C D D D D D D D D D D D E E E E E E C B C C E E E

D D

15. 16. 17. 18. 19. 20. A A A A A

B B B

D D

E E E

C C D

E E

B B

C C D

Verbal D D D D D D D E E E E E E E E E E E E

Section- II Analytical Reasoning

Student Guide

155

DRILL TEST III - ANSWER KEY Section- I Quantitative 1. 2. 3. 4. 5. 6. 7. 8. 9. 10. 11. 12. 13. 14. 15. 16. 17. 18. 19. 20. 1. 2. 3. 4. 5. 6. 7. 8. 9. 10.
A A A A A A A A A A A A A A A A A A A A B B B B B B B B B B B B B B B B B B B B C C C C C C C C C C C C C C C C C C C C D D D D D D D D D D D D D D D D D D D D E E E E E E E E E E E E E E E E E E E E

11. 12. 13. 14. 15.

A A A A A

B B B B B

C C C C C

D D D D D

E E E E E

Section- III Verbal 1. 2. 3. 4. 5. 6. 7. 8. 9. 10.


A A A A A A A A A A B B B B B B B B B B C C C C C C C C C C D D D D D D D D D D E E E E E E E E E E

Section- II Analytical Reasoning


A A A A A A A A A A B B B B B B B B B B C C C C C C C C C C D D D D D D D D D D E E E E E E E E E E

Student Guide

156

DRILL TEST IV - ANSWER KEY Section- I Quantitative 1. 2. 3. 4. 5. 6. 7. 8. 9. 10. 11. 12. 13. 14. 15. 16. 17. 18. 19. 20. 1. 2. 3. 4. 5. 6. 7. 8. 9. 10.
A A A A A A A A A A A A A A A A A A A A B B B B B B B B B B B B B B B B B B B B C C C C C C C C C C C C C C C C C C C C D D D D D D D D D D D D D D D D D D D D E E E E E E E E E E E E E E E E E E E E

11. 12. 13. 14. 15.

A A A A A

B B B B B

C C C C C

D D D D D

E E E E E

Section- III Verbal 1. 2. 3. 4. 5. 6. 7. 8. 9. 10.


A A A A A A A A A A B B B B B B B B B B C C C C C C C C C C D D D D D D D D D D E E E E E E E E E E

Section- II Analytical Reasoning


A A A A A A A A A A B B B B B B B B B B C C C C C C C C C C D D D D D D D D D D E E E E E E E E E E

Student Guide

157

SAMPLE TEST General

Note: The Sample Test does not include quantitatively the same number of questions as there would be in the actual papers. They are merely meant to provide conceptual guidance to the users or prospective candidates.

Student Guide

158

I VERBAL Section No. Of Questions 15

Choose the correct answer for each question and shade the corresponding CIRCLE in the answer sheet

Each sentence below has one or two blanks, each blank indicates that something has been omitted. Beneath the sentence are five lettered words or sets of words. Choose the word or set of words that, when inserted in the sentence, best fits the meaning of the sentence as a whole. 1. Despite the millions of dollars spent on improvements, the telephone system in India remains ________ and continues to ___________ the citizens who depend upon it. A. B. C. D. E. Primitiveinconvenience Bombastic...upset Suspicious...connect Outdated...elate Impartial...vex

2. Unlike the images in symbolist poetry which are often vague and _______ , the images of surrealist poetry are startlingly ________ and bold. A. B. C. D. E. extraneous...furtive trivial...inadvertent obscure...concrete spectacular...pallid symmetricalvirulent

3. A good trial lawyer will argue only what is central to an issue, eliminating ___________ information or anything else that might __________ the client. A. B. C. D. E. Seminal...amuse Extraneous...jeopardize Erratic...enhance Prodigious...extol Reprehensibleinitiate

4. Pollen grains and spores that are 200 millions old are now being extracted from shale and are ____________ the theory that the breakup of the continents occurred in stages; in fact, it seems that the breakups occurred almost __________ . A. B. C. D. E. refining...blatantly reshaping...simultaneously countermanding...imperceptibly forging...vicariously supporting...haphazardly

Student Guide

159

Each question below consists of a related pair of words or phrases, followed by five lettered pairs of words or phrases. Select the lettered pair that best expresses a relationship similar to that expressed in the original pair. 5. DETENTION : RELEASE :: A. B. C. D. E. viciousness : attack calamity : repair qualification : employ induction : discharge therapy : confuse

6. PONDEROUS : WEIGHT :: A. B. C. D. E. eternal : temporality convincing : decision gargantuan : size ancient : value prototypical : affection

7. FEBRILE : ILLNESS :: A. B. C. D. E. 8. tenacious : astonishment juvenile : maturity classic : cultivation eccentric : discrimination delusional : insanity

EQUIVOCATION : MEANING :: A. B. C. D. E. feint : intention secrecy : stealth geniality : amiability travesty : insight refinement : innovation

Choose the lettered word or phrase that is most nearly opposite in meaning to the word in capital letters. 9. WHIMSICAL : A. B. C. D. E. chivalrous perfect predictable hidden backward

10. REVERE : A. B. C. D. E. Student collide succumb threaten divide despise Guide

160

11. INURED : A. B. C. D. E. authoritative dissolute bereft sensitive taxing

12. ALACRITY : A. B. C. D. E. skullduggery reluctance interment bellicosity specificity

Read the passages and answer the questions asked at its end. Art, like words, is a form of communication. Words, spoken and written, render accessible to humans of the latest generations all the knowledge discovered by the experience and reflection, both of preceding generations and of the best and foremost minds of their own times. Art renders accessible to people of the latest generations all the feelings experienced by their predecessors, and those already felt by their best and foremost contemporaries. Just as the evolution of knowledge proceeds by dislodging and replacing that which is mistaken, so too the evolution of feeling proceeds through art. Feelings less kind and less necessary for the well-being of humankind are replaced by others kinder and more essential to that end. This is the purpose of art, and the more art fulfills that purpose the better the art; the less it fulfills it, the worse the art. 13. The author develops the passage primarily by A. B. C. D. E. theory and refutation example and generalization comparison and contrast question and answer inference and deduction

14. According to the author, knowledge is A. B. C. D. E. evolutionary and emotional cumulative and progressive static and unmoving dynamic and cyclical practical and directionless

15. According to the passage, all of the following are true EXCEPT: A. B. C. D. E. Art is a form of communication. Art helps to refine sensibilities. Art is a repository of experience. Real art can never be bad. Art is a progressive human endeavor.

Student Guide

161

II Analytical Reasoning Section No. Of Questions 20

Choose the correct answer for each question and shade the corresponding CIRCLE in the answer sheet

Questions 16-19 are based on the following. The Western Derby is a race held annually at Bayshore Racetrack. There are eight gates at the racetrack, but only seven horses are entered in this raceJulius Caesar, King's Bounty, Longshot, Man Among Boys, Nocturnal, Odyssey, and Phantom. One of the gates is left empty. The horses are at the gate, waiting for the race to begin. Gate 1, on the inside of the racetrack, is occupied. Phantom is at a gate inside of Nocturnal. The number of gates separating Julius Caesar and King's Bounty equals the number of gates separating Longshot and Man among Boys. Nocturnal and Odyssey are next to each other.

16. If Odyssey is at Gate 2, which of the following must be true? A. B. C. D. E. Nocturnal is at the innermost gate. King's Bounty is at the outermost gate. A horse occupies the outermost gate. Phantom is at the innermost gate. The outermost gate is not empty.

17. Which of the following is a possible assignment for the horses, from the inside to the outside? A. B. C. D. E. Phantom, King's Bounty, Julius Caesar, Odyssey, Nocturnal, Man Among Boys, Longshot, vacant vacant, Phantom, Julius Caesar, Longshot, King's Bounty, Man Among Boys, Nocturnal, Odyssey Longshot, Man Among Boys, Nocturnal, vacant, Phantom, Odyssey, King's Bounty, Julius Caesar Julius Caesar, King's Bounty, Longshot, Phantom, vacant, Man Among Boys, Nocturnal, Odyssey Phantom, Julius Caesar, Nocturnal, vacant, Odyssey, King's Bounty, Longshot, Man Among Boys

18. If Julius Caesar is at Gate 6, King's Bounty is at Gate 7, and Odyssey is at Gate 4, which of the following must be true? I. Longshot is at Gate 1. II. Nocturnal is at Gate 5. III. Man Among Boys is at Gate 2. IV. Gate 8 is vacant.

Building Standards in Education and Professional Testing

162

A. B. C. D. E. 19.

I and II only II and III only II and IV only I, II, and III only I, II, III, and IV

If Julius Caesar and King's Bounty are at the second and fourth gates, respectively, all of the following can be true EXCEPT A. B. C. D. E. Phantom is at Gate 1 Man Among Boys is at Gate 3 Longshot is at Gate 6 Odyssey is at Gate 7 Nocturnal is at Gate 7

20. Studies have shown that families who install smoke detectors and own fire extinguishers have a reduced risk of losing a child in a house fire. Therefore, no family who installs smoke detectors and owns a fire extinguisher will lose a child in a house fire. Of the following, the best criticism of the argument above is that the argument does not A. B. C. D. E. take into account the possibility of losing a child in a house fire despite all precautionary measures indicate that fire extinguishers are effective during early stages of a fire cite the fact that smoke detectors have proven to be effective in waking sleeping children during a house fire differentiate between the two major causes of house fires: cooking and heating take into account that families who buy smoke detectors are also more likely to purchase fire insurance

21. LSD is a drug known to cause synesthesia, a phenomenon in which sensory input somehow becomes interchanged in the brain: a person with synesthesia might smell a symphony, hear sun light, or taste a pinprick. While most cases are drug induced, some people suffer from synesthesia in various forms since birth. Which of the following can be most safely inferred from the information above?

A. B. C. D. E.

Synesthesia is not always a drug-induced phenomenon. Some great artists of this century have been known for their synesthetic proclivities. LSD is an addictive drug. Synesthesia is rarely bothersome to those who experience it. Synesthesia at birth is a result of mothers who have tried LSD.

22. Palindromes are easier to solve than acrostics, but acrostics are more difficult to create than palindromes. Rebuses are more difficult to solve than acrostics, yet rebuses are easier to create than palindromes.

Building Standards in Education and Professional Testing

163

If the above information is true, then it must also be true that A. B. C. D. E. acrostics are more difficult to create than rebuses palindromes are more difficult to solve than rebuses rebuses are easier to solve than acrostics acrostics are easier to create than rebuses rebuses are easier to solve than palindromes

Questions 23-25 are based on the following. A university has a procedure for registering and recording complaints. Due to strict bureaucratic regulations, the following system of passing complaints must be observed: A is the first registrar to receive all incoming complaints. F is the recorder and final administrator to handle a complaint. Personnel B, C, D, and E may pass complaints only as follows: A to B B to either C or D C to either B or E D to C E to either D or F 23. Which is an acceptable path for a complaint to follow, passing from A? A. B. C. D. E. B to B to B to B to D to C to D to F D to C to F C to E to F E to F C to F

24. If a complaint is received and is handled by each personnel member only one time, which of the following could be one of the passes? A. B. C. D. E. A to C to C to D to E to C B F C D

25. Between which two personnel may a complaint pass by means of two different paths without any duplication of passes? A. B. C. D. E. B to C to C to D to E to E D E B B

Building Standards in Education and Professional Testing

164

Questions 26-31 are based on the following. In a baseball field, one team can practice at a time. There are seven teams the Aces, the Bears, the Cubs, the Ducks, the Eagles, the Falcons, and the Giants. The baseball field is open seven eve nings a week from Monday to Sunday (Sunday being considered the last day of the week), and the allocation of practice times is governed by the following rules: On any evening, only one team can play. The Aces must practice on Monday. The Ducks practice exactly one day before the Falcons practice. The Falcons practice exactly one day before the Giants practice. The Cubs and the Bears must practice earlier in the week than the Eagles. 26. The latest day in the week that the Bears can practice is A. B. C. D. E. Tuesday Wednesday Thursday Friday Saturday

27. If a person went to the baseball field on three consecutive evenings, he or she could see which of the following teams in the order listed? A. B. C. D. E. the the the the the Falcons, the Giants, the Cubs Falcons, the Giants, the Ducks Aces, the Ducks, the Cubs Bears, the Cubs, the Falcons Ducks, the Eagles, the Falcons

28. One week, the Cubs practiced on Wednesday and the Ducks practiced the next day. That week, the Bears must have practiced on A. B. C. D. E. Monday Tuesday Friday Saturday Sunday

29. If the Giants practice on Thursday, the Eagles and the Ducks must practice on which days, respectively? A. B. C. D. E. Sunday and Tuesday Saturday and Tuesday Friday and Wednesday Wednesday and Thursday Tuesday and Monday

30. If the Falcons practice on Saturday, the Eagles must practice on what day? A. Tuesday

Building Standards in Education and Professional Testing

165

B. C. D. E.

Wednesday Thursday Friday Sunday

31. The practice schedule has to adhere to which of the following? A. B. C. D. E. The The The The The Ducks practice earlier in the week than the Eagles. Falcons practice on a later day than the Eagles. Falcons practice earlier in the week than the Giants. Cubs practice earlier in the week than the Ducks. Bears practice earlier in the week than the Cubs.

32. Wine, cheese, butter, and raisins are all examples of early techniques to preserve food. In modern times, food scientists have developed other techniques such as dehydration, hermetic sealing, and radiation. Of these, radiation is the most controversial because preliminary studies have shown that radiation alters the natural chemical bonds in fruits and vegetables. Instead of providing salutary effects, eating radiated produce may well introduce irritating chemicals into the body, creating a possible health hazard. Which of the following, if true, supports the conclusion that eating radiated produce poses a possible health hazard? A. Radiation affects only those chemical bonds associated with water, that is, hydrogen and oxygen. B. Radiation kills microorganisms that hasten food decay. C. The radiation-induced bonds are unlike any of those found in non-radiated produce. D. Certain microorganisms, namely those found in yogurt cultures, are essential for proper digestion. E. Radiation has no effect on foods preserved by drying. 33. Blue Blood, Inc., is a private blood products company that buys blood only from qualified donors. To qualify, a person must weigh at least 105 pounds, must not have taken malaria medication in the last three years, must never have had hepatitis, and must never have used intravenous drugs. Blue Blood nurses know that traveling has an effect on the possibilities for blood donation: Everyone who travels to Malaysia is required to take malaria medication; no one who enters Singapore can have ever used intravenous drugs; everyone traveling to Gorisimi gets hepatitis. Which of the following situations would not au tomatically disqualify a person from selling blood to Blue Blood? A. B. C. D. E. traveling to Malaysia two years ago having once weighed 110 pounds and now weighing 95 pounds being denied admission to Singapore traveling to Gorisimi five years ago using intravenous drugs that were legal at the time

Building Standards in Education and Professional Testing

166

34. Before marriage, couples should be tested for AIDS and any other sexually communicable diseases. Negative results will guarantee the health and safe-ness of their m arriage. Which of the following is an assumption of the argument in the passage above ? A. Current state laws require couples who are planning to get married to be tested for infectious disease in order to prevent possible health problems in the future. There are many infectious diseases that can be sexually transmitted from one individual to another. Fortunately even if a test proves positive for a communicable disease, couples can still lead healthy marriages by taking the proper precautions. Due to advances in medical research over the years, infectious diseases that used to be fatal can now be effectively treated. All the diseases detectable through testing have no incubation period and the results of these tests can immediately indicate whether or not the individual has the disease.

B. C. D. E.

Question 35 is based on the following . Nine athletes attend a sports banquet. Three of the athletes}, K, and Lare varsity football players; two of the athletesM and Nare varsity basketball players. The other four athletes O, P, Q, and Rbelong to the hockey club. All nine athletes will be seated at three small tables, each seating three athletes. The athletes must be seated according to the following rules: O and J do not sit at the same table. P sits together with at le ast one of K or M. There can be at most only one football player at a table. There can be at most only one basketball player at a table. 35. Suppose just one varsity athlete sits at a certain table, and that athlete happens to be J. If so, who else sits with J? A. B. C. D. E. P, Q P, R Q, R O, Q O, P

Building Standards in Education and Professional Testing

167

III Quantitative Section

No of Questions

15

Choose the correct answer for each question and shade the corresponding CIRCLE in the answer sheet

36. If vy = 9, then y2 - vy = A. B. C. D. E. v 3-9 0 9-v 3 6552 6561

37. If (x+3)/6 = 12/(x+4), what is the positive value of x? A. B. C. D. E. 2 3 5 v60 12

38. Cindy wants to paint her office. She can buy three cans of the same -priced paint and three identical brushes for $21, or she can buy four cans of the same paint and one brush for $22. How much does a can of paint cost? A. B. C. D. E. $2 $3 $4 $5 $6

39. Which of the following must be true? I . (25 - 81) = (5 - 9) (5 + 9) II. 7(9 + 6) = 7(9) + 7(6) I I I . 6 (3 - 1) = (6 3) - ( 6 1 ) A. B. C. D. E. I only II only III only I and II only I, II, and III

40. The sum of a and 9 - 2a is less than 8. Which of the following is (are) the value(s) of a? I. II. a <- 1 a< 1

Building Standards in Education and Professional Testing

168

III. a>1 A. B. C. D. E. I only II only III only I and II only I and III only

41. Susan is having a party. At 7:00 P.M., guests begin arriving at a uniform rate of 8 people every 15 minutes. If this pattern continues, how many guests will have arrived by 9:30 P.M.? F. G. H. I. J. 10 20 40 64 80

42. For positive integers p and q, if p2 + 2q2 = 41, and 2p2 +q2 = 34, then p2 = A. B. C. D. E. 2.5 7 3 9 16

43. If a:b is 7:6 and 3b:2c is 2:3, what is c/a ? A. B. C. D. E. 14/27 7/9 6/7 9/7 27/14

Building Standards in Education and Professional Testing

169

44. In the figure above, if the radius of the circle is 8, and triangle TRS is inscribed in the circle, then the length of arc TRS is A. B. C. D. E. 16p/3 (32 p)/3 16 p (128 p)/3 64 p

45. For developing pictures, XYZ Photo Lab charges a service fee of $3 for every order it receives in addition to a printing fee. If the order consists of 12 pictures or less, the printing fee per picture is $0.36. If the order consists of more than 12 pic tures, the printing fee per picture is $0.24. What is the total cost per picture for an order consisting of 30 pictures? A. B. C. D. E. $0.11 $0.24 $0.34 $0.46 $3.24

46. Lisa found an easy way to add up a sequence of positive even integers with an even number of terms. She formed pairs of equal sums by adding the first integer to the last, the second integer to the next-to-last, and so on. She then computed the total by adding these equal sums. If the total Lisa obtained was 930, how many terms were there in the sequence of positive even integers if the sequence started with the number 2? A. B. C. D. E. 30 39 40 60 465

47. December is the busiest month at Lamo nt's Gift Shoppe, where sales in December are 40 percent higher than average. If sales in February are typically 20 percent lower than average, what is the ratio of February sales to December sales? A. B. C. D. E. 1:2 4:2 4:5 4:7 6:7

48. How many 4-digit numbers are there that consist of only odd digits? A. B. C. D. E. 20 625 1,024 4,500 5,000

Building Standards in Education and Professional Testing

170

49. For some integer m, let [m] be defined by the equation {m} = m (1- m). If n + 1 = {n + 1}, then n = A. B. C. D. E. -2 -1 0 1 2

50. Box A and box B have 6 cards each. Each card is marked with one integer, 1 through 6. Both boxes can have more than one card with the same integer, but the sum of all the integers in each box must be 18. Two of the cards in box/1 are 6's and two of the cards in box B are 5's. If one card is drawn from box A and one from box B, but neither a 6 nor a 5 is drawn, what is the largest possible sum of the integers on the cards drawn from the two boxes? A. B. C. D. E. 3 4 7 8 12

Building Standards in Education and Professional Testing

171

Вам также может понравиться